Круги эйлера объяснение: описание, примеры, для дошкольников, для школьников

Содержание

Круги Эйлера в информатике

Сегодня разберём задачи на круги Эйлера в информатике.

Леонард Эйлер — швейцарский, немецкий и российский математик и механик, сыгравший огромную роль в развитии этих наук.

Задача (Простая)

В языке запросов поискового сервера для обозначения логической операции «ИЛИ» используется символ «|», а для логической операции «И» — символ «&». В таблице приведены запросы и количество найденных по ним страниц некоторого сегмента сети Интернет.


Запрос Найдено страниц (в тысячах)
Пушкин 3500
Лермонтов 2000
Пушкин | Лермонтов 4500

Какое количество страниц (в тысячах) будет найдено по запросу Пушкин & Лермонтов? Считается, что все запросы выполнялись практически одновременно, так что набор страниц, содержащих все искомые слова, не изменялся за время выполнения запросов.

Решение:

Видим, что по запросу «Пушкин» в поисковике нашлось 3500 страниц. По запросу «Лермонтов» — 2000 страниц.

Запрос «Пушкин | Лермонтов» обозначает, что поисковик выдаст страницы, где есть слова про «Пушкина», и страницы, где есть слова про «Лермонтова», а так же могут быть страницы, где написано и про «Пушкина», и про «Лермонтова» одновременно.

Если сложить страницы, в которых написано про «Пушкина» и про «Лермонтова» получается 3500 + 2000 = 5500 страниц. Но почему же при запросе «Пушкин | Лермонтов» получается меньше страниц, всего 4500 ?

Этот факт обозначает то, что когда мы подсчитывали страницы про «Пушкина» (3500 страниц), мы подсчитали и те страницы, где было написано и про «Пушкина», и про «Лермонтова» одновременно.

Тоже самое и для количества страниц, где написано про «Лермонтова» (2000 страниц). В этом числе находятся и те, в которых одновременно упоминается и про «Пушкина», и про «Лермонтова».

В вопросе спрашивается, сколько страниц будет по запросу «Пушкин & Лермонтов«. Это обозначает, что как раз нужно найти количество страниц, где будет одновременно написано и про «Пушкина», и про «Лермонтова».

Отсюда получается:


Пушкин & Лермонтов = (3500 + 2000) — 4500 = 5500 — 4500 = 1000 страниц.

Это и будет ответ!


Теперь решим эту задачу с помощью Кругов Эйлера!

У нас всего есть две сущности: «Пушкин» и «Лермонтов». Поэтому рисуем два пересекающихся круга, желательно разными цветами.

Объединение двух кругов в общую фигуру (показано фиолетовым цветом), показывает операцию «Пушкин | Лермонтов». Эта операция всегда стремится увеличить площадь, объединить площади других фигур!

Обратите внимание, что круги пересекаются, из-за этого сумма площадей двух кругов по отдельности (3500 + 2000 = 5500) больше чем у фигуры, которая характеризует логическую операцию «ИЛИ» «Пушкин | Лермонтов» (4500).

Нужно найти площадь фигуры Пушкин & Лермонтов, которая закрашена золотистым цветом. Данная логическая операция «И» стремится уменьшить площадь. Она обозначает общую площадь других фигур.

Найдём сначала заштрихованную часть синего круга. Она равна: площадь фиолетовой фигуры (4500) минус площадь красного круга (3500).


Теперь легко найти площадь золотистой фигуры. Для этого нужно от площади синего круга вычесть площадь заштрихованной части. Получается:


Пушкин & Лермонтов (Количество страниц) = 2000 — 1000 = 1000

Получается, что по запросу Пушкин & Лермонтов будет найдено 1000 страниц.

Ответ: 1000

Рассмотрим ещё одну не сложную разминочную задачу.

Задача (Разминочная)

В языке запросов поискового сервера для обозначения логической операции «ИЛИ» используется символ «|», а для логической операции «И» – символ «&».

В таблице приведены запросы и количество найденных по ним страниц некоторого сегмента сети Интернет.


Запрос Найдено страниц (в тысячах)
Кокос | Ананас 3400
Кокос & Ананас 900
Кокос 2100

Какое количество страниц (в тысячах) будет найдено по запросу Ананас?

Считается, что все запросы выполнялись практически одновременно, так что набор страниц, содержащих все искомые слова, не изменялся за время выполнения запросов.

Решение:

У нас две сущности: Кокос и Ананас. Нарисуем два круга Эйлера, которые пересекаются между собой. Так же отменим все имеющееся данные.


Найдём заштрихованную часть красного круга.

Весь красный круг 2100. Золотистая область равна 900. Заштрихованная часть равна 2100 — 900 = 1200.


После того, как нашли заштрихованную часть (такой полумесяц), можно найти уже площадь синего круга. Для этого нужно от площади фиолетовой фигуры отнять площадь заштрихованной части!


Ананас (Количество страниц) = 3400 — 1200 = 2200

Ответ: 2200

Разберём классическую задачу из информатики по кругам Эйлера.

Задача (Классическая)

В языке запросов поискового сервера для обозначения логической операции «ИЛИ» используется символ «|», а для логической операции «И» — символ «&».

В таблице приведены запросы и количество найденных по ним страниц некоторого сегмента сети Интернет.


Запрос Найдено страниц (в тысячах)
(Космос & Звезда) | (Космос & Планета) 1100
Космос & Планета 600
Космос & Планета & Звезда 50

Какое количество страниц (в тыс. ) будет найдено по запросу Космос & Звезда?

Считается, что все запросы выполнялись практически одновременно, так что набор страниц, содержащих все искомые слова, не изменялся за время выполнения запросов.

Решение:

В этой задаче у нас три сущности: Космос, Планета, Звезда. Поэтому рисуем три круга Эйлера, которые пересекаются между собой.

Могут ли круги не пересекаться ? Могут! Если мы докажем, что площади по отдельности двух кругов в сумме дают площадь фигуры, которая получается при применении операции логического «ИЛИ».


Теперь отметим на нашем рисунке запрос (Космос & Звезда) | (Космос & Планета).

Сначала отменим для себя то, что находится в скобках. Первое Космос & Звезда


Теперь отметим вторую скобку Космос & Планета.


В выражении (Космос & Звезда) | (Космос & Планета) две скобки соединяет знак логического «ИЛИ».

Значит, эти две области нужно объединить! Область (Космос & Звезда) | (Космос & Планета) отмечена фиолетовым цветом!


Отметим Космос & Планета ещё раз, т.к. для этого выражения известно количество страниц.


Площадь фигуры для выражения Космос & Планета & Звезда будет очень маленькая. Это общая часть для всех трёх кругов. Отметим её оранжевым цветом! Каждая точка этой фигуры должна одновременно быть в трёх кругах!


Найти нужно Космос & Звезда. Отменим на рисунке чёрным цветом ту область, которую нужно найти. Мы эту область уже отмечали салатовым цветом.


Теперь у нас есть все компоненты, чтобы решить эту задачу.

Найдём заштрихованную область.


Вся область Космос & Планета равна 600. А заштрихованная часть равна: область

Космос & Планета (600) минус оранжевая область (50).


Количество страниц в заштрихованной части = 600 — 50 = 550

Тогда черная область легко находится: фиолетовая область (1100) минус заштрихованная область (550).


Количество страниц (при запросе Космос & Звезда) = 1100 — 550 = 550

Ответ: 550

Закрепляем материал по задачам на Круги Эйлера.

Задача (На закрепление)

В языке запросов поискового сервера для обозначения логической операции «ИЛИ» используется символ «|», а для обозначения логической операции «И» – символ «&». В таблице приведены запросы и количество найденных по ним страниц некоторого сегмента сети Интернет.


Запрос Найдено страниц (в тысячах)
Море & Солнце 290
Море & Пляж 355
Море & (Пляж | Солнце) 465

Какое количество страниц (в тысячах) будет найдено по запросу Море & Пляж & Солнце? Считается, что все запросы выполнялись практически одновременно, так что набор страниц, содержащих все искомые слова, не изменялся за время выполнения запросов.

Решение:

В задаче используются три сущности: Море, Пляж, Солнце. Поэтому нарисуем три пересекающихся круга Эйлера.


Отметим все области для которых нам даны количество страниц.

В начале отметим Море & (Пляж | Солнце). Для начало нарисуем область, которая в скобках (Пляж | Солнце)

Теперь нужно очертить общую часть фиолетовой области и зелёного круга и получится Море & (Пляж | Солнце). Отметим оранжевым цветом.


Теперь отметим Море & Пляж.


Теперь отметим Море & Солнце.


Найти нужно ту область, которая получается в результате выделения общей части для всех трёх кругов! Обозначим её чёрным цветом!


Найдём заштрихованную область!



Количество страниц (в заштрихованной области) =
= Количество страниц (В оранжевой области) — Море & Солнце =
= 465 — 290 = 175

Чтобы найти искомую чёрную область, нужно из Море & Пляж (355) вычесть заштрихованную область (175).


Количество страниц (Море & Пляж & Солнце) =
= Море & Пляж (355) — Количество страниц (в заштрихованной области) 175 =
= 355 — 175 = 180

Ответ: 180

Решим ещё одну тренировочную задачу из информатики на Круги Эйлера.

Задача (с 4 сущностями)

В языке запросов поискового сервера для обозначения логической операции «ИЛИ» используется символ «|», а для обозначения логической операции «И» – символ «&».

В таблице приведены запросы и количество найденных по ним страниц некоторого сегмента сети Интернет.


Запрос Найдено страниц (в тысячах)
Англия & (Уэльс & Шотландия | Ирландия) 450
Англия & Уэльс & Шотландия 213
Англия & Уэльс & Шотландия & Ирландия 87

Какое количество страниц (в тысячах) будет найдено по запросу


Англия & Ирландия?

Считается, что все запросы выполнялись практически одновременно, так что набор страниц, содержащих все искомые слова, не изменялся за время выполнения запросов.

Решение:

Нужно нарисовать 4 пересекающихся круга. Сначала нарисуем три круга, как обычно, оставив немного места для четвёртого круга.


Четвёртый круг для Ирландии нужно нарисовать так, чтобы он проходил через область (Англия & Уэльс & Шотландия). Это нам подсказывает сама таблица, где есть количество страниц для Англия & Уэльс & Шотландия, а так же для Англия & Уэльс & Шотландия & Ирландия.


Нужно отметить на рисунке Англия & (Уэльс & Шотландия | Ирландия). Это будем делать, как всегда поэтапно.

Область Уэльс & Шотландия выглядит так:


Добавим к этой области Ирландию через логическое «ИЛИ». Получается область (Уэльс & Шотландия | Ирландия). Произошло объединение серой области и жёлтого круга!


Теперь нужно сделать операцию логического «И» получившийся области с «Англией». Тогда область Англия & (Уэльс & Шотландия | Ирландия) примет вид:


Т.е. это общее между предыдущем серым контуром и красным кругом!

Отметим Англия & Уэльс & Шотландия — это общая территория трёх кругов: Красного, Синего и Зелёного. Отмечено оранжевым цветом.


Отметим Англия & Уэльс & Шотландия & Ирландия — это общая территория четырёх кругов. Область получается ещё меньше. Если взять точку в этой области, то мы будем находится сразу в четырёх кругах одновременно. Отмечено фиолетовым цветом.

Отметим то, что нужно найти Англия & Ирландия чёрным цветом.


Искомую чёрную область легко найти, если из серой области вычесть кусочек, окрашенный в бирюзовый цвет!


Найдём, сколько страниц приходится на бирюзовый кусочек:


Количество страниц (для бирюзового кусочка) =
= Англия & Уэльс & Шотландия (213) — Англия & Уэльс & Шотландия & Ирландия (87) =
= 213 — 87 = 126

Найдём искомую чёрную область.


Количество станиц (для чёрной области) =
= Англия & (Уэльс & Шотландия | Ирландия) (450) — Количество (для бирюзового кусочка) =
450 — 126 = 324

Это и будет ответ!

Ответ: 324.

Разберём задачу из реального экзамена по информатике, которая была в 2019 году в Москве! (Сейчас в 2021 задачи не встречаются на Круги Эйлера)

Задача (ЕГЭ по информатике, 2019, Москва)

В таблице приведены запросы и количество страниц, которые нашёл поисковый сервер по этим запросам в некоторым сегменте Интернета:


Запрос Найдено страниц (в тысячах)
Суфле 450
Корзина 200
Эклер 490
Суфле & Корзина 70
Суфле & Эклер 160
Корзина & Эклер 0

Сколько страниц (в тысячах) будет найдено по запросу


Суфле | Корзина | Эклер

Решение:

Видим, что у нас три поисковых разных слова, поэтому будет три разных круга Эйлера!

Так же видим, что логическое «И» между словами Корзина и Эклер даёт 0 страниц. Это значит, что эти круги не пересекаются! Так же круги бы не пересекались, если бы операция логического «ИЛИ» совпадала бы с суммой этих кругов.


Видим, что Суфле имеет с двумя кругами пересечения, а Корзина и Эклер не пересекаются.

Отметим всё, что нам дано в условии.


Жёлтым цветом отмечено Суфле | Корзина | Эклер . Объединение всех трёх кругов. Это то, что нужно найти.


Искомая жёлтая фигура складывается из заштрихованных областей и красного круга! Площадь красного круга мы знаем. Нужно найти площади заштрихованных частей.

Левая заштрихованная область находится просто:


Количество страниц (лев. заштрих. область) =
= Эклер (490) — Суфле & Эклер (160) = 330

Так же найдём площадь правой заштрихованной области:


Количество страниц (прав. заштрих. область) =
= Корзина (200) — Суфле & Корзина (70) = 130

Теперь можно найти искомую жёлтую область

Количество страниц (Суфле | Корзина | Эклер) =
= Красный круг (450) + лев. заштрих. область (310) + прав. заштрих. область (130) =
= 450 + 330 + 130 = 910

Задача решена, можно писать ответ.

Ответ: 910

Разберём ещё одну задачу из реального ЕГЭ уже 2020 года

Задача (ЕГЭ по информатике, 2020, Москва)

В таблице приведены запросы и количество страниц, которые нашёл поисковый сервер по этим запросам в некоторым сегменте Интернета:


Запрос Найдено страниц (в тысячах)
Аврора 50
Крейсер 45
Заря 23
Аврора & Заря 9
Заря & Крейсер 0
Заря | Крейсер | Аврора 93

Сколько страниц (в тысячах) будет найдено по запросу


Аврора & Крейсер

Решение:

Количество страниц при запросе Заря & Крейсер равно нулю. Значит, эти два круга не будут пересекаться.


Нарисуем все данные на рисунке.

Нужно найти для начала заштрихованную правую часть.



Количество страниц (для двух заштрих. частей) =
З | К | А (93) — Красный круг (50) = 43

Левую заштрихованную область легко найти.


Количество страниц (для левой заштрих. части) =
Синий круг (23) — А & З (9) = 14

Тогда для правой заштрихованной области получается:


Колич. страниц (для правой заштрих. части) =
Колич. страниц (для двух заштрих. частей) (43) — Колич. страниц (для лев. заштрих. части) (14) =
= 43 — 14 = 29

Тогда искомую область легко найти:


Колич. страниц (А & K) =
Зелёный круг (45) — Колич. страниц (для правой заштрих. части) (29) =
45 — 29 = 16

Ответ: 16

На этом всё! Надеюсь, вы теперь будете с удовольствием решать задачи по информатике с помощью Кругов Эйлера.

Развитие детей. Как решать логические задачи с помощью кругов Эйлера? | Обучение

Круги Эйлера — это геометрическая схема. С ее помощью можно изобразить отношения между подмножествами (понятиями), для наглядного представления. Метод Эйлера является незаменимым при решении некоторых задач, а также упрощает рассуждения.

Леонард Эйлер был гениальным математиком, который умел применять математические приемы на практике. Он успешно использовал для решения различных задач идею изображения понятий и классов предметов в виде кругов. Впервые Эйлер их применил в письмах к немецкой принцессе. Он писал тогда, что «круги очень подходят для того, чтобы облегчить наши размышления». И действительно, с помощью этих диаграмм можно легко и наглядно решить задачи, для решения которых обычным способом понадобилось бы составление системы из нескольких уравнений, например, с тремя неизвестными.

Способ изображения понятий в виде кругов позволяет развивать воображение и логическое мышление не только детям, но и взрослым (конечно, для взрослых подойдут более сложные логические задачи). Начиная с 4−5 лет детям доступно решение простейших задач с кругами Эйлера, сначала с разъяснениями взрослых, а потом и самостоятельно. Овладение методом решения задач с помощью кругов Эйлера формирует у ребенка способность анализировать, сопоставлять, обобщать и группировать свои знания для более широкого применения.

Вот несколько задач для маленьких детей на логическое мышление:

Определить круги, которые подходят к описанию предмета. При этом желательно обратить внимание на те качества, которыми предмет обладает постоянно и которыми временно. Например, стеклянный стакан с соком всегда остается стеклянным, но сок в нем есть не всегда. Или существует какое-то обширное определение, которое включает в себя разные понятия, подобную классификацию тоже можно изобразить с помощью кругов Эйлера. Например, виолончель — это музыкальный инструмент, но не каждый музыкальный инструмент окажется виолончелью.

Определение круга, который не подходит к описанию предмета. Например, баранка — она круглая и вкусная, а определение зеленая к ней не подходит. Можно также придумать, какой предмет подойдет для пересечения другой пары кругов. Пример — круглая и зеленая может быть пуговица.

Определить предмет, который подходит под описание всех кругов. Для каждого круга выбирается какое-либо качество (например — сладкое, оранжевое, круглое). Ребенок должен назвать предмет, который одновременно соответствует всем этим описаниям (в данном примере подойдет апельсин), также можно спросить ребенка, какие предметы могут соответствовать двум описаниям из трех, то есть будут находиться на пересечении каждой пары кругов (например, сладкое и оранжевое — карамелька, оранжевое и круглое — мяч, круглое и сладкое — арбуз).

Для детей постарше можно предлагать варианты задач с вычислениями — от достаточно простых до совсем сложных. Причем самостоятельное придумывание этих задач для детей обеспечит родителям очень хорошую разминку для ума. Приведем два простых примера с диаграммами.

1. Из 27 пятиклассников все изучают иностранные языки — английский и немецкий. 12 изучают немецкий язык, а 19 — английский. Необходимо определить, сколько пятиклассников заняты изучением двух иностранных языков; сколько не изучают немецкий; сколько не изучают английский; сколько изучают только немецкий и только английский?

При этом первый вопрос задачи намекает в целом на путь к решению этой задачи, сообщая, что некоторые школьники изучают оба языка, и в этом случае использование схемы также упрощает понимание задачи детьми.

2. В одном доме в 45 квартирах есть домашние животные. При этом в 22 квартирах хозяева держат только кошек, а еще в 7 квартирах есть и кошка, и собака. Нужно узнать, в скольких квартирах находятся собаки, в скольких кошки, а в скольких нет кошки, но есть собака.

Задача, по сути, такая же, однако изменены исходные данные, сектор пересечения кругов известен, но нужно узнать информацию о каждом полном круге. Собаки находятся в числе квартир, оставшемся после вычитания из количества всех квартир с животными количества квартир только с кошками. Круг с общим числом кошек состоит из известных данных секторов «только кошек» и «кошек и собак», поэтому общее число кошек находится объединением сумм этих секторов. Последнее неизвестное находится соответственно. Определенно, значительно проще объяснить решение этой задачи с помощью кругов Эйлера.

Задачи, связанные с множествами, могут быть гораздо более сложными, причем чем более запутанными будут условия задачи, тем более очевидна рациональность применения диаграмм для ее решения. Конечно, иногда встречаются задачи, которые проще решить с помощью арифметических действий, поэтому, прежде чем приступить к решению, желательно проанализировать условия задачи.

Круги Эйлера имеют прикладное значение не только в решении школьных задач, ими также пользуются для усвоения и структуризации изучаемых материалов, конспектирования и добавления наглядности в некоторых обучающих курсах. Кстати, некоторые предлагают использовать круги Эйлера для того, чтобы сделать выбор в каком-нибудь вопросе, например, определиться с профессией.

Так что обязательно научите ребенка рисовать такие кружочки, это, несомненно, обернется пользой в развитии логического мышления, поможет решать задачи интересно и с пониманием происходящего.

Теги: развитие способностей, логическое мышление, задачи, решение, интеллект, обучение, логические задачи

Решение задач с помощью кругов Эйлера

В языке запросов поискового сервера для обозначения логической операции «ИЛИ» используется символ «|», а для логической операции «И» — символ «&».

В таблице приведены запросы и количество найденных по ним страниц некоторого сегмента сети Интернет.

Считается, что все запросы выполнялись практически одновременно, так что набор страниц, содержащих все искомые слова, не изменялся за время выполнения запросов.

Решение задачи №1

Для решения задачи отобразим множества Тортов и Пирогов в виде кругов Эйлера.

Обозначим каждый сектор отдельной буквой (А, Б,В).

Из условия задачи следует:

Торты │Пироги =  А+Б+В = 12000

Торты & Пироги = Б = 6500

Пироги = Б+В = 7700

Чтобы найти количество Тортов (Торты = А+Б), надо найти сектор А, для этого из общего множества (Торты│Пироги) отнимем множество Пироги.

Торты│Пироги – Пироги = А+Б+В-(Б+В) = А = 1200 – 7700 = 4300

Сектор А равен 4300, следовательно

Торты = А+Б = 4300+6500 = 10800


Задача №2

В языке запросов поискового сервера для обозначения логической операции «ИЛИ» используется символ «|», а для логической операции «И» — символ «&».

В таблице приведены запросы и количество найденных по ним страниц некоторого сегмента сети Интернет.

ЗапросНайдено страниц (в тысячах)
Пироженое & Выпечка5100
Пироженое9700
Пироженое | Выпечка14200

Какое количество страниц (в тысячах) будет найдено по запросу Выпечка?

Считается, что все запросы выполнялись практически одновременно, так что набор страниц, содержащих все искомые слова, не изменялся за время выполнения запросов.Решение задачи №2

Для решения задачи отобразим множестваПироженых и Выпечек в виде кругов Эйлера.

Обозначим каждый сектор отдельной буквой (А, Б,В).

Из условия задачи следует:

Пироженое & Выпечка = Б = 5100

Пироженое = А+Б = 9700

Пироженое │ Выпечка =  А+Б+В = 14200

Чтобы найти количество Выпечки (Выпечка = Б+В), надо найти сектор В, для этого из общего множества (Пироженое │ Выпечка ) отнимем множествоПироженое.

Пироженое │ Выпечка – Пироженное = А+Б+В-(А+Б) = В = 14200–9700 = 4500

Сектор В равен 4500, следовательно  Выпечка = Б + В = 4300+5100 = 9400


Задача №3
В таблице приведены запросы к поисковому серверу. Расположите номера запросов в порядке убывания количества страниц, которые найдет поисковый сервер по каждому запросу.
Для обозначения логической операции «ИЛИ» используется символ «|», а для логической операции «И» — символ «&».

1спаниели | (терьеры & овчарки)
2спаниели | овчарки
3спаниели | терьеры | овчарки
4терьеры | овчарки

Решение задачи №3

Представим множества овчарок, терьеров и спаниелей в виде кругов Эйлера, обозначим сектора буквами (А, Б, В, Г).

Преобразим условие задачи в виде суммы секторов:

спаниели │(терьеры & овчарки) = Г + Б

спаниели│овчарки = Г + Б + В

спаниели│терьеры│овчарки = А + Б + В + Г

терьеры & овчарки = Б

Из сумм секторов мы видим какой запрос выдал больше количества страниц.

Расположим номера запросов в порядке убывания количества страниц: 3 2 1 4


Задача №4

В таблице приведены запросы к поисковому серверу. Расположите номера запросов в порядке возврастания количества страниц, которые найдет поисковый сервер по каждому запросу.
Для обозначения логической операции «ИЛИ» используется символ «|», а для логической операции «И» — символ «&».

1барокко | классицизм | ампир
2барокко | классицизм & ампир
3классицизм & ампир
4барокко | классицизм

Решение задачи №4

Представим множества классицизм, ампир и классицизм в виде кругов Эйлера, обозначим сектора буквами (А, Б, В, Г).

Преобразим условие задачи в виде суммы секторов:

барокко│ классицизм │ампир = А + Б + В + Г
барокко │(классицизм & ампир) = Г + Б
классицизм & ампир = Б
барокко│ классицизм = Г + Б + А

Из сумм секторов мы видим какой запрос выдал больше количества страниц.

Расположим номера запросов в порядке возрастания количества страниц: 3 2 4 1


Задача №5В таблице приведены запросы к поисковому серверу. Расположите номера запросов в порядке возврастания количества страниц, которые найдет поисковый сервер по каждому запросу.
Для обозначения логической операции «ИЛИ» используется символ «|», а для логической операции «И» — символ «&».

1канарейки | терьеры | содержание
2канарейки & содержание
3канарейки & щеглы & содержание
4разведение & содержание & канарейки & щеглы

Решение задачи №5

Для решения задачи представим запросы в виде кругов Эйлера.

K —  канарейки,

Щ – щеглы,

С – содержание,

Р – разведение.

Далее будем закрашивать красным цветом сектора согласно запросам, наибольший по величине сектор даст большее количество страниц на запрос.


В порядке возрастания по количеству страниц запросы будут представлены в следующем порядке: 

4 3 2 1

Самая большая область закрашенных секторов у первого запроса, затем у второго, затем у третьего, а у четвертого запроса самый маленький.

Обратите внимание что в первом запросе закрашенные сектора кругов Эйлера содержат в себе закрашенные сектора второго запроса, а закрашенные сектора второго запроса содержат закрашенные сектора третьего запроса, закрашенные сектора третьего запроса содержат закрашенный сектор четвертого запроса.

Только при таких условиях мы можем быть уверены, что правильно решили задачу.  


Задачи для самостоятельного решения

Задача №6

В таблице приведены запросы к поисковому серверу. Расположите номера запросов в порядке возрастания количества страниц, которые найдет поисковый сервер по каждому запросу.
Для обозначения логической операции «ИЛИ» используется символ «|», а для логической операции «И» — символ «&».

1принтеры & сканеры & продажа
2принтеры  & продажа
3принтеры | продажа
4принтеры | сканеры | продажа

Задача №7

В таблице приведены запросы к поисковому серверу. Расположите номера запросов в порядке возрастания количества страниц, которые найдет поисковый сервер по каждому запросу.
Для обозначения логической операции «ИЛИ» используется символ «|», а для логической операции «И» — символ «&».

1физкультура
2физкультура & подтягивания & отжимания
3физкультура & подтягивания
4физкультура | фитнесс

Использование кругов Эйлера в русском языке

Использование кругов Эйлера в русском языке.

Что такое урок русского языка сегодня? Это не просто передача знаний, умений и навыков, ориентированных на усвоение учеником правил и норм русского языка, но и развитие творческого мышления учащихся.

Как же разнообразить уроки русского языка в современной школе? Какие приемы следует применять, чтобы материал усваивался и надолго оставался в памяти учеников? Все эти вопросы, несомненно, волнуют каждого учителя, заинтересованного в повышении интереса учащихся к учебному предмету и в повышении качества обучения.

Сегодня, наряду с активными формами обучения, широко востребованы и интерактивные, в ходе которого осуществляется взаимодействие между учеником и учителем, а так же между самими учениками. И это не случайно. Интерактивное обучение предполагает решение проблем, связанных с будущей профессиональной деятельностью учащихся, с человеческими взаимоотношениями, личными трудностями. Учебный процесс, опирающийся на использование интерактивных методов обучения, организуется с учетом включенности в процесс познания всех обучающихся класса без исключения.

И сегодня я бы хотела подробнее остановиться на методе кругов Эйлера и его применении на уроках русского языка в школе.

Леонардо Эйлера называют идеальным математиком 18 века. Он принадлежит к числу тех гениев, чьё творчество стало достоянием всего человечества. Эйлер за свою долгую жизнь написал более 850 научных работ. В одной из них и появились эти круги. До сих пор школьники всех стран изучают тригонометрию и логарифмы в том виде, какой придал им Эйлер. Студенты проходят высшую математику под руководством, первыми образцами которых явились классические монографии Эйлера. Более того, данный алгоритм может применяться и в гуманитарных науках, в частности, в русском языке.

Что же это за круги, которые способны охватить огромную сферу информации и способствовать удобному решению многих задач.

Круги Эйлера – это геометрическая схема, которая помогает находить и делать более наглядными логические связи между явлениями и понятиями, а также помогает изобразить отношения между каким-либо множеством и его частью.

Иными словами круги Эйлера – это тот метод, который наглядно демонстрирует: лучше один раз увидеть, чем сто раз услышать. Его заслуга в том, что наглядность упрощает рассуждения и помогает быстрее и проще получить ответ на поставленный вопрос, а главное усвоить материал.

Ведь как известно небольшой процент людей одарен феноменальной памятью, следовательно, не все учащиеся могут с легкостью усваивать большой объем учебного материала, если он логически не структурирован. Обычно человек обладает развитыми одним-двумя видами памяти.

Метод кругов Эйлера делает упор на зрительную память. Развивая этот вид памяти, ученики смогут хорошо запоминать учебный материал. Очень важный вид зрительной памяти – фотографическая память. Люди, обладающие такой памятью, запоминают все в мельчайших деталях. Некоторым дан этот вид памяти с рождения, но чаще всего фотографическая память нуждается в развитии с помощью специальных методик путем регулярных тренировок. Круги Леонардо Эйлера также являются одной из методик, способствующих развитию данного вида памяти.

Использование кругов Эйлера в русскому языке довольно популярен. Он способствует развитию умения сравнивать объекты, находить общее и различия в их строении, значении. Используют его при разных видах разбора:

  • лексическом (нахождение общего и различного в значении слов),

  • морфологическом (сравнение слов одной и той же части речи, а также разных частей речи, имеющих общие морфологические признаки),

  • синтаксическом (сравнение предложений разных по цели высказывания, составу, наличию или отсутствию второстепенных членов и т.п.).

Этот приём помогает ученикам разобраться в похожих лингвистических явлениях, помогают запоминанию структуры различных сочетаний мыслей и облегчают решение ряда задач, стоящих перед формальной логикой.

Известно, что с помощью эйлеровых кругов можно проверить истинность того или иного вида непосредственного умозаключения, основанного на сравнении.

Суть заданий с использованием приема “Круги Эйлера” заключается в следующем: круг предполагает наглядное изображение какого-нибудь понятия. Например, “Часть речи” – это круг.

Часть речи

Если мы предложим учащимся другой круг с надписью “Глагол”, то взаимное расположение этих кругов должно выглядеть так:

Часть речи

Глагол

Данный рисунок показывает, что все глаголы относятся к такому понятию, как “часть” речи.

Взаимное расположение кругов может быть разным: они могут совпадать, могут не иметь точек совпадения, а могут перекрещиваться и т.д.

Имя прилагательное

Имя существительное

Данный рисунок показывает, что у кругов есть общая область, которая показывает согласование существительного с прилагательным в роде, числе и падеже.

Попробуем составить схему предложения. Задание может быть такое: найти в предложении существительное, прилагательное и наречие и представить в виде кругов отношения между этими словами.

Медленно плывет белый пароход.

Правильной будет следующая комбинация:

Белый

(прилагательное)

Медленно

(наречие)

Пароход

(существительное)

С точки зрения морфологии наречие не имеет никаких общих признаков с существительными и прилагательными, поэтому круг со словом “медленно” не перекрещивается с другими словами, а морфологические признаки слов “белый” и “пароход” частично совпадают, так как общими для этих слов будет число, род и падеж. Данное совпадение изображается посредством двух перекрещивающихся кругов.

В любом случае учащимся приходится анализировать те или иные объекты сравнения по ряду признаков.

Задания с использованием приема “Круги Эйлера” отличаются наглядностью, способствуют моделированию усваиваемой информации, развивают абстрактное мышление и приобретают особую важность при работе с отстающими учащимися и с учениками , для которых русский язык не является родным.

Эйлер писал, что «круги очень подходят для того, чтобы облегчить наши размышления». Позднее аналогичный прием использовал ученый Венн и его назвали «диаграммы Венна».

Метод Эйлера получил заслуженное признание и популярность. И после него немало ученых использовали его в своей работе, а также видоизменяли на свой лад. Например, чешский математик Бернард Больцано использовал тот же метод, но с прямоугольными схемами.

Круги Эйлера – это не просто занимательная и интересная штука, но и весьма полезный, просто и наглядный метод решения задач. Причем не только абстрактных задач на школьных уроках, но и вполне себе житейских проблем.

Как решать задачи с кругами эйлера по информатике: Круги Эйлера в информатике — ЭкоДом: Дом своими руками

Содержание

Круги Эйлера в информатике

Сегодня разберём задачи на круги Эйлера в информатике.

Леонард Эйлер — швейцарский, немецкий и российский математик и механик, сыгравший огромную роль в развитии этих наук.

Задача (Простая)

В языке запросов поискового сервера для обозначения логической операции «ИЛИ» используется символ «|», а для логической операции «И» — символ «&». В таблице приведены запросы и количество найденных по ним страниц некоторого сегмента сети Интернет.

ЗапросНайдено страниц
(в тысячах)
Пушкин3500
Лермонтов2000
Пушкин | Лермонтов4500


Какое количество страниц (в тысячах) будет найдено по запросу Пушкин & Лермонтов? Считается, что все запросы выполнялись практически одновременно, так что набор страниц, содержащих все искомые слова, не изменялся за время выполнения запросов.

Решение:

Видим, что по запросу «Пушкин» в поисковике нашлось 3500 страниц. По запросу «Лермонтов» — 2000 страниц.

Запрос «Пушкин | Лермонтов» обозначает, что поисковик выдаст страницы, где есть слова про «Пушкина», и страницы, где есть слова про «Лермонтова», а так же могут быть страницы, где написано и про «Пушкина», и про «Лермонтова» одновременно.

Если сложить страницы, в которых написано про «Пушкина» и про «Лермонтова» получается 3500 + 2000 = 5500 страниц. Но почему же при запросе «Пушкин | Лермонтов» получается меньше страниц, всего 4500 ?

Этот факт обозначает то, что когда мы подсчитывали страницы про «Пушкина» (3500 страниц), мы подсчитали и те страницы, где было написано и про «Пушкина», и про «Лермонтова» одновременно.

Тоже самое и для количества страниц, где написано про «Лермонтова» (2000 страниц). В этом числе находятся и те, в которых одновременно упоминается и про «Пушкина», и про «Лермонтова».

В вопросе спрашивается, сколько страниц будет по запросу «Пушкин & Лермонтов«. Это обозначает, что как раз нужно найти количество страниц, где будет одновременно написано и про «Пушкина», и про «Лермонтова».

Отсюда получается:


Пушкин & Лермонтов = (3500 + 2000) — 4500 = 5500 — 4500 = 1000 страниц.

Это и будет ответ!

Теперь решим эту задачу с помощью Кругов Эйлера!

У нас всего есть две сущности: «Пушкин» и «Лермонтов». Поэтому рисуем два пересекающихся круга, желательно разными цветами.

Объединение двух кругов в общую фигуру (показано фиолетовым цветом), показывает операцию «Пушкин | Лермонтов». Эта операция всегда стремится увеличить площадь, объединить площади других фигур!

Обратите внимание, что круги пересекаются, из-за этого сумма площадей двух кругов по отдельности (3500 + 2000 = 5500) больше чем у фигуры, которая характеризует логическую операцию «ИЛИ» «Пушкин | Лермонтов» (4500).

Нужно найти площадь фигуры Пушкин & Лермонтов, которая закрашена золотистым цветом. Данная логическая операция «И» стремится уменьшить площадь. Она обозначает общую площадь других фигур.

Найдём сначала заштрихованную часть синего круга. Она равна: площадь фиолетовой фигуры (4500) минус площадь красного круга (3500).

Теперь легко найти площадь золотистой фигуры. Для этого нужно от площади синего круга вычесть площадь заштрихованной части. Получается:


Пушкин & Лермонтов (Количество страниц) = 2000 — 1000 = 1000

Получается, что по запросу Пушкин & Лермонтов будет найдено 1000 страниц.

Ответ: 1000

Рассмотрим ещё одну не сложную разминочную задачу.

Задача (Разминочная)

В языке запросов поискового сервера для обозначения логической операции «ИЛИ» используется символ «|», а для логической операции «И» – символ «&».

В таблице приведены запросы и количество найденных по ним страниц некоторого сегмента сети Интернет.

ЗапросНайдено страниц
(в тысячах)
Кокос | Ананас3400
Кокос & Ананас900
Кокос2100

Какое количество страниц (в тысячах) будет найдено по запросу Ананас?

Считается, что все запросы выполнялись практически одновременно, так что набор страниц, содержащих все искомые слова, не изменялся за время выполнения запросов.

Решение:

У нас две сущности: Кокос и Ананас. Нарисуем два круга Эйлера, которые пересекаются между собой. Так же отменим все имеющееся данные.

Найдём заштрихованную часть красного круга.

Весь красный круг 2100. Золотистая область равна 900. Заштрихованная часть равна 2100 — 900 = 1200.

После того, как нашли заштрихованную часть (такой полумесяц), можно найти уже площадь синего круга. Для этого нужно от площади фиолетовой фигуры отнять площадь заштрихованной части!


Ананас (Количество страниц) = 3400 — 1200 = 2200

Ответ: 2200

Разберём классическую задачу из информатики по кругам Эйлера.

Задача (Классическая)

В языке запросов поискового сервера для обозначения логической операции «ИЛИ» используется символ «|», а для логической операции «И» — символ «&».

В таблице приведены запросы и количество найденных по ним страниц некоторого сегмента сети Интернет.

ЗапросНайдено страниц
(в тысячах)
(Космос & Звезда) | (Космос & Планета)1100
Космос & Планета600
Космос & Планета & Звезда50

Какое количество страниц (в тыс. ) будет найдено по запросу Космос & Звезда?

Считается, что все запросы выполнялись практически одновременно, так что набор страниц, содержащих все искомые слова, не изменялся за время выполнения запросов.

Решение:

В этой задаче у нас три сущности: Космос, Планета, Звезда. Поэтому рисуем три круга Эйлера, которые пересекаются между собой.

Могут ли круги не пересекаться ? Могут! Если мы докажем, что площади по отдельности двух кругов в сумме дают площадь фигуры, которая получается при применении операции логического «ИЛИ».

Теперь отметим на нашем рисунке запрос (Космос & Звезда) | (Космос & Планета).

Сначала отменим для себя то, что находится в скобках. Первое Космос & Звезда

Теперь отметим вторую скобку Космос & Планета.

В выражении (Космос & Звезда) | (Космос & Планета) две скобки соединяет знак логического «ИЛИ». Значит, эти две области нужно объединить! Область (Космос & Звезда) | (Космос & Планета) отмечена фиолетовым цветом!

Отметим Космос & Планета ещё раз, т. к. для этого выражения известно количество страниц.

Площадь фигуры для выражения Космос & Планета & Звезда будет очень маленькая. Это общая часть для всех трёх кругов. Отметим её оранжевым цветом! Каждая точка этой фигуры должна одновременно быть в трёх кругах!

Найти нужно Космос & Звезда. Отменим на рисунке чёрным цветом ту область, которую нужно найти. Мы эту область уже отмечали салатовым цветом.

Теперь у нас есть все компоненты, чтобы решить эту задачу.

Найдём заштрихованную область.

Вся область Космос & Планета равна 600. А заштрихованная часть равна: область Космос & Планета (600) минус оранжевая область (50).

Количество страниц в заштрихованной части = 600 — 50 = 550

Тогда черная область легко находится: фиолетовая область (1100) минус заштрихованная область (550).

Количество страниц (при запросе Космос & Звезда) = 1100 — 550 = 550

Ответ: 550

Закрепляем материал по задачам на Круги Эйлера.

Задача (На закрепление)

В языке запросов поискового сервера для обозначения логической операции «ИЛИ» используется символ «|», а для обозначения логической операции «И» – символ «&». В таблице приведены запросы и количество найденных по ним страниц некоторого сегмента сети Интернет.

ЗапросНайдено страниц
(в тысячах)
Море & Солнце290
Море & Пляж355
Море & (Пляж | Солнце)465

Какое количество страниц (в тысячах) будет найдено по запросу Море & Пляж & Солнце? Считается, что все запросы выполнялись практически одновременно, так что набор страниц, содержащих все искомые слова, не изменялся за время выполнения запросов.

Решение:

В задаче используются три сущности: Море, Пляж, Солнце. Поэтому нарисуем три пересекающихся круга Эйлера.

Отметим все области для которых нам даны количество страниц.

В начале отметим Море & (Пляж | Солнце). Для начало нарисуем область, которая в скобках (Пляж | Солнце)

Теперь нужно очертить общую часть фиолетовой области и зелёного круга и получится Море & (Пляж | Солнце). Отметим оранжевым цветом.

Теперь отметим Море & Пляж.

Теперь отметим Море & Солнце.

Найти нужно ту область, которая получается в результате выделения общей части для всех трёх кругов! Обозначим её чёрным цветом!

Найдём заштрихованную область!


Количество страниц (в заштрихованной области) =
= Количество страниц (В оранжевой области) — Море & Солнце =

= 465 — 290 = 175

Чтобы найти искомую чёрную область, нужно из Море & Пляж (355) вычесть заштрихованную область (175).

Количество страниц (Море & Пляж & Солнце) =

= Море & Пляж (355) — Количество страниц (в заштрихованной области) 175 =

= 355 — 175 = 180

Ответ: 180

Решим ещё одну тренировочную задачу из информатики на Круги Эйлера.

Задача (с 4 сущностями)

В языке запросов поискового сервера для обозначения логической операции «ИЛИ» используется символ «|», а для обозначения логической операции «И» – символ «&».

В таблице приведены запросы и количество найденных по ним страниц некоторого сегмента сети Интернет.

ЗапросНайдено страниц
(в тысячах)
Англия & (Уэльс & Шотландия | Ирландия)450
Англия & Уэльс & Шотландия213
Англия & Уэльс & Шотландия & Ирландия87

Какое количество страниц (в тысячах) будет найдено по запросу

Англия & Ирландия?

Считается, что все запросы выполнялись практически одновременно, так что набор страниц, содержащих все искомые слова, не изменялся за время выполнения запросов.

Решение:

Нужно нарисовать 4 пересекающихся круга. Сначала нарисуем три круга, как обычно, оставив немного места для четвёртого круга.

Четвёртый круг для Ирландии нужно нарисовать так, чтобы он проходил через область (Англия & Уэльс & Шотландия). Это нам подсказывает сама таблица, где есть количество страниц для Англия & Уэльс & Шотландия, а так же для Англия & Уэльс & Шотландия & Ирландия.

Нужно отметить на рисунке Англия & (Уэльс & Шотландия | Ирландия). Это будем делать, как всегда поэтапно.

Область Уэльс & Шотландия выглядит так:

Добавим к этой области Ирландию через логическое «ИЛИ». Получается область (Уэльс & Шотландия | Ирландия). Произошло объединение серой области и жёлтого круга!

Теперь нужно сделать операцию логического «И» получившийся области с «Англией». Тогда область Англия & (Уэльс & Шотландия | Ирландия) примет вид:

Т.е. это общее между предыдущем серым контуром и красным кругом!

Отметим Англия & Уэльс & Шотландия — это общая территория трёх кругов: Красного, Синего и Зелёного. Отмечено оранжевым цветом.

Отметим Англия & Уэльс & Шотландия & Ирландия — это общая территория четырёх кругов. Область получается ещё меньше. Если взять точку в этой области, то мы будем находится сразу в четырёх кругах одновременно. Отмечено фиолетовым цветом.

Отметим то, что нужно найти Англия & Ирландия чёрным цветом.

Искомую чёрную область легко найти, если из серой области вычесть кусочек, окрашенный в бирюзовый цвет!

Найдём, сколько страниц приходится на бирюзовый кусочек:

Количество страниц (для бирюзового кусочка) =

= Англия & Уэльс & Шотландия (213) — Англия & Уэльс & Шотландия & Ирландия (87) =

= 213 — 87 = 126

Найдём искомую чёрную область.

Количество станиц (для чёрной области) =

= Англия & (Уэльс & Шотландия | Ирландия) (450) — Количество (для бирюзового кусочка) =

450 — 126 = 324

Это и будет ответ!

Ответ: 324.

Разберём задачу из реального экзамена по информатике, которая была в 2019 году в Москве! (Сейчас в 2021 задачи не встречаются на Круги Эйлера)

Задача (ЕГЭ по информатике, 2019, Москва)

В таблице приведены запросы и количество страниц, которые нашёл поисковый сервер по этим запросам в некоторым сегменте Интернета:

ЗапросНайдено страниц
(в тысячах)
Суфле450
Корзина200
Эклер490
Суфле & Корзина70
Суфле & Эклер160
Корзина & Эклер0

Сколько страниц (в тысячах) будет найдено по запросу

Суфле | Корзина | Эклер

Решение:

Видим, что у нас три поисковых разных слова, поэтому будет три разных круга Эйлера!

Так же видим, что логическое «И» между словами Корзина и Эклер даёт 0 страниц. Это значит, что эти круги не пересекаются! Так же круги бы не пересекались, если бы операция логического «ИЛИ» совпадала бы с суммой этих кругов.

Видим, что Суфле имеет с двумя кругами пересечения, а Корзина и Эклер не пересекаются.

Отметим всё, что нам дано в условии.

Жёлтым цветом отмечено Суфле | Корзина | Эклер . Объединение всех трёх кругов. Это то, что нужно найти.

Искомая жёлтая фигура складывается из заштрихованных областей и красного круга! Площадь красного круга мы знаем. Нужно найти площади заштрихованных частей.

Левая заштрихованная область находится просто:

Количество страниц (лев. заштрих. область) =

= Эклер (490) — Суфле & Эклер (160) = 330

Так же найдём площадь правой заштрихованной области:

Количество страниц (прав. заштрих. область) =

= Корзина (200) — Суфле & Корзина (70) = 130

Теперь можно найти искомую жёлтую область

Количество страниц (Суфле | Корзина | Эклер) =

= Красный круг (450) + лев. заштрих. область (310) + прав. заштрих. область (130) =

= 450 + 330 + 130 = 910

Задача решена, можно писать ответ.

Ответ: 910

Разберём ещё одну задачу из реального ЕГЭ уже 2020 года

Задача (ЕГЭ по информатике, 2020, Москва)

В таблице приведены запросы и количество страниц, которые нашёл поисковый сервер по этим запросам в некоторым сегменте Интернета:

ЗапросНайдено страниц
(в тысячах)
Аврора50
Крейсер45
Заря23
Аврора & Заря9
Заря & Крейсер0
Заря | Крейсер | Аврора93

Сколько страниц (в тысячах) будет найдено по запросу

Аврора & Крейсер

Решение:

Количество страниц при запросе Заря & Крейсер равно нулю. Значит, эти два круга не будут пересекаться.

Нарисуем все данные на рисунке.

Нужно найти для начала заштрихованную правую часть.


Количество страниц (для двух заштрих. частей) =
З | К | А (93) — Красный круг (50) = 43

Левую заштрихованную область легко найти.


Количество страниц (для левой заштрих. части) =

Синий круг (23) — А & З (9) = 14

Тогда для правой заштрихованной области получается:


Колич. страниц (для правой заштрих. части) =

Колич. страниц (для двух заштрих. частей) (43) — Колич. страниц (для лев. заштрих. части) (14) =

= 43 — 14 = 29

Тогда искомую область легко найти:


Колич. страниц (А & K) =

Зелёный круг (45) — Колич. страниц (для правой заштрих. части) (29) =

45 — 29 = 16

Ответ: 16

На этом всё! Надеюсь, вы теперь будете с удовольствием решать задачи по информатике с помощью Кругов Эйлера.

Решение задач с помощью кругов Эйлера

Круги Эйлера — геометрическая схема, с помощью которой можно
изобразить отношения между подмножествами, для наглядного представления.
Изобретены Леонардом Эйлером. Используется в математике, логике, менеджменте и
других прикладных направлениях.

 Задача №1

В языке запросов поискового сервера для обозначения логической операции «ИЛИ» используется символ «|», а для логической операции «И» — символ «&».

В таблице приведены запросы и количество найденных по ним страниц некоторого сегмента сети Интернет.

ЗапросНайдено страниц (в тысячах)
Торты | Пироги12000
Торты & Пироги6500
Пироги7700

Какое количество страниц (в тысячах) будет найдено по запросу Торты?Считается,
что все запросы выполнялись практически одновременно, так что набор
страниц, содержащих все искомые слова, не изменялся за время выполнения
запросов.

Решение задачи №1

Для решения задачи отобразим множества Тортов и Пирогов в
виде кругов Эйлера.

Обозначим каждый сектор отдельной буквой (А, Б, В).

Из условия задачи следует:

Торты │Пироги =  А+Б+В = 12000

Торты & Пироги = Б = 6500

Пироги = Б+В = 7700

Чтобы найти количество Тортов (Торты = А+Б), надо найти сектор А, для этого из общего
множества (Торты│Пироги) отнимем множество Пироги.

Торты│Пироги –
Пироги = А+Б+В-(Б+В) = А = 1200 – 7700 = 4300

Сектор А равен 4300, следовательно

Торты = А+Б = 4300+6500 = 10800


Задача №2

В языке запросов поискового сервера для обозначения логической операции «ИЛИ» используется символ «|», а для логической операции «И» — символ «&».

В таблице приведены запросы и количество найденных по ним страниц некоторого сегмента сети Интернет.

ЗапросНайдено страниц (в тысячах)
Пироженое & Выпечка5100
Пироженое9700
Пироженое | Выпечка14200

Какое количество страниц (в тысячах) будет найдено по запросу Выпечка?

Считается,
что все запросы выполнялись практически одновременно, так что набор
страниц, содержащих все искомые слова, не изменялся за время выполнения
запросов. Решение задачи №2

Для решения задачи отобразим множества Пироженых и Выпечек в виде кругов
Эйлера.

Обозначим каждый сектор отдельной буквой (А, Б, В).

Из условия задачи следует:

Пироженое & Выпечка = Б = 5100

Пироженое = А+Б = 9700

Пироженое │ Выпечка =  А+Б+В = 14200

Чтобы найти количество Выпечки (Выпечка = Б+В), надо найти сектор В, для этого из общего
множества (Пироженое │ Выпечка ) отнимем множество Пироженое.

Пироженое │ Выпечка – Пироженное = А+Б+В-(А+Б) = В = 14200–9700 = 4500

Сектор В равен 4500, следовательно 
Выпечка = Б + В = 4300+5100 = 9400


Задача №3
В таблице приведены запросы к поисковому серверу. Расположите номера запросов в порядке убывания количества страниц, которые найдет поисковый сервер по каждому запросу.
Для обозначения логической операции «ИЛИ» используется символ «|», а для логической операции «И» — символ «&».

1спаниели | (терьеры & овчарки)
2спаниели | овчарки
3спаниели | терьеры | овчарки
4терьеры | овчарки

Решение задачи №3

Представим множества овчарок, терьеров и спаниелей в виде
кругов Эйлера, обозначим сектора буквами (А, Б, В, Г).

Преобразим условие задачи в виде суммы секторов:

спаниели │(терьеры &
овчарки) = Г + Б

спаниели│овчарки = Г + Б + В

спаниели│терьеры│овчарки = А + Б + В + Г

терьеры & овчарки = Б

Из сумм секторов мы видим какой запрос выдал больше
количества страниц.

Расположим номера запросов в порядке убывания количества
страниц: 3 2 1 4


Задача №4

В таблице приведены запросы к поисковому серверу. Расположите номера запросов в порядке возврастания количества страниц, которые найдет поисковый сервер по каждому запросу.
Для обозначения логической операции «ИЛИ» используется символ «|», а для логической операции «И» — символ «&».

1барокко | классицизм | ампир
2барокко | классицизм & ампир
3классицизм & ампир
4барокко | классицизм

Решение задачи №4

Представим множества классицизм, ампир и классицизм в виде
кругов Эйлера, обозначим сектора буквами (А, Б, В, Г).

Преобразим условие задачи в виде суммы секторов:

барокко│ классицизм │ампир = А + Б + В + Г
барокко │(классицизм & ампир) = Г + Б
классицизм & ампир = Б
барокко│ классицизм = Г + Б + А

Из сумм секторов мы видим какой запрос выдал больше
количества страниц.

Расположим номера запросов в порядке возрастания количества
страниц: 3 2 4 1


Задача №5В таблице приведены запросы к поисковому серверу. Расположите номера запросов в порядке возврастания количества страниц, которые найдет поисковый сервер по каждому запросу.
Для обозначения логической операции «ИЛИ» используется символ «|», а для логической операции «И» — символ «&».

1канарейки | терьеры | содержание
2канарейки & содержание
3канарейки & щеглы & содержание
4разведение & содержание & канарейки & щеглы

Решение задачи №5

Для решения задачи представим запросы в виде кругов Эйлера.

K —  канарейки,

Щ – щеглы,

С – содержание,

Р – разведение.

Далее будем закрашивать красным цветом сектора согласно
запросам, наибольший по величине сектор даст большее количество страниц на
запрос.

канарейки | терьеры | содержаниеканарейки & содержаниеканарейки & щеглы & содержаниеразведение & содержание & канарейки & щеглы

Самая большая область закрашенных секторов у первого
запроса, затем у второго, затем у третьего, а у четвертого запроса самый
маленький.

В порядке возрастания по количеству страниц запросы будут
представлены в следующем порядке: 4 3 2 1

Обратите внимание что в первом запросе закрашенные сектора
кругов Эйлера содержат в себе закрашенные сектора второго запроса, а
закрашенные сектора второго запроса содержат закрашенные сектора третьего
запроса, закрашенные сектора третьего запроса содержат закрашенный сектор
четвертого запроса.

Только при таких условиях мы можем быть уверены, что
правильно решили задачу. 


Задачи для самостоятельного решения

Задача №6

В таблице приведены запросы к поисковому серверу. Расположите номера запросов в порядке возрастания количества страниц, которые найдет поисковый сервер по каждому запросу.
Для обозначения логической операции «ИЛИ» используется символ «|», а для логической операции «И» — символ «&».

1принтеры & сканеры & продажа
2принтеры  & продажа
3принтеры | продажа
4принтеры | сканеры | продажа

Задача №7

В таблице приведены запросы к поисковому серверу. Расположите номера запросов в порядке возрастания количества страниц, которые найдет поисковый сервер по каждому запросу.
Для обозначения логической операции «ИЛИ» используется символ «|», а для логической операции «И» — символ «&».

1физкультура
2физкультура & подтягивания & отжимания
3физкультура & подтягивания
4физкультура | фитнесс

Использованные материалы >>> 

Решение подобных задач  по информатике >>>

Ответы к задачам для самостоятельного решения

Номер задачиОтвет
6ГБВА
7БВАГ

Использование метода кругов Эйлера (диаграмм Эйлера–Венна) при решении задач в курсе информатики и ИКТ

1. Введение

В курсе Информатики и ИКТ основной и старшей
школы рассматриваются такие важные темы как
“Основы логики” и “Поиск информации в
Интернет”. При решении определенного типа задач
удобно использовать круги Эйлера (диаграммы
Эйлера-Венна).

Математическая справка. Диаграммы Эйлера-Венна
используются прежде всего в теории множеств как
схематичное изображение всех возможных
пересечений нескольких множеств. В общем случае
они изображают все 2n комбинаций n свойств.
Например, при n=3 диаграмма Эйлера-Венна обычно
изображается в виде трех кругов с центрами в
вершинах равностороннего треугольника и
одинаковым радиусом, приблизительно равным
длине стороны треугольника.

2. Представление логических связок в поисковых
запросах

При изучении темы “Поиск информации в
Интернет” рассматриваются примеры поисковых
запросов с использованием логических связок,
аналогичным по смыслу союзам “и”, “или”
русского языка. Смысл логических связок
становится более понятным, если
проиллюстрировать их с помощью графической
схемы – кругов Эйлера (диаграмм Эйлера-Венна).

Логическая связкаПример запросаПояснениеКруги Эйлера
& — “И”Париж & университетБудут отобраны все страницы, где
упоминаются оба слова: Париж и университет
Рис.1
| — “ИЛИ”Париж | университетБудут отобраны все страницы, где
упоминаются слова Париж и/или университет
Рис. 2

3. Связь логических операций с теорией множеств

С помощью диаграмм Эйлера-Венна можно наглядно
представить связь логических операций с теорией
множеств. Для демонстрации можно
воспользоваться слайдами в Приложение
1.

Логические операции задаются своими таблицами
истинности. В Приложении 2
подробно рассматриваются графические
иллюстрации логических операций вместе с их
таблицами истинности. Поясним принцип
построения диаграммы в общем случае. На
диаграмме – область круга с именем А отображает
истинность высказывания А (в теории множеств
круг А – обозначение всех элементов, входящих в
данное множество). Соответственно, область вне
круга отображает значение “ложь”
соответствующего высказывания. Что бы понять
какая область диаграммы будет отображением
логической операции нужно заштриховать только
те области, в которых значения логической
операции на наборах A и B равны “истина”.

Например, значение импликации равно “истина”
в трех случаях (00, 01 и 11). Заштрихуем
последовательно: 1) область вне двух
пересекающихся кругов, которая соответствует
значениям А=0, В=0; 2) область, относящуюся только к
кругу В (полумесяц), которая соответствует
значениям А=0, В=1; 3) область, относящуюся и к кругу
А и к кругу В (пересечение) – соответствует
значениям А=1, В=1. Объединение этих трех областей
и будет графическим представлением логической
операции импликации.

4. Использование кругов Эйлера при
доказательстве логических равенств (законов)

Для того, чтобы доказать логические равенства
можно применить метод диаграмм Эйлера-Венна.
Докажем следующее равенство ¬(АvВ) = ¬А&¬В (закон
де Моргана).

Для наглядного представления левой части
равенства выполним последовательно:
заштрихуем оба круга (применим дизъюнкцию) серым
цветом, затем для отображения инверсии
заштрихуем область за пределами кругов черным
цветом:

Рис. 3 Рис.4

Для визуального представления правой части
равенства выполним последовательно:
заштрихуем область для отображения инверсии (¬А)
серым цветом и аналогично область ¬В также серым
цветом; затем для отображения конъюнкции нужно
взять пересечение этих серых областей (результат
наложения представлен черным цветом):

Рис.5 Рис.6 Рис.7

Видим, что области для отображения левой и
правой части равны. Что и требовалось доказать.

5. Задачи в формате ГИА и ЕГЭ по теме: “Поиск
информации в Интернет”

Задача №18 из демо-версии ГИА 2013.

В таблице приведены запросы к поисковому
серверу. Для каждого запроса указан его код –
соответствующая буква от А до Г. Расположите коды
запросов слева направо в порядке убывания количества
страниц, которые найдет поисковый сервер по
каждому запросу.

КодЗапрос
А(Муха & Денежка) | Самовар
БМуха & Денежка & Базар & Самовар
ВМуха | Денежка | Самовар
ГМуха & Денежка & Самовар

Решение:

Для каждого запроса построим диаграмму
Эйлера-Венна:

Запрос А

Рис. 8

Запрос Б

Рис. 9

Запрос В

Рис. 10

Запрос Г

Рис. 11

Ответ: ВАГБ.

Задача В12 из демо-версии ЕГЭ-2013.

В таблице приведены запросы и количество
найденных по ним страниц некоторого сегмента
сети Интернет.

ЗапросНайдено страниц (в тысяч)
Фрегат | Эсминец3400
Фрегат & Эсминец900
Фрегат2100

Какое количество страниц (в тысячах) будет
найдено по запросу Эсминец?

Считается, что все запросы выполнялись
практически одновременно, так что набор страниц,
содержащих все искомые слова, не изменялся за
время выполнения запросов.

Решение:

Пусть

Ф – количество страниц (в тысячах) по запросу Фрегат;

Э – количество страниц (в тысячах) по запросу Эсминец;

Х – количество страниц (в тысячах) по запросу, в
котором упоминается Фрегат и не упоминается
Эсминец;

У – количество страниц (в тысячах) по запросу, в
котором упоминается Эсминец и не
упоминается Фрегат.

Построим диаграммы Эйлера-Венна для каждого
запроса:

ЗапросДиаграмма Эйлера-ВеннаКоличество страниц
Фрегат | ЭсминецРис.123400
Фрегат & ЭсминецРис. 13900
ФрегатРис.142100
ЭсминецРис.15?

Согласно диаграммам имеем:

  1. Х+900+У = Ф+У = 2100+У = 3400. Отсюда находим У = 3400-2100 = 1300.
  2. Э = 900+У = 900+1300= 2200.

Ответ: 2200.

6. Решение логических содержательных
задач методом диаграмм Эйлера-Венна

Задача 1.

В классе 36 человек. Ученики этого класса
посещают математический, физический и
химический кружки, причем математический кружок
посещают 18 человек, физический — 14 человек,
химический — 10. Кроме того, известно, что 2
человека посещают все три кружка, 8 человек — и
математический и физический, 5 и математический и
химический, 3 — и физический и химический.

Сколько учеников класса не посещают никаких
кружков?

Решение:

Для решения данной задачи очень удобным и
наглядным является использование кругов Эйлера.

Самый большой круг – множество всех учеников
класса. Внутри круга три пересекающихся
множества: членов математического (М),
физического (Ф), химического (Х) кружков.

Пусть МФХ – множество ребят, каждый из
которых посещает все три кружка. МФ¬Х
множество ребят, каждый из которых посещает
математический и физический кружки и не
посещает химический. ¬М¬ФХ — множество ребят,
каждый из которых посещает химический кружок и
не посещает физический и математический кружки.

Аналогично введем множества: ¬МФХ, М¬ФХ,
М¬Ф¬Х, ¬МФ¬Х, ¬М¬Ф¬Х.

Известно, что все три кружка посещают 2
человека, следовательно, в область МФХ впишем
число 2. Т.к. 8 человек посещают и математический и
физический кружки и среди них уже есть 2 человека,
посещающих все три кружка, то в область МФ¬Х впишем
6 человек (8-2). Аналогично определим количество
учащихся в остальных множествах:

Круги Эйлера с названиями
непересекающихся множеств:

Рис. 16

Круги Эйлера с количественной
информацией:

Рис. 17

Например, количество человек, которые посещают
физический кружок 2+6+1+5=14

Просуммируем количество человек по всем
областям: 7+6+3+2+4+1+5=28. Следовательно, 28 человек из
класса посещают кружки.

Значит, 36-28 = 8 учеников не посещают кружки.

Ответ: 8.

Задача 2.

После зимних каникул классный руководитель
спросил, кто из ребят ходил в театр, кино или цирк.
Оказалось, что из 36 учеников класса двое не были
ни в кино. ни в театре, ни в цирке. В кино побывало
25 человек, в театре — 11, в цирке 17 человек; и в кино,
и в театре — 6; и в кино и в цирке — 10; и в театре и в
цирке — 4.

Сколько человек побывало и в кино, и в театре, и
в цирке?

Решение:

Пусть х – количество ребят, которые побывали и
в кино, и в театре, и в цирке.

Тогда можно построить следующую диаграмму и
посчитать количество ребят в каждой области:

Рис.18.

В кино и театре побывало 6 чел., значит,
только в кино и театре (6-х) чел.

Аналогично,
только в кино и цирке (10-х) чел.

Только в театре и цирке (4-х) чел.

В кино побывало 25 чел., значит, из них только в
кино были 25 — (10-х) – (6-х) – х = (9+х).

Аналогично, только в театре были (1+х) чел.

Только в цирке были (3+х) чел.

Не были в театре, кино и цирке – 2 чел.

Значит, 36-2=34 чел. побывали на мероприятиях.

С другой стороны можем просуммировать
количество человек, которые были в театре, кино и
цирке:

(9+х)+(1+х)+(3+х)+(10-х)+(6-х)+(4-х)+х = 34

33+х = 34.

Отсюда следует, что только один человек побывал
на всех трех мероприятиях.

Ответ: 1.

Таким образом, круги Эйлера (диаграммы
Эйлера-Венна) находят практическое применение
при решении задач в формате ЕГЭ и ГИА и при
решении содержательных логических задач.

Литература

  1. В. Ю. Лыскова, Е.А. Ракитина. Логика в информатике.
    М.: Информатика и Образование, 2006. 155 с.
  2. Л.Л. Босова. Арифметические и логические основы
    ЭВМ. М.: Информатика и образование, 2000. 207 с.
  3. Л.Л. Босова, А.Ю. Босова. Учебник. Информатика и
    ИКТ для 8 класса: БИНОМ. Лаборатория знаний, 2012. 220
    с.
  4. Л.Л. Босова, А.Ю. Босова. Учебник. Информатика и
    ИКТ для 9 класса: БИНОМ. Лаборатория знаний, 2012. 244
    с.
  5. Сайт ФИПИ: http://www.fipi.ru/

круги Эйлера — Основы логики и логические основы компьютера

В языке запросов поискового сервера для обозначения логической операции «ИЛИ» используется символ «|», а для логической операции «И» — символ «&».

В таблице приведены запросы и количество найденных по ним страниц некоторого сегмента сети интернет.Какое количество страниц (в тысячах) будет найдено по запросу Крейсер & Линкор?Считается, что все вопросы выполняются практически одновременно, так что набор страниц, содержащих все искомые слова, не изменялся за время выполнения запросов.

При помощи кругов Эйлера изобразим условия задачи. При этом цифры 1, 2 и 3 используем, чтобы обозначить полученные в итоге области.

Опираясь на условия задачи, составим уравнения:

Чтобы найти Крейсер & Линкор (обозначенный на чертеже как область 2), подставим уравнение (2) в уравнение (1) и выясним, что:

4800 + 3 = 7000, откуда получаем 3 = 2200.

Теперь этот результат мы можем подставить в уравнение (3) и выяснить, что:

2 + 2200 = 4500, откуда 2 = 2300.

Ответ: 2300 — количество страниц, найденных по запросу Крейсер & Линкор.

В языке запросов поискового сервера для обозначения логической операции «ИЛИ» используется символ «|», а для логической операции «И» — символ «&».

Считается, что все запросы выполнялись практически одновременно, так что набор страниц, содержащих все искомые слова, не изменялся за время выполнения запросов.

Решение

Для решения задачи отобразим множества Тортов и Пирогов в виде кругов Эйлера.

Обозначим каждый сектор отдельной буквой (А, Б, В).

Из условия задачи следует:

Торты │Пироги =  А+Б+В = 12000

Торты & Пироги = Б = 6500

Пироги = Б+В = 7700

Чтобы найти количество Тортов (Торты = А+Б), надо найти сектор А, для этого из общего множества (Торты│Пироги) отнимем множество Пироги.

Торты│Пироги – Пироги = А+Б+В-(Б+В) = А = 1200 – 7700 = 4300

Сектор А равен 4300, следовательно

Торты = А+Б = 4300+6500 = 10800


Задача 3

В языке запросов поискового сервера для обозначения логической операции «ИЛИ» используется символ «|», а для логической операции «И» — символ «&».

В таблице приведены запросы и количество найденных по ним страниц некоторого сегмента сети Интернет.

Запрос
Найдено страниц (в тысячах)
Пироженое & Выпечка
5100
Пироженое
9700
Пироженое | Выпечка
14200

Какое количество страниц (в тысячах) будет найдено по запросуВыпечка?

Считается, что все запросы выполнялись практически одновременно, так что набор страниц, содержащих все искомые слова, не изменялся за время выполнения запросов. Решение 

Для решения задачи отобразим множестваПироженых и Выпечек в виде кругов Эйлера.

Обозначим каждый сектор отдельной буквой (А, Б, В).

Из условия задачи следует:

Пироженое & Выпечка = Б = 5100

Пироженое = А+Б = 9700

Пироженое │ Выпечка =  А+Б+В = 14200

Чтобы найти количество Выпечки (Выпечка = Б+В), надо найти секторВ, для этого из общего множества (Пироженое │ Выпечка ) отнимем множество Пироженое.

Пироженое │ Выпечка – Пироженное = А+Б+В-(А+Б) = В = 14200–9700 = 4500

Сектор В равен 4500, следовательно  Выпечка = Б + В = 4500+5100 =9600

Задача 4
В таблице приведены запросы к поисковому серверу. Расположите номера запросов в порядке убывания количества страниц, которые найдет поисковый сервер по каждому запросу.
Для обозначения логической операции «ИЛИ» используется символ «|», а для логической операции «И» — символ «&».

1
спаниели | (терьеры & овчарки)
2
спаниели | овчарки
3
спаниели | терьеры | овчарки
4
терьеры & овчарки

Решение 

Представим множества овчарок, терьеров и спаниелей в виде кругов Эйлера, обозначим сектора буквами (А, Б, В, Г).

Преобразим условие задачи в виде суммы секторов:

спаниели │(терьеры & овчарки) = Г + Б

спаниели│овчарки = Г + Б + В

спаниели│терьеры│овчарки = А + Б + В + Г

терьеры & овчарки = Б

Из сумм секторов мы видим какой запрос выдал больше количества страниц.

Расположим номера запросов в порядке убывания количества страниц: 3 2 1 4


Задача 5

В таблице приведены запросы к поисковому серверу. Расположите номера запросов в порядке возрастания количества страниц, которые найдет поисковый сервер по каждому запросу.
Для обозначения логической операции «ИЛИ» используется символ «|», а для логической операции «И» — символ «&».

1
барокко | классицизм | ампир
2
барокко | (классицизм & ампир)
3
классицизм & ампир
4
барокко | классицизм

Решение 

Представим множества классицизм, ампир и классицизм в виде кругов Эйлера, обозначим сектора буквами (А, Б, В, Г).

Преобразим условие задачи в виде суммы секторов:

барокко│ классицизм │ампир = А + Б + В + Г
барокко │(классицизм & ампир) = Г + Б
классицизм & ампир = Б
барокко│ классицизм = Г + Б + А

Из сумм секторов мы видим какой запрос выдал больше количества страниц.

Расположим номера запросов в порядке возрастания количества страниц: 3 2 4 1


Задача 6
В таблице приведены запросы к поисковому серверу. Расположите номера запросов в порядке возрастания количества страниц, которые найдет поисковый сервер по каждому запросу.
Для обозначения логической операции «ИЛИ» используется символ «|», а для логической операции «И» — символ «&».

1
канарейки | щеглы | содержание
2
канарейки & содержание
3
канарейки & щеглы & содержание
4
разведение & содержание & канарейки & щеглы

Решение 

Для решения задачи представим запросы в виде кругов Эйлера.

K —  канарейки,

Щ – щеглы,

С – содержание,

Р – разведение.

Далее будем закрашивать красным цветом сектора согласно запросам, наибольший по величине сектор даст большее количество страниц на запрос.

канарейки | терьеры | содержаниеканарейки & содержаниеканарейки & щеглы & содержаниеразведение & содержание & канарейки & щеглы

Самая большая область закрашенных секторов у первого запроса, затем у второго, затем у третьего, а у четвертого запроса самый маленький.

В порядке возрастания по количеству страниц запросы будут представлены в следующем порядке: 4 3 2 1

Обратите внимание что в первом запросе закрашенные сектора кругов Эйлера содержат в себе закрашенные сектора второго запроса, а закрашенные сектора второго запроса содержат закрашенные сектора третьего запроса, закрашенные сектора третьего запроса содержат закрашенный сектор четвертого запроса.

Только при таких условиях мы можем быть уверены, что правильно решили задачу.

 

Задача 7 (ЕГЭ 2013)

 В языке запросов поискового сервера для обозначения логической операции «ИЛИ» используется символ «|», а для логической операции «И» – символ «&». 

В таблице приведены запросы и количество найденных по ним страниц некоторого сегмента сети Интернет. 

ЗапросНайдено страниц
(в тысячах)
Фрегат | Эсминец3400
Фрегат & Эсминец900
Фрегат2100

Какое количество страниц (в тысячах) будет найдено по запросу Эсминец
Считается, что все запросы выполнялись практически одновременно, так что набор страниц, содержащих все искомые слова, не изменялся за время выполнения запросов.

Ответ: 2200

Решение: Запрос «Фрегат» обозначим символом «Ф», «Эсминец» — символом «Э».

Э=(Ф|Э)-Ф+(Ф&Э)=3400-2100+900=2200.

Разбор задачи B12 (демо ЕГЭ 2012)

Время выполнения-2 мин, уровень сложности-повышенный

В языке запросов поискового сервера для обозначения логической операции «ИЛИ» используется символ «|», а для логической операции «И» – символ «&».
В таблице приведены запросы и количество найденных по ним страниц некоторого сегмента сети Интернет.

ЗапросНайдено страниц
(в тысячах)
Шахматы | Теннис7770
Теннис5500
Шахматы & Теннис1000

Какое количество страниц (в тысячах) будет найдено по запросу Шахматы?
Считается, что все запросы выполнялись практически одновременно, так что набор страниц, содержащих все искомые слова, не изменялся за время выполнения запросов.

Ответ: 3270

Решение: Изобразим запросы в виде диаграмм Эйлера-Венна.

Запрос «Шахматы» обозначим символом «Ш», «Теннис» — символом «Т».

Ш=(Ш|Т)-Т+(Ш&Т)=7770-5500+1000=3270.


Задачи для самостоятельного решения

Задача 1

В таблице приведены запросы к поисковому серверу. Расположите номера запросов в порядке возрастания количества страниц, которые найдет поисковый сервер по каждому запросу.
Для обозначения логической операции «ИЛИ» используется символ «|», а для логической операции «И» — символ «&».

1
принтеры & сканеры & продажа
2
принтеры  & продажа
3
принтеры | продажа
4
принтеры | сканеры | продажа

Задача 2

В таблице приведены запросы к поисковому серверу. Расположите номера запросов в порядке возрастания количества страниц, которые найдет поисковый сервер по каждому запросу.
Для обозначения логической операции «ИЛИ» используется символ «|», а для логической операции «И» — символ «&».

1
физкультура
2
физкультура & подтягивания & отжимания
3
физкультура & подтягивания
4
физкультура | фитнесс

Задача №17. Построение запросов для поисковых систем. Расположение запросов по возрастанию (убыванию). Подсчет количества страниц.

Автор — Лада Борисовна Есакова.

Для быстрого поиска информации в Интернете используют поисковые запросы. Поисковый запрос – это набор ключевых слов, соединенных знаками логических операций И, ИЛИ, НЕ.

Приоритет выполнения операций, если нет специально поставленных скобок, следующий: сначала НЕ, затем И, затем ИЛИ.

Нужно понимать, что операция И (одновременное выполнение условий) сокращает объем получаемого результата, а операция ИЛИ (выполнение хотя бы одного из условий) наоборот увеличивает объем.

Если в запросе стоит фраза в кавычках, система будет искать точно такую фразу целиком.

1. Расположение запросов по возрастанию (убыванию)

Операция «И» (&) обозначает одновременное присутствие ключевых слов в искомых документах, а потому уменьшает количество найденной информации. Чем больше ключевых слов соединены операцией «И», тем меньше количество найденной информации. И наоборот, операция «ИЛИ» (|) обозначает присутствие хотя бы одного ключевого слова в искомых документах, а потому увеличивает количество найденной информации.

Пример 1.

В таблице приведены запросы к поисковому серверу. Расположите обозначения запросов в порядке возрастания количества страниц, которые найдет поисковый сервер по каждому запросу.

А) реферат | математика | Гаусс
Б) реферат | математика | Гаусс | метод
В) реферат | математика
Г) реферат & математика & Гаусс

Решение:

Самое маленькое количество страниц будет отобрано по запросу с наибольшим количеством операций «И» (запрос Г), Самое большое количество страниц будет отобрано по запросу с наибольшим количеством операций «ИЛИ» (запрос Б). По запросу А будет отобрано больше страниц, чем по запросу В, т.к. запрос А содержит больше ключевых слов, связанных операцией «ИЛИ».

Ответ: ГВАБ

2. Подсчет найденных по запросу страниц

Такой тип задач обычно решают системой уравнений. Предложу более наглядный и простой способ.

Принцип отбора информации по поисковым запросам хорошо иллюстрирует диаграмма Эйлера-Венна (круги Эйлера). На диаграмме множества изображаются пересекающимися кругами. Операция «И» (&) — это пересечение кругов, а операция «ИЛИ» (|) – это объединение кругов.

Например, обозначим кругами множества Яблоки, Груши, Бананы. По запросу Яблоки & Груши & Бананы будет отобрано пересечение (общая часть) всех трех кругов:

По запросу Яблоки | Груши будет отобрано объединение двух кругов:

Пример 2.

В таблице приведены запросы и количество страниц, которые нашел поисковый сервер по этим запросам в некотором сегменте Интернета:

Сколько страниц (в тысячах) будет найдено по запросу шахматы?

Решение:

Нарисуем диаграмму Эйлера-Венна. Прием решения задачи состоит в подсчете количества страниц, соответствующего каждой области, ограниченной линиями:

Запросу шахматы & теннис соответствует средняя область (1000 тыс. страниц), а запросу теннис – весь правый круг (5500 тыс. страниц).

Тогда правый «обрезанный круг» — это 5500-1000=4500:

Запросу шахматы | теннис соответствуют оба круга (7770), тогда левый «обрезанный круг» — это 7770-5500=2270

Итак, мы посчитали количества страниц для каждой ограниченной линиями области:

Несложно увидеть, что по запросу шахматы будет найдено 2270+1000=3270 тыс. страниц.

Ответ: 3270

Пример 3.

В таблице приведены запросы и количество страниц, которые нашел поисковый сервер по этим запросам в некотором сегменте Интернета:

Сколько страниц (в тысячах) будет найдено по запросу

Москва & (Париж | Лондон)

Решение:

Как и в предыдущей задаче, нарисуем диаграмму Эйлера-Венна и посчитаем количество страниц, соответствующее каждой известной области, ограниченной линиями:

Несложно увидеть, что запросу Москва & (Париж | Лондон) соответствует область:

Ответ: 427

Круги Эйлера.

Задачи | Тренажёр по информатике и икт (6 класс) на тему:

Круги Эйлера. Задачи для самостоятельного решения

Задача №1:

    Из 100 туристов, отправляющихся в заграничное путешествие, немецким языком владеют 30 человек, английским – 28, французским – 42. Английским и немецким одновременно владеют 8 человек, английским и французским -10 , немецким и французским – 5, всеми тремя языками – 3.    

Сколько туристов не владеют ни одним языком?

Ответ:    20 человек.

Задача №2

В ясельной группе 11 деток любят манную кашу, 13 – гречневую и 7 малышей – перловую. Четверо любят и манную, и гречневую, 3 – манную и перловую, 6- гречневую и перловую, а двое с удовольствием «уплетают» все три вида каши. Сколько детей в этой группе, если в ней нет ни одного ребёнка, вовсе не любящего кашу?

Ответ:    20 человек

Задача №3

В одной семье было много детей. 7 из них любили капусту, 6 – морковь, 5 – горох, 4 – капусту и морковь, 3 – капусту и горох, 2 – морковь и горох, 1 – и капусту, и морковь, и горох. Сколько детей было в семье?

Ответ: 10

Задача №4

В группе 29 студентов. Среди них 14 любителей  классической музыки, 15-джаза, 14 – народной музыки. Классическую музыку и джаз слушают 6 студентов, народную музыку и джаз – 7, классику и народную – 9. Пятеро студентов слушают всякую музыку, а остальные не любят никакой музыки. Сколько их?

Ответ:3

Задача №5

Учащиеся 5 и 6 классов отправились на экскурсию. Мальчиков было 16, учащихся 6 класса – 24, пятиклассниц столько, сколько мальчиков из 6 класса. Сколько всего детей побывали на экскурсии?

Ответ: 40

Задача №6

На полу комнаты площадью 24 м² лежат три ковра. Площадь одного из них  -10 м², другого – 8 м², третьего – 6 м². Каждые два ковра перекрываются по площади 3 м², а площадь участка пола, покрытого всеми тремя коврами, составляет 1 м². Найдите площадь участка пола:

а)покрытого первым и вторым коврами, но не покрытого третьим ковром;

б)покрытого только первым ковром;

в)не покрытого коврами.

Ответ:10, 5,8

Задача №7

Из 100 приехавших туристов 75 знали немецкий язык и 83 знали французский. 10 человек не знали ни немецкого, ни французского. Сколько туристов знали оба эти языка?

Ответ: 68

Задача №8

Часть жителей нашего дома выписывают только газету «Комсомольская правда», часть – только газету «Известия», а часть – и ту, и другую газету. Сколько процентов жителей дома выписывают обе газеты, если на газету «Комсомольская правда» из них подписаны 85%, а на «Известия» – 75%?

Ответ: 60%

Раздаточный материал по информатике на тему «Решение логических задач с помощью кругов Эйлера» (10 класс)

Составление запросов для поисковых систем с использованием логических выражений.

Задача 1

Расположите обозначения запросов в порядке возрастания количества страниц, которые найдет поисковый сервер по каждому запросу.

А

Музыка | классика | Моцарт | серенада

Б

Музыка | классика

В

Музыка | классика | Моцарт

Г

Музыка & классика & Моцарт

Задача 2

Расположите обозначения запросов в порядке возрастания количества страниц, которые найдет поисковый сервер по каждому запросу.

А

реферат | математика | Гаусс

Б

реферат | математика | Гаусс | метод

В

реферат | математика

Г

реферат & математика & Гаусс

Задача 3

Расположите обозначения запросов в порядке возрастания количества страниц, которые найдет поисковый сервер по каждому запросу.

А

Америка | Колумб

Б

Америка | путешественники | Колумб | открытие

В

Америка | путешественники | Колумб

Г

Америка & путешественники & Колумб

Задача 4

В таблице приведены запросы к поисковому серверу. Расположите номера запросов в порядке убывания количества страниц, который найдет поисковый сервер по каждому запросу.

Для обозначения логической операции «ИЛИ» в запросе используется символ |, а для логической операции «И» — &

1

усы & хвост & (лапы | документы)

2

усы & лапы & хвост & документы

3

лапы & хвост

4

лапы | хвост

Задача 5

В таблице приведены запросы к поисковому серверу. Расположите номера запросов в порядке убывания количества страниц, который найдет поисковый сервер по каждому запросу.

Для обозначения логтческой операции «ИЛИ» в запросе используется символ |, а для логической операции «И» — &

1

рога & копыта | хвосты

2

рога | копыта | хвосты

3

рога & копыта

4

рога & (копыта | хвосты)

Задача 6

В таблице приведены запросы к поисковому серверу. Расположите номера запросов в порядке возрастания количества страниц, который найдет поисковый сервер по каждому запросу.

Для обозначения логической операции «ИЛИ» в запросе используется символ |, а для логической операции «И» — &

1

Сыр

2

сыр | плесень

3

сыр & плесень

4

сыр & плесень & (Голландия | Франция)

Задача 7

В таблице приведены запросы и количество страниц, которые нашел поисковый сервер по этим запросам в некотором сегменте Интернета:

Запрос

Кол-во страниц (тыс. )

1

мезозой

50

2

кроманьонец

60

3

неандерталец

70

4

мезозой | кроманьонец

80

5

мезозой | неандерталец

100

6

неандерталец & (мезозой | кроманьонец)

20

Сколько страниц (в тысячах) будет найдено по запросу

кроманьонец & (мезозой | неандерталец)

Задача 8

В таблице приведены запросы и количество страниц, которые нашел поисковый сервер по этим запросам в некотором сегменте Интернета:

Запрос

Количество страниц (тыс.)

пирожное & выпечка

3200

пирожное

8700

выпечка

7500

Сколько страниц (в тысячах) будет найдено по запросу

пирожное | выпечка

Задача 9

В таблице приведены запросы и количество страниц, которые нашел поисковый сервер по этим запросам в некотором сегменте Интернета:

Запрос

Количество страниц (тыс. )

Динамо & Рубин

320

Спартак & Рубин

280

(Динамо | Спартак) & Рубин

430

Сколько страниц (в тысячах) будет найдено по запросу

Рубин & Динамо & Спартак

Задача 10

В таблице приведены запросы и количество страниц, которые нашел поисковый сервер по этим запросам в некотором сегменте Интернета:

Запрос

Количество страниц (тыс.)

фрегат & эсминец

500

фрегат

2000

эсминец

2500

Сколько страниц (в тысячах) будет найдено по запросу

фрегат | эсминец

Задача 11

В таблице приведены запросы и количество страниц, которые нашел поисковый сервер по этим запросам в некотором сегменте Интернета:

Запрос

Количество страниц (тыс. )

фрегат & эсминец

500

фрегат | эсминец

4500

эсминец

2500

Сколько страниц (в тысячах) будет найдено по запросу фрегат

Задача 12

В таблице приведены запросы и количество страниц, которые нашел поисковый сервер по этим запросам в некотором сегменте Интернета:

Запрос

Количество страниц (тыс.)

пирожное | выпечка

14200

пирожное

9700

пирожное & выпечка

5100

Сколько страниц (в тысячах) будет найдено по запросу

выпечка

Задача 13

Некоторый сегмент сети Интернет состоит из 1000 сайтов. Поисковый сервер в автоматическом режиме составил таблицу ключевых слов для сайтов этого сегмента. Вот ее фрагмент:

Ключевое слово

Количество сайтов, для которых данное слово является ключевым

Сомики

250

меченосцы

200

Гуппи

500

Сколько сайтов будет найдено по запросу

сомики | меченосцы | гуппи

если по запросу сомики & гуппи было найдено 0 сайтов, по запросу сомики & меченосцы – 20, а по запросу меченосцы & гуппи – 10.

Задача 14

Некоторый сегмент сети Интернет состоит из 1000 сайтов. Поисковый сервер в автоматическом режиме составил таблицу ключевых слов для сайтов этого сегмента. Вот ее фрагмент:

Ключевое слово

Количество сайтов, для которых данное слово является ключевым

Сомики

250

меченосцы

200

Гуппи

500

Сколько сайтов будет найдено по запросу

(сомики & меченосцы) | гуппи

если по запросу сомики | гуппи было найдено 750 сайтов, по запросу сомики & меченосцы – 100, а по запросу меченосцы & гуппи – 0.

Задача 15. В таблице приведены запросы и количество страниц, которые нашел поисковый сервер по этим запросам в некотором сегменте Интернета:

Количество страниц (тыс.)

Атос & Портос

335

Атос & Арамис

235

Атос & Портос & Арамис

120

Сколько страниц (в тысячах) будет найдено по запросу

Атос & (Портос | Арамис)

Задача 16. В таблице приведены запросы и количество страниц, которые нашел поисковый сервер по этим запросам в некотором сегменте Интернета:

Количество страниц (тыс.)

васильки & ландыши

650

ландыши & лютики

230

ландыши & (васильки | лютики)

740

Сколько страниц (в тысячах) будет найдено по запросу

ландыши & васильки & лютики

Задача 17. В таблице приведены запросы и количество страниц, которые нашел поисковый сервер по этим запросам в некотором сегменте Интернета:

Количество страниц (тыс.)

декабрь & январь & февраль

113

декабрь & январь

225

декабрь & (январь | февраль)

645

Сколько страниц (в тысячах) будет найдено по запросу

декабрь & февраль

Задача 18. В таблице приведены запросы и количество страниц, которые нашел поисковый сервер по этим запросам в некотором сегменте Интернета:

Запрос

Количество страниц (тыс. )

март & май & июнь

150

март & май

420

март & (май | июнь)

520

Сколько страниц (в тысячах) будет найдено по запросу

март & июнь

Задача 19. В таблице приведены запросы и количество страниц, которые нашел поисковый сервер по этим запросам в некотором сегменте Интернета:

Запрос

Количество страниц (тыс.)

март & май

472

май & апрель

425

май & (март | апрель)

620

Сколько страниц (в тысячах) будет найдено по запросу

март & апрель & май

Задача 20. В таблице приведены запросы и количество страниц, которые нашел поисковый сервер по этим запросам в некотором сегменте Интернета:

Количество страниц (тыс. )

Фрегат | Эсминец

3400

Фрегат & Эсминец

900

Фрегат

2100

Сколько страниц (в тысячах) будет найдено по запросу

Эсминец

Задача 21. В таблице приведены запросы и количество страниц, которые нашел поисковый сервер по этим запросам в некотором сегменте Интернета:

Запрос

Количество страниц (тыс.)

Пушкин | Лермонтов

5200

Лермонтов

3000

Пушкин & Лермонтов

1200

Сколько страниц (в тысячах) будет найдено по запросу

Пушкин

Задача 22.
 «Ребята из нашего класса посещали три кружка: математический, физический и химический. Списки членов этих кружков хранились у Холмса, так как он был почетным членом всех трех кружков. Однажды Ватсон решил организовать еще и кружок юных медиков. В этот кружок он решил пригласить только тех ребят, которые пока ни в какие кружки еще не были записаны. Чтобы узнать, сколько таких ребят, Ватсон обратился к Холмсу.

Холмс сказал, что всего в классе 36 человек, а кружки посещают: математический – 18 человек, физический – 14 человек, химический – 10 человек.

Ватсон удивился: «Как же это может быть? Ведь , а в классе только 36 человек». Холмс объяснил, что дело тут просто в том, что некоторые ребята ходят в два, а возможно, и в три кружка. Ватсон согласился и спросил: «А как же мы узнаем, сколько человек не посещают никаких кружков?» Холмс ответил: «Чтобы это узнать, нужно сначала взять списки математического и физического кружков и подсчитать, сколько ребят посещают оба кружка. Потом нужно сделать то же самое и с другими списками».

Через некоторое время Ватсон получил следующие данные: все три кружка посещают 2 человека, математический и физический – 8, математический и химический – 5, физический и химический – 3″. Сколько человек не посещают ни одного кружка?

Интернет ДЗ ОТВЕТЫ

ГБВА

2

ГВАБ

3

ГАВБ

4

4312

5

2143

6

4312

7

30

8

13000

9

170

10

4000

11

2500

12

9600

13

920

14

600

Интернет ДЗ ОТВЕТЫ

ГБВА

2

ГВАБ

3

ГАВБ

4

4312

5

2143

6

4312

7

30

8

13000

9

170

10

4000

11

2500

12

9600

13

920

14

600

Онлайн-вычисления областей для диаграмм Эйлера с упрощенными правилами рисования

Основные

Методология (MMPA) для эффективных онлайн-вычислений области для обобщенных диаграмм Эйлера.

Возможность ослабления условных обозначений рисования, включая зоны параллелизма и отключенные зоны.

Анализ сложности демонстрирует компромисс с методом необобщенных диаграмм Эйлера.

Реализация, реализующая алгоритмы для специализированного случая диаграмм на основе эллипсов.

Экспериментальные данные предоставлены для целей сравнительного анализа.

Abstract

Диаграммы Эйлера — это доступная и эффективная визуализация данных, включающая простые теоретико-множественные отношения. Эффективные алгоритмы для быстрого вычисления абстрактных областей диаграммы Эйлера при добавлении и удалении кривой были разработаны ранее (подход с единственной отмеченной точкой, SMPA), но был применен строгий набор соглашений о рисовании (называемых условиями правильного формирования), что означает, что некоторые абстрактные диаграммы не могут быть представлены в виде конкретных диаграмм. Мы представляем новую методологию (подход с несколькими отмеченными точками, MMPA), позволяющую вычислять области в режиме онлайн для диаграмм Эйлера при ослаблении соглашения о рисовании, согласно которому зоны должны быть соединенными областями. Кроме того, мы указываем, как расширить методы, чтобы справиться с ослаблением любого из соглашений о рисовании, при этом использование параллельных сегментов линии имеет особое значение. Мы проводим анализ сложности и сравниваем MMPA с SMPA. Мы показываем, что эти методы теоретически не хуже, чем другие компараторы, в то время как наши методы применимы к любому случаю и, вероятно, будут быстрее на практике из-за их интерактивного характера.Механизм, разработанный для случая параллелизма, может быть использован в методах рисования диаграмм Эйлера (в контексте графа Эйлера) и в компьютерной графике (например, при разработке расширенного варианта структуры данных с крылатым краем, которая имеет дело с параллелизмом). Алгоритмы представлены для общих кривых; В приложениях могут возникать такие специализации, как использование фиксированных геометрических форм для кривых, которые могут расширить возможности для быстрых вычислений входных структур алгоритмов. Мы обеспечиваем реализацию этих алгоритмов с использованием эллипсов и предоставляем временные экспериментальные данные для целей сравнительного анализа.

Ключевые слова

диаграмм Эйлера

Вычисление области

Он-лайн алгоритмы

Построение интерактивных диаграмм

Рекомендуемые статьиЦитирующие статьи (0)

© 2016 Авторы. Опубликовано Elsevier Ltd.

Рекомендуемые статьи

Цитирующие статьи

(PDF) Компьютерное представление диаграмм Венна и Эйлера

Компьютерное представление диаграмм Венна и Эйлера 6

1–2 сентября 2016 г. Международная конференция по достижениям в области ИКТ для развивающихся стран Области ICTer2016

Диаграммы 1 и 2 содержат два набора, а остальные диаграммы

содержат 3 набора.Рисунки для диаграммы 1 были собраны из

сценариев ответов школьников, а остальные

были получены от студентов университетов. На диаграммах 5 и 6 было меньше

ответов из-за большей сложности вопроса.

Все ошибки синтаксического анализа вызваны неоднозначностью текста меток

, а некоторые текстовые метки находятся слишком далеко от стрелок

, которые должны быть метками стрелок. На рис. 10 показана диаграмма Венна

с неоднозначными метками набора.Метка

«Cricket» может быть распознана как метка набора универсального набора

или метка набора, представленная первым эллипсом, так как она близка к

, ближайшему концу строки и границе прямоугольника. Метка набора

«Футбол» может быть или не быть меткой набора для набора

, представленного вторым эллипсом, но она находится слишком далеко от конца строки

. Чтобы устранить эти неоднозначности текстовых меток, необходим более высокий уровень анализа

.

V. ВЫВОДЫ

G. Будущая работа

Можно определить количество возможных дальнейших расширений

применительно к внедренной системе.

В этом исследовании наборы могут быть построены из нескольких фигур, таких как

прямоугольников, кругов и эллипсов. Этот метод может быть расширен для применения к множествам, нарисованным с помощью любого типа кривых Jordan

. В некоторых представлениях Венна и Эйлера одна метка набора

может иметь более одной кривой Жордана.Этот метод

можно расширить для обращения к этим представлениям.

Подходы машинного обучения можно использовать для улучшения классификации и кластеризации меток

. В настоящее время методы машинного обучения

не используются, поскольку у нас нет значительно большой базы данных векторных изображений

.

Этот метод может быть легко расширен на другие векторные форматы изображений

, поскольку уже разработаны методы преобразования пиксельных изображений формата

в векторный формат.Кроме того, этот метод

можно расширить для анализа распечатанных изображений.

H. Заключительные замечания

Понимание диаграмм — сложная проблема в области компьютерных исследований

.

структурированных диаграмм можно рассматривать с использованием онтологии предметной области, но понимание неструктурированных чертежей

является очень сложной проблемой. В частности,

, имеющий дело с диаграммами, нарисованными человеком, затруднен из-за проблем неопределенности

и человеческих ошибок.

В этом исследовании мы успешно установили

необходимых методов для интерпретации диаграмм Венна и Эйлера

, представленных в векторном формате SVG, и представили общий формат

для представления диаграммы Венна и Эйлера. Мы полагаем, что

, что решения, которые мы внедрили, помогут разработать

систем, связанных с пониманием изображений, таких как автоматические системы оценки

и системы баз данных изображений.

ПОДТВЕРЖДЕНИЕ

Это исследование финансируется исследовательским грантом Сената Моратувы (SRC) Университета

2015 года и исследовательским грантом DL4D 2016

.

СПРАВОЧНАЯ ИНФОРМАЦИЯ

[1]. Р. П. Футрелле, И. Какадиарис, Дж. Александер, К. М. Карриеро, Н.

Николакис и Дж. М. Футрелле, «Понимание диаграмм в технических документах

», IEEE Computer, 1992, Vol. 25 (7), pp. 75-

78.

[2].Р. П. Футрелле и Н. Николакис, «Эффективный анализ сложных диаграмм

с использованием синтаксического анализа на основе ограничений», в «Анализ документов

и распознавание», в материалах Третьей международной конференции

по IEEE, 1995 г., том. 2. С. 782-790.

[3]. Р. Футрелл, «Неоднозначность в теории визуального языка и ее роль в анализе диаграмм

», в Proceedings 1999 IEEE Symposium on

Visual Languages, 1999.

[4].В. Хуанг, К. Тан и В. К. Леоу (2005, август).

«Связывание текста и графики для научной диаграммы

понимания» в Анализе и распознавании документов, в

Proceedings of the VIII International Conference on IEEE,

2005, pp. 580-584.

[5]. В. Хуанг, К. Тан, «Система для понимания визуализированной инфографики

и ее приложений», в материалах симпозиума

ACM по проектированию документов 2007 г. — DocEng ’07, 2007.

[6]. К. Целонис, Дж. Сарджант и М. М. Вуд, «Соответствие диаграммы

для совместной оценки человека и компьютера», в Proceedings

9-й конференции CAA, 2005.

[7]. Р. Ф. Ганстон и Р. Т. Уайт (1986). «Оценка понимания

с помощью диаграмм Венна», Science Education,

Vol 70 (2), pp. 151-158, 1986.

[8]. А. Цинцифас, «Структура компьютерной оценки

курсовой работы на основе диаграмм», в компьютерных науках и

информационных технологиях, 2002.

[9]. Н. Смит, П. Томас и К. Во, «Интерпретация неточных диаграмм

», Диаграммное представление и вывод.

Springer Berlin Heidelberg, 2004. 239-241.

[10]. П. Томас, К. Во и Н. Смит, «Эксперименты по автоматической маркировке

ER-диаграмм», Бюллетень ACM SIGCSE,

Vol 37 (3), стр. 158-162, 2005.

[ 11]. П. Томас, К. Во и Н. Смит, «Обобщенное построение диаграмм

инструментов с автоматической маркировкой», Инновации в обучении и

обучения в области информационных и компьютерных наук, том 10 (1), стр.

22-34, 2011.

[12]. П. Томас, К. Во и Н. Смит, «Использование шаблонов в

автоматической разметке ER-диаграмм», Бюллетень ACM SIGCSE.

Том. 38 (3), 2006.

[13]. Дж. Амбикеш и М. Шепперд, «Проблема меток в оценке диаграмм E-

«, Журнал образовательных ресурсов в

Computing (JERIC), том 8 (4), стр. 12, 2009.

[ 14]. К. Во, П. Томас и Н. Смит, «На пути к автоматизированной оценке диаграмм сущность-отношения

», Второй семинар

Сети поддержки обучения и преподавания —

Информация и информатика (TLAD), 2004 г.

[15]. Б. Блай и К. А. Хиггинс, «Формирующая компьютерная оценка

в областях, основанных на диаграммах». Бюллетень ACM SIGCSE,

Том 38 (3), стр. 98-102, 2006

[16]. М. Андерсон и Р. Маккартни, «Обработка диаграмм:

Вычисления с диаграммами», «Искусственный интеллект», том 145 (1),

стр. 181-226, 2003.

[17]. Э. Фоксли, К. Хиггинс, Т. Хегази, П. Симеонидис и А.

Цинцифас. «Система coursemaster cba: улучшения по сравнению с

ceilidh», в The Computer-Assisted Assessment Conference,

2001.

[18]. Ф. Батмаз и К. Дж. Хайнде, «Инструмент для рисования диаграмм для полуавтоматической оценки концептуальных диаграмм баз данных

», в материалах

Proceedings of the 10th CAA Conference, 2006.

[19]. Г. Хоггарт и М. Локьер, «Автоматическая система оценки учеников

», Бюллетень ACM SIGCSE, Vol. 30 (3), 1998.

[20]. П. А. Роджерс, «Обзор диаграмм Эйлера», Journal of Visual

Languages ​​& Computing, том 25 (3), стр.134-155, 2014.

[21]. (2014) Векторная графика [Online]. Доступно:

https://en.wikipedia.org/wiki/Vector_graphics

% PDF-1.3
%
204 0 объект>
эндобдж

xref
204 91
0000000016 00000 н.
0000003133 00000 п.
0000003311 00000 н.
0000002116 00000 п.
0000003359 00000 н.
0000003487 00000 н.
0000004025 00000 н.
0000004091 00000 н.
0000004237 00000 п.
0000004383 00000 п.
0000004529 00000 н.
0000004675 00000 н.
0000004850 00000 н.
0000004996 00000 н.
0000005407 00000 н.
0000012476 00000 п.
0000012990 00000 п.
0000013043 00000 п.
0000013254 00000 п.
0000013654 00000 п.
0000017653 00000 п.
0000017965 00000 п.
0000018052 00000 п.
0000018366 00000 п.
0000019177 00000 п.
0000019393 00000 п.
0000019486 00000 п.
0000019809 00000 п.
0000020891 00000 п.
0000021122 00000 п.
0000021360 00000 п.
0000021764 00000 п.
0000027278 00000 н.
0000027624 00000 н.
0000028196 00000 п.
0000028947 00000 п.
0000029180 00000 п.
0000029823 00000 п.
0000030414 00000 п.
0000030450 00000 п.
0000030855 00000 п.
0000033914 00000 п.
0000034171 00000 п.
0000036464 00000 н.
0000036839 00000 п.
0000036992 00000 п.
0000037473 00000 п.
0000044202 00000 п.
0000044414 00000 п.
0000045572 00000 п.
0000045901 00000 п.
0000045989 00000 п.
0000046440 00000 п.
0000047135 00000 п.
0000047509 00000 п.
0000047916 00000 п.
0000048514 00000 п.
0000048722 00000 п.
0000049273 00000 п.
0000049573 00000 п.
0000049650 00000 п.
0000049820 00000 н.
0000050489 00000 п.
0000050693 00000 п.
0000051180 00000 п.
0000051353 00000 п.
0000051558 00000 п.
0000051951 00000 п.
0000055375 00000 п.
0000055672 00000 п.
0000055742 00000 п.
0000056032 00000 п.
0000056069 00000 п.
0000056273 00000 п.
0000056729 00000 п.
0000057916 00000 п.
0000058045 00000 п.
0000062071 00000 п.
0000064741 00000 п.
0000064845 00000 п.
0000065191 00000 п.
0000066509 00000 п.
0000066723 00000 п.
0000076197 00000 п.
00000

00000 н.
0000091903 00000 п.
0000091955 00000 п.
0000092008 00000 н.
0000092061 00000 п.
0000092114 00000 п.
0000092169 00000 п.
трейлер
] >>
startxref
0
%% EOF

207 0 obj> поток
x ڤ TKLQ} 3LN ӢX ~ u- t`, 1Q (ЊBA> N
1aAbkĉ $ nHlh] 7 @% 1 $ t ޽ s {

Определение диаграммы Венна

Что такое диаграмма Венна?

Диаграмма Венна — это иллюстрация, в которой круги используются для отображения отношений между вещами или конечными группами вещей. Перекрывающиеся круги имеют общие черты, в то время как круги, которые не перекрываются, не имеют этих черт.

Диаграммы Венна помогают визуально представить сходства и различия между двумя концепциями. Их давно признали за их полезность в качестве учебных пособий. С середины 20 века диаграммы Венна использовались как часть вводной программы обучения логике и в учебных планах начального уровня по всему миру.

Ключевые выводы
  • Диаграмма Венна использует круги, которые перекрываются или не перекрываются, чтобы показать общие черты и различия между вещами или группами вещей.
  • Вещи, которые имеют общие черты, показаны в виде перекрывающихся кругов, в то время как отдельные элементы выделяются отдельно.
  • Диаграммы Венна теперь используются в качестве иллюстраций в бизнесе и во многих академических областях.

Понимание диаграммы Венна

Английский логик Джон Венн популяризировал диаграмму в 1880-х годах. Он назвал их кругами Эйлера в честь швейцарского математика Леонарда Эйлера, создавшего аналогичные диаграммы в 1700-х годах.

Термин диаграмма Венна не появлялся до 1918 года, когда Кларенс Льюис, американский академический философ и возможный основатель концептуального прагматизма, в своей книге «Обзор символической логики» назвал круговое изображение диаграммой Венна.»

Диаграммы Венна используются с середины 20 века в классах от уровня начальной школы до вводной логики.

Венн изучал и преподавал логику и теорию вероятностей в Кембриджском университете, где он разработал свой метод использования диаграмм для иллюстрации раздела математики, известного как теория множеств.

Венн опубликовал прецедентную работу «Логика случая», в которой объяснил частотную теорию вероятности. В нем он утверждал, что вероятность, вопреки популярному предположению, должна быть установлена ​​на основе регулярности, с которой что-то предсказывается.

В другой книге, Symbolic Logic, Venn основаны и развиты теории алгебры математика Джорджа Буля. Эта работа помогла ему разработать диаграмму Венна.

Приложения для диаграмм Венна

Диаграммы Венна используются для отображения того, как элементы соотносятся друг с другом на общем фоне, юниверсе, наборе данных или среде. Диаграмму Венна можно использовать, например, для сравнения двух компаний в одной отрасли, иллюстрируя продукты, предлагаемые обеими компаниями (где круги перекрываются), и продукты, которые являются эксклюзивными для каждой компании (внешние круги).

Диаграммы Венна на базовом уровне представляют собой простые графические представления отношений, существующих между двумя наборами вещей. Однако они могут быть намного сложнее. Тем не менее, упрощенная цель диаграммы Венна для иллюстрации концепций и групп привела к их популярному использованию во многих областях, включая статистику, лингвистику, логику, образование, информатику и бизнес.

Примеры диаграмм Венна

Можно нарисовать диаграмму Венна, чтобы проиллюстрировать фрукты красного или оранжевого цвета.Ниже мы видим, что есть оранжевые фрукты (круг B), такие как хурма и мандарины, а яблоки и вишня (круг A) имеют красный цвет. Перец и помидоры бывают красного и оранжевого цветов, что представлено областью перекрытия двух кругов.

Изображение Джули Банг © Investopedia 2020

Вы также можете нарисовать диаграмму Венна, чтобы решить, какую из двух машин купить. На диаграмме Венна показаны особенности, присущие каждому автомобилю, а также функции, присущие обоим автомобилям.

Ниже мы видим, что Car A — это седан, который приводится в действие бензином и имеет скорость 20 миль на галлон, а Car B — гибрид, который получает 40 миль на галлон за пробег и является хэтчбеком.

Изображение Джули Банг © Investopedia 2020

Заштрихованная область, где два круга перекрываются, показывает общие черты обоих автомобилей, в том числе радио, четыре двери, возможность Bluetooth и подушки безопасности.

Диаграмма Венна графически отображает сходства и различия между двумя автомобилями, чтобы помочь решить, какой из них купить.

Часто задаваемые вопросы о диаграмме Венна

Что такое диаграмма Венна в математике?

Диаграмма Венна в математике используется в теории логики и теории множеств, чтобы показать различные наборы или данные и их взаимосвязь друг с другом.

Как читать диаграмму Венна?

Диаграмму Венна можно прочитать, наблюдая за всеми кругами, составляющими всю диаграмму. Каждый круг — это отдельный элемент или набор данных. Части кругов, которые перекрываются, указывают на области, которые являются общими для различных элементов, тогда как части, которые не перекрываются, указывают на уникальные черты элемента или набора данных, представленных кругом.

Почему они называются диаграммами Венна?

Они называются диаграммами Венна, потому что диаграмма была разработана английским логиком Джоном Венном.

Как называется середина диаграммы Венна?

Середина диаграммы Венна, где два или более набора перекрываются, называется пересечением.

Компьютерные науки для развлечения — cs4fn: Головоломки викторианской эпохи

Пол Керзон, Лондонский университет королевы Марии


Похоронен в архиве личных документов Ады в Бодлианской библиотеке.
в Оксфорде — это лист бумаги, который на первый взгляд выглядит как
куча дудлов со странными каракулями в Аде Лавлейс и Чарльзе
Рука Бэббиджа. К тому времени они пытались решить вековую
головоломка, которая заложила основу основного способа, которым компьютерные ученые
теперь представляют данные.

Их рисунки включают в себя простую карту, которую любой компьютерный ученый мог бы
узнайте — на нем изображены мосты Кенигсберга в Пруссии. Другие дудлы
показать варианты с другим набором мостов. Город был разделен на
два у реки Прегель, с двумя большими островами посередине. В
острова были соединены с двумя берегами реки и друг с другом
серия из 7 мостов.Еще в 1700-х годах он использовался как основа
головоломка. Не могли бы вы придумать пешеходную экскурсию по всему городу,
потребует пересечения каждого моста один раз, но только один раз?

Загадку решил математик Леонард Эйлер еще в 1736 году, но
это было не столько его решение, сколько способ его решения
это было революционным. Он использовал то, что сейчас является основными частями компьютера.
инструментарий вычислительного мышления ученого.

Эйлер понял, что загадку легче решить, если бросить
Вдали большую часть информации на карте города.Это просто
вычислительное мышление идея абстракции: проблемы легче решать
если скрыть всю ненужную информацию. Все что нужно для моста
Проблема заключается в информации, показывающей различные участки суши и то, как они
соединены (то есть мосты между ними). Он сделал это очень просто
и нарисовал маленькие кружки для каждого массива суши (по кружку для каждого из двух
острова и два берега реки). Затем он провел линии между каждым
пара кругов, соединенных перемычками. Компьютерный ученый
называет изображение или «представление» графом.Линии
назвал «ребра», а окружности «узлами» графа. Проблема
теперь встает вопрос о том, можете ли вы посетить каждый узел на графике
следуя каждому краю один раз и только один раз. Упрощенная картина, представленная
граф позволял Эйлеру видеть, в сочетании с некоторыми логическими
мышление, ответ на проблему.

Эти идеи легли в основу того, что сейчас называется теорией графов.
Так же, как они помогли Эйлеру найти более простое решение загадки, графики
теперь помогают компьютерным ученым решать всевозможные проблемы и являются
основы всех видов вычислений, начиная с того, как спутниковые навигаторы находят лучшее
маршрутов к тому, как компьютеры упорядочивают данные, чтобы упростить поиск.Его
неудивительно, что Ада и Чарльз сочли загадку интересной.
Удивительно, но несмотря на то, что есть множество каракулей, которые выглядят как графики,
ни один не соответствует графику Эйлера головоломки. На рисунках показаны маршруты, которые
решать вариации придуманной ими головоломки, предположительно по мере того, как они
исследовал проблему. Нацарапанная заметка также предполагает, что они действительно потренировались.
решение реальной головоломки тоже.

Что такое диаграмма Венна с примерами

Что такое

Диаграмма Венна ?

Термин Диаграмма Венна не является чуждым, поскольку у всех нас была математика, особенно теория вероятностей и алгебра. Теперь для непрофессионала диаграмма Венна представляет собой наглядную демонстрацию всех возможных реальных отношений между коллекцией различных наборов предметов. Он состоит из нескольких перекрывающихся кругов или овальных форм, каждая из которых представляет собой отдельный набор или предмет.

Диаграммы Венна отображают сложные теоретические взаимосвязи и идеи для лучшего и легкого понимания. Эти диаграммы также профессионально используются профессорами для отображения сложных математических концепций, классификации в науке и разработки стратегий продаж в бизнес-индустрии.

Источник изображения : pinterest.com

Эволюция диаграммы Венна

Развитие диаграммы Венна восходит к 1880 году, когда Джон Венн воплотил их в жизнь в статье под названием «О схематическом и механическом представлении суждений и рассуждений». Она была опубликована в Philosophical Magazine и Journal of Science. Джон Венн провел тщательное исследование этих диаграмм и предвидел их формализацию. Он — тот, кто первоначально их обобщил, неудивительно их название, то есть Диаграммы Венна в 1918 году.

Существует небольшой разрыв между диаграммой Венна и диаграммой Эйлера, изобретенной в 18 веке Леонардом Эйлером, который также приложил руку к ее развитию в 1700-х годах. Иоанн называл диаграммы кругами Эйлера.

Разработка диаграмм Венна продолжалась и в 20 веке. Например, примерно в 1963 году Д. В. Хендерсон обнаружил существование n-графа Венна, состоящего из n-кратной рациональной симметрии, что указывало на то, что n было простым числом.В эту концепцию в последующие годы углубились четыре других интеллекта, которые пришли к выводу, что вращательно-симметричные диаграммы Венна существуют только в том случае, если n — простое число.

С тех пор эти диаграммы стали частью сегодняшней учебной программы и иллюстрируют бизнес-информацию. Диаграммы Венна и Эйлера были включены в качестве компонента обучения теории множеств нового математического движения в 1960 году.

Почему диаграммы Венна важны?

Диаграммы Венна полезны в качестве учебных и учебных пособий для ученых, учителей и профессоров.Они помогают представлять простые математические концепции в начальных школах, а также теории и задачи ракетостроения среди логиков и математиков.

Кроме того, вместе с теорией множеств, диаграммы Венна способствовали более четкому и современному пониманию бесконечных чисел и действительных чисел в математике. Они также способствовали созданию общего языка и системы символов, касающихся теории множеств, среди исследователей и математиков.

Они идеально подходят для иллюстрации сходства и различий между предметами или идеями, когда круги перекрываются или иначе.Эта функция обычно используется в бизнес-индустрии для поиска и создания ниши на рынке товаров и услуг. Благодаря им предприниматели получают невероятные отчеты о продажах и получают огромную реализованную прибыль.

Вы также можете использовать диаграмму Венна , чтобы принимать важные жизненные решения, например, в какой колледж поступить, в какую школу взять вашего ребенка, какие материалы лучше использовать для конструирования или изготовления одежды, в каком ресторане пообедать и т. Д.

Когда использовать диаграммы Венна?

Вы можете использовать диаграммы Венна , чтобы продемонстрировать взаимосвязи между статистикой, логикой, вероятностью, лингвистикой, информатикой, организацией бизнеса и многими другими областями.

  • В математике Диаграммы Венна — это обучающий инструмент, который объясняет такие математические понятия, как множества, объединения и пересечения. Они также решают серьезные задачи по высшей математике. Вы можете подробно прочитать о них в академических журналах в своей библиотеке и поразиться тому, насколько теория множеств является законченным разделом математики.

    Статистики используют идею диаграмм Венна , чтобы предсказать вероятность конкретных событий.То же самое и в области прогнозной аналитики. Наборы выборочных данных сравниваются и исследуются, чтобы выявить их сходства и различия.

Источник изображения : pinterest. com

  • Они также эффективны при определении логических оснований в аргументах и ​​выводах. Как и в дедуктивном рассуждении, если посылки реальны, а форма аргумента верна, результат должен быть правильным.Диаграмма, аналогичная диаграмме Венна по логике, — это таблица истинности. Он помещает переменные в столбцы, чтобы расшифровать то, что логически возможно. Еще одна диаграмма Рэндольфа, также известная как R-диаграмма, использует линии для объяснения множеств.

Источник изображения : youtube.com

  • В лингвистике Диаграммы Венна помогают узнать, как языки различаются или соотносятся друг с другом с точки зрения алфавита, гласных, произношения и т. Д.

Источник изображения : slideshare.net

Источник изображения : kdnuggets.com

  • Диаграммы также полезны в сфере продаж и маркетинга для сравнения и сопоставления продуктов, услуг, процессов и всего, что происходит при организации бизнеса. Они практичны и эффективны в улучшении продаж и прибылей, а также в расширении деятельности предприятий.

Источник изображения : businessbullet.co.uk

Символы на диаграмме Венна

Когда дело доходит до диаграммы Венна, существует множество символов, но мы рассмотрим три.
ꓵ — пересечение двух наборов: показывает элементы, общие для обоих наборов.

Источник изображения : youtube.com

∪ — это представляет собой полная диаграмма Венна.

Источник изображения : math-only-math.com

A ’- обозначает завершение набора A. Он состоит из всего, что не входит в коллекцию.

Источник изображения : mathonline.wikidot.com

Примеры диаграмм Венна

Математика

Первый пример диаграммы Венна относится к математике.Они доступны при освещении тем, посвященных теории множеств и теории вероятностей.

На схеме ниже представлены два набора: A = {1, 5, 6, 7, 8, 9, 10, 12} и B = {2, 3, 4, 6, 7, 9, 11, 12, 13}. Раздел, в котором два набора перекрываются, имеет числа, содержащиеся в обоих наборах A и B, называемый пересечением A и B. Два набора, вместе взятые, дают их объединение, которое включает в себя все объекты в A, B, которые являются { 1 2 3 4 5 6 7 8 9 10 11 12 13}.

Источник изображения : bbc.co.uk

Бизнес

В приведенном ниже примере диаграммы Венна анализируются сходства и различия в различных областях работы. Менеджеры по персоналу и специалисты по карьерной лестнице используют его, чтобы консультировать людей по поводу их карьерного пути.

Источник изображения : pinterest.com

Наука

Ученый использует диаграммы Венна для изучения здоровья человека и лекарств. На иллюстрации ниже вы можете увидеть аминокислоты, жизненно важные для человека.

Источник изображения : researchgate.com

Как создать простую диаграмму Венна за считанные минуты?

Теперь мы будем использовать онлайн-программное обеспечение Edraw Max.В нем есть все основные символы и формы, которые вам нужны, а также множество бесплатных шаблонов диаграмм Венна , и модный и продвинутый интерфейс, понятный для начинающих.

Перед тем, как начать онлайн-диаграмму Венна , вы должны убедиться, что вы:

  • Определите цель, которую вы хотите достичь. Имейте четкое представление о том, что вы хотели бы сравнить и для какой цели это сравнение необходимо. Это облегчает определение множеств.
  • Просмотрите и найдите список предметов, содержащихся в наборах.
  • Просмотрите доступные шаблоны, чтобы получить представление о том, что вы собираетесь рисовать, а затем создайте свою собственную диаграмму Венна , выполнив следующие действия.

Шаг 1: Войдите на веб-сайт программного обеспечения с https://www.edrawmax.com/online/ . Если вы не создавали учетную запись ранее, войдите в систему, используя действительные учетные данные, подтвердите свою учетную запись, а затем войдите в систему.

Шаг 2: Выберите параметры бизнес-диаграммы на вкладке «Доступные шаблоны» и дважды щелкните значок диаграммы Венна, чтобы отобразить пустую страницу, на которой вы будете рисовать.

Шаг 3: На левой панели экрана вы найдете все необходимые символы и формы диаграммы Венна. Перетащите подходящие и поместите их на холст для рисования, чтобы создать диаграмму Венна.

Шаг 4: Сохраните готовую диаграмму Венна в доступных форматах или экспортируйте или поделитесь ею на других платформах прямо с веб-страницы Edraw.

Шаг 5: Настройка. Большинство встроенных фигур предназначены для изменения размера, редактирования и изменения цвета.

  • Чтобы добавить личную тему и стиль, выберите один из доступных шрифтов, эффектов и цветовых схем. Создайте уникальную и профессиональную диаграмму Венна, щелкнув то, что вам больше нравится.

Статьи по теме

Диаграмма Венна

— обзор, символы, примеры, преимущества

Что такое диаграмма Венна?

Диаграмма Венна — это схематическое представление элементов в наборе или группе.Это диаграмма, которая показывает все возможные логические отношения между конечным набором наборов или групп. Это также называется установленной схемой или логической схемой.

Диаграмма Венна использует несколько перекрывающихся форм (обычно кругов), представляющих наборы различных элементов. Он направлен на обеспечение графической визуализации элементов, подчеркивая сходства и различия между ними.

Они в основном используются в теории множеств, а также для иллюстрации взаимосвязей между элементами в различных областях, таких как статистика Основные концепции статистики для финансов Твердое понимание статистики имеет решающее значение для того, чтобы помочь нам лучше понять финансы. Более того, концепции статистики могут помочь инвесторам отслеживать, логику, вероятность, лингвистику, бизнес и информатику. В приведенном выше примере диаграммы Венна показаны три набора, помеченные X, Y и Z, и соответствующие отношения между элементами в каждом наборе.

Диаграммы Венна обеспечивают мощное визуальное отображение данных, обычно используемых в презентациях, деловых и научных отчетах. Они связаны с диаграммами Эйлера, которые отличаются только тем, что они не иллюстрируют набор, если в нем отсутствуют элементы.

Символы диаграмм Венна

Union (∪) : представляет объединение всех наборов, то есть совокупность всех элементов в наборах X и Y.

Пересечение (∩) : представляет все элементы, общие или общие в выбранных наборах или группировках. Пересечение представляет общие элементы (посередине) в наборах X и Y.

Дополнение (X C ) : представляет все, что не представлено в конкретном наборе; в этом случае все, чего нет в наборе X. Уравнение для иллюстрации дополнения к X: X C = U / A, где U представляет заданную совокупность элементов. На приведенной ниже диаграмме показано абсолютное дополнение X в U, то есть все во вселенной, кроме X (серая область).

История диаграмм Венна

Концепция диаграммы Венна была основана британским математиком и логиком Джоном Венном. Впервые он был опубликован в его журнале 1980 года под названием «О схематическом и механическом представлении суждений и рассуждений».Однако развитие диаграмм Венна можно проследить до 1200-х годов через философа и логика Рамона Лулля, который нарисовал подобные типы диаграмм.

Существуют различные другие логики, которые также рисовали подобные диаграммы, но наиболее близкие диаграммы, напоминающие диаграммы Венна, были впервые нарисованы Леонардом Эйлером в 1700-х годах. Он нарисовал то, что он определил как диаграммы Эйлера. Джон Венн ссылался на Эйлера в своих диаграммах, которые он сначала описал как круги Эйлера.

Термин диаграмма Венна впервые был опубликован Кларенсом Ирвином Льюисом в его книге 1918 года «Обзор символической логики.Математики и логики продолжали улучшать диаграммы в 19 и 20 веках, чтобы показать более четкие и сложные взаимосвязи с использованием большего количества наборов. Диаграммы Венна были приняты в различных дисциплинах и сложностях, чему способствовали развитие технологий и использование компьютеров.

Использование диаграмм Венна

Как отмечалось выше, диаграммы Венна используются несколькими способами, чтобы показать взаимосвязи между различными элементами набора. Ниже приведены примеры использования диаграмм Венна.

Пример 1: Предметы, изучаемые учениками

В школе проводится исследование учеников, изучающих математику и экономику. 12 учеников посещают оба класса и 2 ученика, которые не изучают ни один из предметов.

Математика ∪ Экономика : {Шарлотта, Льюис, Оливия, Эдди, София, Лиам, Бри, Эйвери, Ноа, Изабелла, Миа, Эрик}. Иллюстрация ниже:

Объединение двух предметов — это совокупность всех учащихся, которые посещают оба класса — i.э., 12 учеников.

Математика ∩ Экономика : {София, Лиам, Бри, Эйвери, Ноа}

Выше показано пересечение студентов, изучающих математику и экономику. Это студенты, которые изучают оба предмета, но не изучают только один из них.

Математика C : {Изабелла, Миа, Эрик, Джо, Нина}

Математика представляет всех учащихся, не изучающих математику.К ним относятся студенты, изучающие экономику, но не изучающие математику, и студенты, не изучающие ни один из предметов (серая зона).

Пример 2: Коллективные инвестиционные фонды

В приведенной выше таблице сравниваются восемь инвестиционных фондов с точки зрения типов фондов, которыми управляет каждый фонд. Рассматриваются различные типы фондов, в частности фонд акций, фонд денежного рынка Фонды денежного рынка Фонды денежного рынка — это открытые паевые инвестиционные фонды с фиксированным доходом, которые инвестируют в краткосрочные долговые ценные бумаги, такие как казначейские векселя, муниципальные векселя и гибридный фонд ( я. е., сочетание долевых инструментов и инструментов денежного рынка) и хедж-фонда. Восемь инвестиционных фондов управляют одним или несколькими из этих типов фондов.

Благодаря четкой цветовой кодировке диаграмма Венна ясно показывает, где находится каждый фонд. Несколько наблюдений на приведенной выше диаграмме обсуждаются ниже:

  • Все инвестиционные фонды, за исключением хедж-фондов, представлены следующими обозначениями:

Equity Fund ∪ Hybrid Fund ∪ Money Market Fund : {AB Fund , SM Fund, GW Fund, ZK Fund, FC Fund, MX Fund, DD Fund}

  • Только два фонда {SM Fund, GW Fund} имеют все три типа фондов, за исключением хедж-фонда.Это представлено следующим обозначением:

Фонд акций ∩ Фонд денежного рынка ∩ Гибридный фонд

  • Фонд AB — единственный фонд, который имеет только фонды как акций, так и денежного рынка. Это представлено следующим обозначением:

Фонд акций ∩ Фонд денежного рынка

  • Фонд DD — единственный фонд, который имеет только акционерный и гибридный фонды. Это представлено следующим обозначением:

Фонд акций ∩ Гибридный фонд

  • Фонд ZK — единственный фонд без других типов фондов, за исключением фонда денежного рынка.
  • FC Fund и MX Fund — единственные фонды без других типов фондов, за исключением гибридного фонда.
  • Нет фондов с исключительно денежным рынком и гибридными фондами.
  • Нет фондов, которые имеют исключительно фонды собственного капитала.
  • HD Fund — единственный фонд, у которого есть хедж-фонд, и он не управляет никакими другими инвестиционными фондами.
Диаграммы Венна цилиндров

Диаграммы Венна не только показывают перекрытия, но также могут отображать подмножества в более крупном наборе или группе.Ниже мы проиллюстрируем Британские острова, на котором четко показаны подмножества каждого острова из большей группы Британских островов.

Создание диаграммы Венна

Диаграмму Венна можно создать несколькими способами. Ниже приведены различные инструменты, используемые для построения диаграмм Венна:

  • С помощью фигур в Microsoft Office (Word, Excel, PowerPoint)
  • С помощью SmartArt в Microsoft Office (Word, Excel, PowerPoint)
  • С помощью онлайн-инструментов для рисования, таких как canva, SmartDraw, visual-paradigm, visme и creately, среди прочего.
Преимущества диаграмм Венна
  • Визуальная организация : Диаграммы Венна помогают визуально отображать информацию, что помогает студентам и профессионалам увидеть логику взаимосвязей отдельных элементов.
  • Помощь в принятии решений : Диаграммы Венна помогают принимать решения между двумя или более вариантами. Это упрощает сравнение и противопоставление. Следовательно, использование диаграмм Венна для оценки имеет тенденцию вызывать дискуссии и предоставляет информацию о мышлении участников, что в конечном итоге помогает в принятии решений.
  • Размышляйте через логику : Диаграммы Венна помогают логически обосновать сложные проблемы. Математические задачи легко сводятся к ясному и понятному формату.
  • Обнаружение шаблонов данных : легче обнаруживать шаблоны данных, которые могли быть неочевидными. Такие закономерности, как вероятности и корреляции, легко вывести.
Области применения диаграммы Венна
  • Теория множеств : Теория множеств — это раздел математики, в котором находятся такие понятия множеств, как объединения, пересечения и дополнения.Он имеет дело с обширными и сложными проблемами, которые решаются с помощью диаграмм Венна.
  • Логика : В логике диаграммы Венна используются для определения достоверности определенных аргументов и выводов. Он также использует дедуктивное рассуждение. Они полезны в практике логических утверждений, таких как если / тогда, все / некоторые / нет, может быть.
  • Статистика и вероятность : Диаграммы Венна используются в области статистики и вероятности, которая имеет дело с прогнозированием вероятности наступления события. Они тесно связаны с прогнозным анализом.
  • Преподавание : Диаграммы Венна также используются в педагогической профессии, особенно в младших классах, для помощи учащимся в понимании прочитанного. Студенты могут понимать концепции и более четко объяснять сходства и различия между преподаваемыми элементами.
  • Лингвистика : Диаграммы Венна также используются в лингвистике, чтобы показать отношения между разными языками. Эволюция языков с течением времени делает возможными общности и различия между языками, возникшими на основе общего родного языка.
  • Бизнес : В бизнесе диаграммы Венна применяются по-разному. Многие финансовые аналитики — чем они занимаются, и экономисты — используют диаграммы Венна в презентациях клиентам, инвесторам, поставщикам и т. Д. Их можно использовать для демонстрации взаимосвязей в продуктах, процессах, SBU, идеях и многих других вещах.
  • Информатика : Приложение создается посредством визуализации языков программирования и структур.
Дополнительные ресурсы

CFI предлагает сертификацию коммерческого банковского и кредитного аналитика (CBCA) ™ CBCA®. Аккредитация коммерческого банковского и кредитного аналитика (CBCA) ™ является мировым стандартом для кредитных аналитиков, который охватывает финансы, бухгалтерский учет, кредитный анализ, анализ денежных потоков, моделирование ковенантов, погашение ссуд и многое другое.программа сертификации для тех, кто хочет вывести свою карьеру на новый уровень. Чтобы продолжить изучение и развитие своей базы знаний, ознакомьтесь с дополнительными соответствующими ресурсами ниже:

  • Шаблоны диаграмм и графиков Шаблон диаграмм и графиков Этот шаблон диаграмм и графиков предоставляет вам 10 различных типов диаграмм и графиков, используемых в финансовом планировании и анализе.

Как построить круги эйлера в презентации. Презентация — решение задач с помощью кругов эйлера. Презентация

Эйлеровы круги (круги Эйлера).

Цель урока: Познакомить обучающихся с решением простейших логических задач методом кругов Задачи урока Образовательная: дать обучающимся представление о методе кругов Эйлера; Развивающая: развитие логического и аналитического мышления; Воспитательная: воспитание умения выслушивать мнение других обучающихся и отстаивать свою точку зрения.

Эйлеровы круги (круги Эйлера) — принятый в логике способ моделирования, наглядного изображения отношений между объемами понятий с помощью кругов, предложенный знаменитым математиком Л. Эйлером (1707–1783). Обозначение отношений между объемами понятий посредством кругов было применено еще представителем афинской неоплатоновской школы — Филопоном (VI в.), написавшим комментарии на «Первую Аналитику» Аристотеля.

1.Условно принято, что круг наглядно изображает объем одного какого-нибудь понятия. Объем же понятия отображает совокупность предметов того или иного класса предметов. Поэтому каждый предмет класса предметов можно изобразить посредством точки, помещенной внутри круга:

2. Группа предметов, составляющая вид данного класса предметов, изображается в виде меньшего круга, нарисованного внутри большего круга. Такое именно отношение существует между объемами понятий «небесное тело» (А) и «комета» (B). Объему понятия «небесное тело» соответствует больший круг, а объему понятия «комета» — меньший круг. Это означает, что все кометы являются небесными телами. Весь объем понятия «комета» входит в объем понятия «небесное тело».

3 . Когда же ни один предмет, отображенный в объеме понятия A, не может одновременно отображаться в объеме понятия B, то в таком случае отношение между объемами понятий изображается посредством двух кругов, нарисованных один вне другого. Ни одна точка, лежащая на поверхности одного круга, не может оказаться на поверхности другого круга. Такое именно отношение существует, например, между понятиями «тупоугольный треугольник» и «остроугольный треугольник». В объеме понятия «тупоугольный треугольник» не отображается ни один остроугольный треугольник, а в объеме понятия «остроугольный треугольник» не отображается ни один тупоугольный треугольник.

4 . Иначе выглядит схема отношения между объемами субъекта и предиката в общеутвердительном суждении, не являющемся определением понятия. В таком суждении объем предиката больше объема субъекта, объем субъекта целиком входит в объем предиката. Поэтому отношение между ними изображается посредством большого и малого кругов, как показано на рисунке:

5.Отношения между равнозначащими понятиями, объемы которых совпадают, отображаются наглядно посредством одного круга, на поверхности которого написаны две буквы, обозначающие два понятия, имеющие один и тот же объем: Такое отношение существует, например, между понятиями «родоначальник английского материализма» и «автор „Нового Органона“». Объемы этих понятий одинаковы, в них отобразилось одно и то же историческое лицо — английский философ Ф. Бэкон.

6 . Нередко бывает и так: одному понятию (родовому) подчиняется сразу несколько видовых понятий, которые в таком случае называются соподчиненными. Отношение между такими понятиями изображается наглядно посредством одного большого круга и нескольких кругов меньшего размера, которые нарисованы на поверхности большего круга: Такое именно отношение существует между понятиями «скрипка», «флейта», «пианино», «рояль», «барабан». Эти понятия в равной мере подчинены одному общему родовому понятию «музыкальные инструменты».

7. В тех случаях, когда между понятиями имеется отношение противоположности, отношение между объемами таких понятий отображается посредством одного круга, обозначающего общее для обоих противоположных понятий родовое понятие, а отношение между противоположными понятиями обозначается так: А — родовое понятие, B и C — противоположные понятия. Противоположные понятия исключают друг друга, но входят в один и тот же род, что можно выразить такой схемой: При этом видно, что между противоположными понятиями возможно третье, среднее, так как они не исчерпывают полностью объема родового понятия. Такое именно отношение существует между понятиями «легкий» и «тяжелый». Они исключают друг друга. Нельзя об одном и том же предмете, взятом в одно и то же время и в одном и том же отношении, сказать, что он и легкий, и тяжелый. Но между данными понятиями есть среднее, третье: предметы бывают не только легкого и тяжелого веса, но также и среднего веса.

8.Когда же между понятиями существует противоречащее отношение, тогда отношение между объемами понятий изображается иначе: круг делится на две части так: А — родовое понятие, B и не-B (обозначается как B) — противоречащие понятия. Противоречащие понятия, исключают друг друга и входят в один и тот же род, что можно выразить такой схем ой: При этом видно, что между противоречащими понятиями третье, среднее, невозможно, так как они полностью исчерпывают объем родового понятия. Такое отношение существует, например, между понятиями «белый» и «небелый». Они исключают друг друга. Нельзя об одном и том же предмете, взятом в одно и то же время и в одном и том же отношении, сказать, что он и белый и небе лый.

9.Посредством Эйлеровых кругов изображаются также отношения между объемами субъекта и предиката в суждениях. Так, в общеутвердительном суждении, выражающем определение какого-либо понятия, объемы субъекта и предиката, как известно, равны. Наглядно такое отношение между объемами субъекта и предиката изображается посредством одного круга, подобно изображению отношений между объемами равнозначащих понятий. Разница только в том, что в данном случае всегда на поверхности круга надписываются две определенные буквы: S (субъект) и P (предикат), как это показано на рисунке:

Задача 1. Домашние любимцы. У всех моих подруг есть домашние питомцы. Шестеро из них любят и держат кошек, а пятеро — собак. И только у двоих есть и те и другте. Угадайте, сколько у меня подруг? Решение: Изобразим два круга, так как у нас два вида питомцев. В одном будем фиксировать владелиц кошек, в другом — собак. Поскольку у некоторых подруг есть и те, и другие животные, то круги нарисуем так, чтобы у них была общая часть. В этой общей части ставим цифру 2 так как кошки и собаки есть у двоих. В оставшейся части «кошачьего» круга ставим цифру 4 (6 — 2 = 4). В свободной части «собачьего» круга ставим цифру 3 (5 — 2 = 3). А теперь рисунок сам подсказывает, что всего у меня 4 + 2 + 3 = 9 подруг.

Ответ. 9 подруг.

Задача 2. Библиотеки. В классе 30 учеников. Все они являются читателями школьной и районной библиотек. Из них 20 ребят берут книги в школьной библиотеке, 15 — в районной. Сколько учеников не являются читателями школьной библиотеки? Решение: Пусть круг Ш изображает читателей только школьной библиотеки, круг Р — только районной. Тогда ШР — изображение читателей и районной, и школьной библиотек одновременно. Из рисунка следует, что число учеников, не являющихся читателями школьной библиотеки, равно: (не Ш) = Р — ШР. Всего 30 учеников, Ш = 20 человек, Р = 15 человек. Тогда значение ШР может быть найдено так (см. рисунок): ШР = (Ш + Р) — 30 = (20 + 15) — 30 = = 5, т.е. 5 учеников являются читателями школьной и районной библиотек одновременно. Тогда (не Ш) = = Р — ШР= 15 — 5= 10.

Ответ: 10 учеников не являются читателями школьной библиотеки.

Задача 3. Любимые мультфильмы. Среди школьников пятого класса проводилось анкетирование по любимым мультфильмам. Самыми популярными оказались три мультфильма: «Белоснежка и семь гномов», » Винни Пух», «Микки Маус». Всего в классе 28 человек. «Белоснежку и семь гномов» выбрали 16 учеников, среди которых трое назвали еще «Микки Маус», шестеро — » Винни Пух», а один написал все три мультфильма. Мультфильм «Микки Маус» назвали 9 ребят, среди которых пятеро выбрали по два мультфильма. Сколько человек выбрали мультфильм » Винни Пух»? Решение: В этой задаче 3 множества, из условий задачи видно, что все они пересекаются между собой. Только «Белоснежку» выбрали 16-6-3-1=6 человек. Только «Микки-Маус» выбрали 9-3-2-1=3 человека. Только » Винни-Пух » выбрали 28-(6+3+3+2+6+1)=7 человек. Тогда, учитывая, что некоторые выбрали по несколько мультфильмов, получаем, что » Винни-Пух » выбрали 7+6+1+2=16 человек.

Задача 7. Спорт для всех. В классе 38 человек. Из них 16 играют в баскетбол, 17 — в хоккей, 18 — в футбол. Увлекаются двумя видами спорта — баскетболом и хоккеем — четверо, баскетболом и футболом — трое, футболом и хоккеем — пятеро. Трое не увлекаются ни баскетболом, ни хоккеем, ни футболом. Сколько ребят увлекаются одновременно тремя видами спорта? Сколько ребят увлекается лишь одним из этих видов спорта? Решение. Воспользуемся кругами Эйлера. Пусть большой круг изображает всех учащихся класса, а три меньших круга Б, Х и Ф изображают соответственно баскетболистов, хоккеистов и футболистов. Тогда фигура Z, общая часть кругов Б, Х и Ф, изображает ребят, увлекающихся тремя видами спорта. Из рассмотрения кругов Эйлера видно, что одним лишь видом спорта — баскетболом занимаются 16 — (4 + z + 3) = 9 — z ; одним лишь хоккеем 17 — (4 + z + 5) = 8 — z ; одним лишь футболом

18 — (3 + z + 5) = 10 — z . Составляем уравнение, пользуясь тем, что класс разбился на отдельные группы ребят; количества ребят в каждой группе обведены на рисунке рамочкам: 3 + (9 — z) + (8 — z) + (10 — z) + 4 + 3 + 5 + z = 38,z = 2. Таким образом, двое ребят увлекаются всеми тремя видами спорта. Складывая числа 9 — z , 8 — z и 10 — z , где z = 2, найдем количество ребят, увлекающихся лишь одним видом спорта: 21 человек. Ответ: Двое ребят увлекаются всеми тремя видами спорта человека. Увлекающихся лишь одним видом спорта: 21 человек.

Задача Спортивный класс. В классе 35 учеников. 24 из них играют в футбол, 18 — в волейбол, 12 — в баскетбол. 10 учеников одновременно играют в футбол и волейбол, 8 — в футбол и баскетбол, а 5 — в волейбол и баскетбол. Сколько учеников играют и в футбол, и в волейбол, и в баскетбол одновременно? Домашнее задание

1 слайд

2 слайд

3 слайд

Один из величайших математиков петербургский академик, за свою долгую жизнь он написал более 850 научных работ. В одной из них появились эти круги. Эйлер писал, что «они очень подходят для того, чтобы облегчит наши размышления». Леонардо Эйлер 1707-1783

4 слайд

Задача №1 В классе 35 учеников. Из них 20 занимаются в математическом кружке, 11 – в биологическом, 10 ребят не посещают эти кружки. Сколько биологов увлекаются математикой?

5 слайд

Решение (По рисунку) в левом кругу (М) помещены все математики, а в правом – все биологи, те ребята, которые не ходят на кружки и помещены они в самый большой круг. Теперь посчитаем: Внутри большого круга 35 ребят. Внутри 2-х меньших 35-10=25 ребят. Внутри М находятся 20 ребят. Внутри Б находятся 25-20=5 биологов (не посещающих математический кружок) Внутри МБ находятся 11-5=6 биологов увлекающиеся математикой. 2. А В С 5-1-0,5-1=2,5 4-1-0,5-1=1,5 3-1-0,5-1=0,5 1 1 АВС 0,5

10 слайд

Задача №4 В классе 38 человек. Из них 16 играют в баскетбол, 17 – в хоккей, 18 – в волейбол. Увлекаются двумя видами спорта — баскетболом и хоккеем – четверо, баскетболом и волейболом – трое, волейболом и хоккеем – пятеро. Трое не увлекаются ни баскетболом, ни волейболом, ни хоккеем. Сколько ребят увлекается одновременно тремя видами спорта? Сколько ребят увлекается лишь одним из этих видов спорта?

Слайд 2

Круги Эйлера

  • Слайд 3

    Один из величайших математиков петербургский академик, за свою долгую жизнь он написал более 850 научных работ. В одной из них появились эти круги. Эйлер писал, что «они очень подходят для того, чтобы облегчит наши размышления». Леонардо Эйлер 1707-1783

    Слайд 4

    Задача №1

    В классе 35 учеников. Из них 20 занимаются в математическом кружке, 11 – в биологическом, 10 ребят не посещают эти кружки. Сколько биологов увлекаются математикой?

    Слайд 5

    Решение

    (По рисунку) в левом кругу (М) помещены все математики, а в правом – все биологи, те ребята, которые не ходят на кружки и помещены они в самый большой круг. Теперь посчитаем: Внутри большого круга 35 ребят. Внутри 2-х меньших 35-10=25 ребят. Внутри М находятся 20 ребят. Внутри Б находятся 25-20=5 биологов (не посещающих математический кружок) Внутри МБ находятся 11-5=6 биологовувлекающиеся математикой. М Б МБ

    Слайд 6

    Задача №2

    В пионерском лагере 70 ребят. Из них 27 занимаются в драмкружке, 32 поют в хоре, 22 увлекаются спортом. В драмкружке 10 ребят из хора, в хоре 10 спортсменов; 3 спортсмена посещают и драмкружок, и хор. Сколько ребят не поют в хоре, не увлекаются спортом и не занимаются в драмкружке? Сколько ребят заняты только спортом?

    Слайд 7

    Решение

    (По рисунку) Д – драмкружок, Х – хор, С – спортсмены. 5+3+3=11спортсменов посещают хор и драмкружок тогда 22-11=11 увлекаются только спортом 70-12-7-19-5-3-3-11=10 ребят не поют в хоре, не увлекаются спортом и не занимаются в драмкружке. Х С Д ДХС 12 19 10-3=7 3 8-3=5 6-3=3 22-5-3-3=11

    Слайд 8

    Задача №3

    Пол комнаты площадью 12 м^2 покрыт тремя коврами: площадь одного ковра 5 м^2, другого – 4м^2 и третьего – 3 м^2. 2. А В С 5-1-0,5-1=2,5 4-1-0,5-1=1,5 3-1-0,5-1=0,5 1 1 АВС 0,5

    Слайд 10

    Задача №4

    В классе 38 человек. Из них 16 играют в баскетбол, 17 – в хоккей, 18 – в волейбол. Увлекаются двумя видами спорта — баскетболом и хоккеем – четверо, баскетболом и волейболом – трое, волейболом и хоккеем – пятеро. Трое не увлекаются ни баскетболом, ни волейболом, ни хоккеем. Сколько ребят увлекается одновременно тремя видами спорта? Сколько ребят увлекается лишь одним из этих видов спорта?

    Слайд 11

    Решение

    Большой круг изображает всех учащихся класса, а три меньших круга Б, Х и В изображают соответственно баскетболистов, хоккеистов и волейболистов. Фигура БХВ обозначает ребят, увлекающихся всеми тремя видами спорта – z. Одним баскетболом занимаются 16-(4+z+3)=9-z, хоккеем 8-z, волейболом 10-z. Составляем уравнение: 38=3+(9-z)+(8-z)+(10-z)+4+3+5+z, откуда z=2 3 Б Х В 9-Z 8-Z 4 Z 3 5 10-Z

    Слайд 12

    П.А.Вакульчик «Нестандартные и олимпиадные задачи по математике» В. А.Гусев. А.Н.Орлов. А П. Розенталь «Внеклассная работа по математике» И.Л. Бабинская «Задачи математических олимпиад» А.В.Фарков «Готовимся к олимпиадам по математике» И.С.Петраков «Математические кружки» Литература: http://poznayko.at.ua/photo/16-2-0-0-2 http://www.math-on-line.com.forum-tur http://images.yandex.ru/yandsearch?text

    Посмотреть все слайды

    Вычислите значение выражения. Ничего не сказала рыбка, лишь хвостом по воде плеснула и ушла в глубокое море. Сказка о мертвой царевне и семи богатырях. Из какой сказки этот отрывок. Сказка о золотом петушке. Сказка о царе Салтане. К 213–летию со дня рождения А.С.Пушкина. Выполните действия, результаты найдите в таблице и отгадайте зашифрованные слова. Найдите значение выражения. Устная работа. Ответы уравнений в конкурсе «Рыбалка».

    «Координаты точек на координатной плоскости» — Карл Гаусс. Тренажер. Выбери нужную ячейку таблицы. Абсцисса точки. Учебники. Координатная плоскость. Координаты точки. Николай Иванович Лобачевский. Объяснение нового материала. Цвет. Леонард Эйлер. Исаак Ньютон. Четверть. Готфрид Лейбниц. Координата. Курсор. Координатная четверть. Точка лежит на оси Х. Великие математики. Тест. Технические рекомендации. Отметьте точку. Рене Декарт. Блез Паскаль.

    «Приемы устного умножения» — Умножение и деление на 25 и 75. Значимость устных приёмов умножения. Умножение чисел, оканчивающихся на 1. Устные приёмы умножения двузначных натуральных чисел. Умножение чисел, близких к 100. Умножение двузначных чисел, у которых цифры десятков одинаковые. Умножение на 11. Умножение двузначных чисел, у которых сумма цифр десятков равна. Умножение чисел, оканчивающихся на 5. Умножение на число, оканчивающиеся на 5.

    «Дни недели» — Воскресенье — день Солнца (старое название – неделя). Понедельник. Воскресенье -день Солнца. У славян неделя называлась седмица. Суббота – день Сатурна. Гипотеза. Среда. Названия дней недели в русском и английском языках. Загадка. Библия говорит, что семидневная неделя создана Богом. Как появились 7 дней недели. Среда день Меркурия (среда – середина). Неделя днями красна. Конёк – Горбунок. Четверг – день Юпитера (четвёртый).

    «Путешествие в мир математики» — Остров « Умейка». Решить с помощью координатной прямой. Измерим температуру за бортом корабля. Прибавить к числу А число В — значит изменить число А на число В. Сумма двух противоположных чисел равна нулю. Остров « Повторика». Найдём сумму чисел. Результаты двух последовательных изменений находят с помощью сложения. Найдём сумму чисел. Показания приборов на корабле. Любое число от прибавления положительного числа увеличивается.

    «Десятичная система и двоичная» — Перевод целых десятичных чисел в двоичную систему счисления. Перевод целых чисел из двоичной системы счисления в десятичную. Ей было 1100 лет, она в 101 класс ходила. Калькулятор. Какую цель перед собой мы ставили в начале первого урока. Переведите числа из двоичной системы счисления в десятичную. «Рождение» цветка. Необычное стихотворение. Закрасьте клеточки. Разделить целое десятичное число на 2.


    «Письма о разных физических и философических материях, написанные к некоторой немецкой принцессе…», где появились впервые «круги Эйлера» «Письма о разных физических и философических материях, написанные к некоторой немецкой принцессе…», где появились впервые «круги Эйлера»




    Решение задач с помощью кругов Эйлера. Часть жителей нашего города умеет говорить только по-русски, часть – только по- башкирски и часть умеет говорить на обоих языках. По- башкирски говорят 85%, по-русски 75%. Сколько процентов жителей говорят на обоих языках?



    Спортивная задача В футбольной команде «Баймак» 30 игроков: 18 нападающих. 11 полузащитников, 17 защитников Вратари 3 могут быть нападающими и защитниками, 10 защитниками и полузащитниками, 6 нападающими и защитниками 1 и нападающим, и защитником, и полузащитником. Вратари не заменимы. Сколько в команде «Баймак» вратарей?

    Решение =28 (игроков) на этой диаграмме. Но в команде всего 30 футболистов. Значит вратарей будет 30-28=2. Ответ: 2 вратаря.

    «Озеро Графское» Из 100 отдыхающих на турбазе «Графское», 30 детей — отличники учебы, 28 — участники олимпиад, 42 — спортсмены. 8 учащихся одновременно участники олимпиад и спортсмены, 10 – участники олимпиад и отличники, 5 – спортсмены и отличники учебы, 3 – и отличники, и участники олимпиад, и спортсмены. Сколько отдыхающих не относятся ни к одной из групп?
    Выводы Применение кругов Эйлера (диаграмм Эйлера-Венна) позволяет легко решить задачи, которые обычным путем разрешимы лишь при составлении системы трех уравнений с тремя неизвестными. Применение кругов Эйлера (диаграмм Эйлера-Венна) позволяет легко решить задачи, которые обычным путем разрешимы лишь при составлении системы трех уравнений с тремя неизвестными.

  • Обзор диаграмм Эйлера

    Обзор диаграмм Эйлера

    Диаграммы Эйлера 2004

    Брайтон, Великобритания

    сентября 22-23
     

     

    Главная | Требуют документов | Материалы | Регистрация   | Программа   | Местный аранжировки   | Диаграммы Эйлера   | Брайтон

    Что такое диаграммы Эйлера?

    Леонард Эйлер (произносится «Ойлер») был одним из величайших математиков всех времен. время. Многие утверждают, что он был величайшим. Одно из его менее известных изобретений это диаграммы Эйлера, которые он использовал для иллюстрации рассуждений.

    Выше показана диаграмма Эйлера. Одна из распространенных интерпретаций Эйлера диаграммы — это пересечение множеств. При такой интерпретации вышеизложенное на диаграмме используются области для представления наборов A, B и C. На диаграмме также есть площади для пересечений АВ, АС и АВС. Никакая область не представляет набор (не A)C, поэтому множество C целиком содержится в A.

    Визуально диаграммы Эйлера состоят из контуров , нарисованных как простые замкнутые кривые. Контуры разбивают плоскость на зоны . Зона можно определить по содержащимся в нем контурам. На схеме выше контуры обозначены A, B и C, а зоны A, B, AB, AC и ABC присутствуют в диаграмме (а также внешней зоне, которая не содержится ни в каких контурах). Мы тут связать с каждой зоной метку, образованную из контуров, внутри которых она находится. содержится.

    Диаграммы Венна, диаграммы Эйлера и Лейбница

    Часто путают термины диаграмма Эйлера и диаграмма Венна. Диаграммы Венна можно рассматривать как частный случай диаграмм Эйлера, поскольку диаграммы Венна должны содержать все возможные зоны, тогда как диаграммы Эйлера могут содержать подмножество всех возможных зон. На диаграммах Венна заштрихованная зона представляет собой пустое множество, тогда как на диаграмме Эйлера соответствующая зона может отсутствовать на схеме. Этот означает, что по мере увеличения числа контуров диаграммы Эйлера обычно становятся менее визуально сложнее, чем эквивалентная диаграмма Венна, особенно если количество непустой перекрестков мало.

    Барон [Bar69] отмечает, что Лейбниц производил подобные диаграммы до Эйлера, однако большая часть их не публиковалась. Она также наблюдает даже более ранние диаграммы типа Эйлера, созданные Рамоном Луллием в 13 веке.

    Заметки о конкурирующих типах диаграмм можно найти по адресу: Wikipedia, Interactive Математический сборник и головоломки и Венн Фрэнка Раски Диаграмма Обзор.

    Области применения

    В этом разделе показано несколько примеров использования диаграмм Эйлера. Часто, Диаграммы Эйлера дополняются дополнительными структурами, такими как точки, метки или графики, показывающие информацию о том, что содержится в различных зонах.

    Одной из важных особенностей диаграмм Эйлера является их способность визуализировать сложные иерархии. Выше приведено изображение, показывающее, что некоторые животные относятся более чем к одной классификации, например, «собака» и «кошка», которые оба являются домашними животными. и млекопитающие. Нелегко показать такого рода отношения с более обычными иерархическая визуализация классификаций на основе дерева. ВЕННФС [CES03] использует этот подход диаграммы Эйлера для визуализации организации файловой системы. Это позволяет файлам появляться более чем в одном каталоге файловой системы компьютера. [ВВ04] предложить использовать диаграммы Эйлера для визуализации больших баз данных с использованием нескольких классификации.

    Первоначальное применение диаграмм Эйлера как способа схематически демонстрации логики, широко используются в школах, где они являются большим подспорьем для преподавание теории множеств. Другие академические работы включают Hammer [Ham95], который представил надежную и полную логическую систему, основанную на диаграммах Эйлера. Более выразительные рассуждения могут быть достигнуты путем расширения диаграмм с помощью графиков. Шин [Ши94] разработал первую такую ​​формальную систему. Это было распространено на Spider [HMTKG01]. и диаграммы ограничений [GHK01] Группой визуального моделирования в Брайтонском университете вместе с другими. Пример диаграммы ограничений показан выше. Эти расширенные диаграммы Эйлера можно рассматривать как гиперграфы, и поэтому должна быть возможность применять методы визуализации для расширенного Эйлера диаграммы в более общем смысле для приложений, использующих гиперграфы.

    Генерация диаграмм Эйлера

    Большая часть недавних исследований была посвящена встраиванию Эйлера диаграммы в плоскости из текстового описания зон, которые должны появиться в диаграмма. Эта работа сделана более интересной благодаря наличию правильной формы условия. Корректность ограничивает появление диаграмм Эйлера, поэтому в какой-то степени, чем лучше построено, тем лучше понимание диаграммы. Однако некоторые диаграммы Эйлера невозможно нарисовать при некоторой корректности. условия. Общие условия правильности:

    1. Форма контуров может быть ограничена определенными формами, такими как круглая, овальная, прямоугольная или выпуклая формы.
    2. Тройные точки не допускаются, так что только два контура могут пересекаться в любой заданной точке.
    3. Допускаются только поперечные пересечения контуров, чтобы линии не могли соприкасаться без пересечения.
    4. Параллельные контуры не могут быть разрешены, поэтому сегмент линии не может представлять границу двух или более контуров.
    5. Нельзя допускать отключенные зоны, поэтому зоны не могут появляться на диаграмме более одного раза
    6. Контуры должны быть простыми кривыми, чтобы контуры, пересекающиеся сами с собой, не допускаются

    Снятие этих ограничений позволяет рисовать все диаграммы Эйлера. Сам Эйлер рисовал диаграммы только кругами, не нарушая ни одного из правил. условия правильности.

    Некоторые примеры правильной формы:

    1. Это диаграмма Венна 4 (диаграмма Венна с 4 контуры), нарисованные с невыпуклыми контурами. Эту схему можно нарисовать с помощью выпуклые формы.

     

    2. Эта диаграмма, представляющая зоны A, B, C, AB, AC и BC (но не ABC) получил тройную точку. Эта схема не может быть построена без нарушение правил правильной формы 2, 5 или 6.

     

    3. Это пример непоперечной диаграммы представляющие зоны A и B.
    4. Это схема, представляющая зоны AB, AC. для визуализации требуются общие сегменты кривой.

     

    5. На этих диаграммах области одного цвета представляют такая же зона. На схеме слева есть отключенные зоны. Диаграмма справа имеет ту же семантику, что и слева, но имеет правильный формат. Отключенные зоны особенно неприятны там, где предметы предназначены быть сгруппированы по зонам. Если использовались диаграммы, подобные приведенным слева, тогда два элемента, которые должны были быть сгруппированы вместе, могли появиться в разных участках диаграммы, несмотря на то, что семантика схема будет правильной.
    6. Это схема зон A и B, обозначенных непростые кривые.

     

    Первая работа по автоматическому созданию диаграмм Эйлера была выполнена Флауэром и Хоуз [FH02], кто предложил и реализовал метод рисования подмножества диаграмм при соблюдении всех условий корректности, кроме 1, внизу слева диаграмма, построенная этим методом. Дальнейшая работа Чоу и Раски [CR03], описал реализованный метод встраивания небольших диаграмм с ограничениями на форму контуров, включая фигуры, построенные из прямоугольников. Верроуст и Вио [VV04] предложили систему рисования всех диаграмм Эйлера до 8 контуров, расслабляющую правила правильности 1,2,3 и 4.

    Компоновка диаграмм Эйлера

    Это относится к рисованию диаграмм Эйлера с учетом их эстетичный внешний вид. Методы генерации, описанные в предыдущем разделе часто создают трудные для понимания диаграммы Эйлера. Наше недавнее исследование [FRM03] взял диаграммы Эйлера, сгенерированные [FH02] и применил эстетические метрики к контурам, чтобы сделать диаграммы более понятно, смотрите выше фото до и после. Это было тогда расширен для рисования графиков в диаграммах Эйлера [MRF04].

    Контакт

    Если вы заметили ошибку или хотите что-то добавить на эту страницу, свяжитесь с: Питером Роджерсом.

    Ссылки

    Стэнфорд В энциклопедии философии есть подробное резюме и сравнение диаграмм. типы
    Примеры использование диаграмм Эйлера в логических рассуждениях
    Статья в Википедии о Венне и Эйлере диаграммы
    Венн против Эйлера
    Диаграмма Венна Опрос Фрэнка Раски

    Ссылки

    [Bar69] М. Э. Барон. Примечание о Историческое развитие логических диаграмм. Математическая газета: Журнал математической ассоциации. Том LIII, нет. 383, май 1969 г.
    [CES03] Р. Де Кьяра, У Эрра и В. Скарано. VENNFS: файловый менеджер с диаграммами Венна. проц. Визуализация информации IEEE (IV03). стр. 120-126. 2003.
    [CR03] С. Чоу и Ф. Раски. Диаграммы Венна и Эйлера, пропорциональные площади. проц. ГД2003. LNCS 2912. Спрингер Верлаг.
    [Eul61] Л. Эйлер. Lettres a Une Princesse d’Allemagne, том 2. 1761 г. Письма No. 102–108.
    [FH02] Дж. Флауэр и Дж. Хоуз. Генерация Диаграммы Эйлера, Proc. Диаграммы 2002 г., Springer Verlag, 61–75.
    [FRM03] Дж. Флауэр, П. Роджерс и П. Маттон. Метрики макета для диаграмм Эйлера. проц. Визуализация информации IEEE (IV03). стр. 272-280. 2003.
    [GHK01] Дж. Гил, Дж. Хаус и С. Кент. На пути к формализации диаграмм ограничений, Труды Human-Centric Компьютеры (HCC 2001) Стреза, Италия, IEEE Computer Society Press, 72–79. . 2001.
    [Ham95] EM Hammer. Логика и визуальное Информация, публикации CSLI. 1995.
    [HMTKG01] Дж. Хауз, Ф. Молина, Дж. Тейлор, С. Кент и Дж. Гил. Диаграммы пауков: A Схематическая система рассуждений, Журнал визуальных языков и вычислений 12, 299-324. 2001
    [MRF04] П. Маттон, П. Роджерс и Дж. Цветок. Рисование графиков в диаграммах Эйлера. проц. Диаграммы 2004. ЛНАИ 2980. Спрингер Верлаг. 66-81.
    [Shi94] S-J Шин. Логический статус диаграмм. ЧАШКА. 1994.
    [Ven80] Дж. Венн, О диаграммном и механическом представлении предложений и рассуждения, The London, Edinburgh, and Dublin Philosophical Magazine and Journal наук, 9 (1880) 1-18.
    [VV04] А. Верроуст и М.-Л. Вио. Обеспечение возможности рисования расширенных диаграмм Эйлера до 8 Наборы. проц. Диаграммы 2004. LNAI 2980. Springer Verlag. 128-141.

    Диаграмма Эйлера — определение, шаблоны, инструмент

    Объяснение сложных иерархий и перекрывающихся определений

    Что такое диаграмма Эйлера

    Диаграмма Эйлера представляет собой графический инструмент, представляющий отношения субъектов на этом графике. Диаграммы Эйлера часто используются в сфере образования и бизнеса.

    По сравнению с диаграммами Венна диаграмма Эйлера имеет только соответствующие связи между темами. Например, все живые существа, имеющие четыре ноги, являются животными, но не у всех животных четыре ноги, например у рыб. У него только один хвост, но он принадлежит к семейству животных. С этой точки зрения ясно видна связь между животными и четырьмя ногами. В математике мы называем это отношением включения.

    Четыре ноги являются подмножеством животных. Однако минерал не имеет отношения к животным, потому что сходства нет; другой синий круг представляет минерал. И между кругом животного и минерального нет перекрывающихся частей.

    Источник изображения: en.wikipedia.org

    Другой пример диаграммы Эйлера — отношения между различными объектами Солнечной системы.

    Источник изображения: en.wikipedia.org

    Во-первых, плоскость разделяется кривыми диаграммы Эйлера на две части; внутренняя зона представляет элементы наборов, а внешняя показывает все элементы, кроме наборов планет. Кроме того, кривые перекрытия означают только отношение включения. Если кривые не пересекаются и не пересекаются, элементы не являются членами множеств.

    Общие элементы для обоих наборов только в зоне перекрытия внутри обеих кривых, а подмножество одного набора окружено кривыми внутри другого набора. Из этой картинки ясно видно, насколько отличается диаграмма Эйлера и как прояснить сложные предметы.

    Поэтому диаграммы Эйлера состоят из нескольких четких фигур или кругов, демонстрирующих положительные отношения между этими наборами.

    Диаграммы Венна и диаграммы Эйлера

    Большинство людей путают диаграммы Венна и диаграммы Эйлера. «Эйлеровы круги», придуманные швейцарским математиком Леонардом Эйлером (1707–1783), включены вместе с диаграммами Венна в качестве средств обучения теории множеств.

    Диаграммы Венна, более творческая форма диаграмм Эйлера, состоят из всех 2n логически возможных зон перекрытия между его n кривыми и сосредоточены на включении/исключении всех взаимосвязей составляющих его наборов. Черным цветом обозначены области исключения, что означает, что области, исключающие набор, будут черными.

    Однако диаграммы Эйлера должны быть окрашены только перекрытием, что означает, что области, включающие только набор, будут окрашены. Таким образом, наиболее существенная разница заключается в перекрывающихся частях отношений. Обычно отношения можно разделить на две части: релевантность и нерелевантность. Диаграмма Венна покажет как релевантность, так и нерелевантность между субъектами, но на диаграммах Эйлера будет существовать либо релевантность, либо нерелевантность.

    Взяв в качестве примера млекопитающих и птиц, птиц нельзя разделить на млекопитающих, потому что птицы являются видами, откладывающими яйца, а не живородящими, поэтому птицы не имеют отношения к млекопитающим. На диаграмме Венна круги млекопитающих и птиц репрезентативно должны пересекаться, а их круги на диаграмме Эйлера не должны пересекаться. Поэтому диаграммы Венна и Эйлера совершенно по-другому показывают нерелевантность субъектов.

    Источник изображения: creately.com

    Шаблоны диаграмм Эйлера

    1. Этот шаблон показывает отношения стран на Британских островах.

    Источник изображения: pinterest.ca

    2: Шаблон показывает соотношение цветов в карточках.

    Источник изображения: /creately.com/

    3: Этот шаблон демонстрирует отношения между магией, волшебниками и ящерицами. Магия включает волшебников, но не имеет отношения к ящерицам.

    Источник изображения: statshowto.com

    4: Эта диаграмма Эйлера демонстрирует географические отношения региона Океании, который делится на 4 субрегиона: Меланезию, Микронезию, Полинезию и Австралазию.

    Источник изображения: reddit.com

    Универсальное программное обеспечение для построения диаграмм

    Легко создавайте более 280 типов диаграмм

    Легко начинайте строить диаграммы с помощью различных шаблонов и символов

    • Превосходная совместимость файлов: Импорт и экспорт чертежей в файлы различных форматов, например Visio
    • Кроссплатформенная поддержка (Windows, Mac, Linux, Интернет)

    Скачать бесплатно

    Безопасность проверена | Переключиться на Mac >>

    ПОПРОБУЙТЕ БЕСПЛАТНО

    Безопасность подтверждена | Перейти на Linux >>

    ПОПРОБУЙТЕ БЕСПЛАТНО

    Безопасность подтверждена | Переключиться на Windows >>

    Как построить диаграмму Эйлера

    Любые графические инструменты и программное обеспечение, которые вы все можете использовать, но если вам нужно построить сложные диаграммы Эйлера, EdrawMax может быть одним из ваших вариантов.

    Шаг 1 : Подумайте о своей цели и использовании ваших диаграмм Эйлера.

    • Сколько острых предметов или кругов вам нужно?
    • С какими отношениями взаимодействуют ваши темы?

    Шаг 2 : Чтобы найти нужный формат и острые предметы и насекомое.

    Шаг 3 : Быстро сохраните и загрузите желаемое видение.

    Если вы собираетесь создавать сложные диаграммы Эйлера, это универсальный инструмент для создания диаграмм с индивидуальным дизайном и обширными предложениями шаблонов.

    Советы по созданию диаграммы Эйлера

    Существуют некоторые ограничения для построения диаграмм Эйлера:

    1: Первая — это диаграмма Венна с 4 контурами, а не стандартная диаграмма Эйлера, а вторые две. Разница связана с эстетическим видом. Второй с кругами мог бы сделать диаграммы более понятными и сравнить элементы в каждом наборе. Кроме того, изменение более эффективно для людей, чтобы понять информацию, представленную создателями диаграмм.

    Источник изображения: cs.kent.ac.uk

    Первая — это диаграмма Венна с 4 контурами, не стандартная диаграмма Эйлера, а вторые две. Разница связана с эстетическим видом.

    Второй с кругами мог бы сделать диаграммы более понятными и сравнить элементы в каждом наборе. Кроме того, изменение более эффективно для людей, чтобы понять информацию, представленную создателями диаграмм.

    2: В данной точке пересекаются только два контура из-за нецелесообразности тройных точек. Как на картинке, которую вы видели, это не разрешено рисовать. Если вы это сделаете, это будут диаграммы Венна, а не диаграммы Эйлера. Пожалуйста, уделите больше времени примерам и шаблонам диаграмм Эйлера.

    Источник изображения: cs.kent.ac.uk

    Эта диаграмма также не является стандартной диаграммой Эйлера, потому что зоны A, B, C представлены и пересекаются тройной точкой. Это нарушение правил 2 и 4.

    3: Поскольку граница контуров не может быть продемонстрирована сегментами линии, она не может быть Параллельными контурами.

    Источник изображения: cs.kent.ac.uk

    На этой схеме показаны зоны AB и AC, причем две зоны принадлежат одному набору. Однако зоны АВ и АС не имеют общих элементов.

    4: На диаграммах Эйлера несвязанные зоны не должны появляться дважды или более.

    5: Не допускается пересечение контуров на диаграммах Эйлера, если контуры диаграмм Эйлера являются простыми кривыми.

    Вывод

    Ключом к освоению диаграмм Эйлера является понимание разницы между диаграммами Венна и Эйлера. Процесс создания не очень сложен для людей, поэтому, прежде чем рисовать диаграммы Эйлера, вы можете найти больше шаблонов и примеров, чтобы пройтись по ним и подумать, как представить свою идею на этой диаграмме на основе других творческих диаграмм Эйлера. Практика делает совершенным.

    17.9: Оценка дедуктивных аргументов с помощью диаграмм Эйлера

    1. Последнее обновление
    2. Сохранить как PDF
  • Идентификатор страницы
    41408
    • Дэвид Липпман
    • Pierce College через OpenTextBookStore

    Мы можем визуально интерпретировать дедуктивный аргумент с помощью диаграммы Эйлера, которая по сути является тем же самым, что и диаграмма Венна. Это может облегчить определение того, является ли аргумент действительным или недействительным.

    Пример 31

    Рассмотрим дедуктивный аргумент «Все кошки — млекопитающие, а тигр — кошка, поэтому тигр — млекопитающее». Является ли этот аргумент действительным?

    Решение

    Помещения:

    Все кошки млекопитающие.

    Тигр — это кошка.

    Вывод:

    Тигр — млекопитающее.

    Обе посылки верны. Чтобы увидеть, что посылки должны логически вести к заключению, мы можем использовать диаграмму Венна. Исходя из первой посылки, мы рисуем множество кошек как подмножество множества млекопитающих. Из второй посылки нам говорят, что тигр содержится в множестве кошек. Из этого мы видим на диаграмме Венна, что тигр также должен быть внутри множества млекопитающих, так что вывод верен.

    Анализ аргументов с помощью диаграмм Эйлера

    Чтобы проанализировать аргумент с помощью диаграммы Эйлера:

    1) Нарисуйте диаграмму Эйлера на основе предпосылок аргумента

    2) Аргумент недействителен, если есть способ нарисовать диаграмму что делает вывод ложным

    3) Аргумент действителен, если нельзя построить диаграмму, чтобы сделать вывод ложным

    4) Если посылок недостаточно для определения местоположения элемента или множества, упомянутых в заключении, то аргумент неверный.

    Попробуйте сейчас 9

    Определите правильность этого аргумента:

    \(\begin{array} {ll} \text{Предпосылка:} & \text{Все кошки боятся пылесосов.} \\ \text {Предпосылка:} & \text{Макс — кошка.} \\ \text{Вывод:} & \text{Макс боится пылесосов.} \end{массив}\)

    Ответ

    Действителен. Кошки — это подмножество существ, которых пугают пылесосы. Макс входит в набор котов, значит, он тоже должен быть в наборе существ, которых пугают пылесосы.

    Пример 32

    \(\begin{array} {ll} \text{Предпосылка:} & \text{Все пожарные знают СЛР.} \\ \text{Предпосылка:} & \text{Джилл знает СЛР.} \ \ \text{Вывод:} & \text{Джилл — пожарный.} \end{array}\)

    Решение

    Из первой посылки мы знаем, что все пожарные находятся внутри набора тех, кто знает СЛР. (Пожарные — это подмножество людей, знающих СЛР.) Из второй посылки мы знаем, что Джилл является членом этой большей группы, но у нас недостаточно информации, чтобы узнать, является ли она также членом меньшей подгруппы, т. е. пожарные.

    Поскольку заключение не обязательно следует из посылок, это неверный аргумент. Возможно, что Джилл — пожарный, но структура аргумента не позволяет сделать вывод, что это определенно так.

    Важно отметить, что действительно ли Джилл пожарный или нет, не имеет значения для оценки обоснованности аргумента; нас интересует, достаточно ли посылок для доказательства вывода.

    Попробуйте сейчас 10

    Определите правильность этого аргумента:

    \(\begin{array} {ll} \text{Предпосылка:} & \text{Все велосипеды имеют два колеса.} \\ \text{Предпосылка:} & \text{Это У Harley-Davidson два колеса.} \\ \text{Вывод:} & \text{Этот Harley-Davidson — велосипед.} \end{array}\)

    Ответ

    Недействительно. Множество велосипедов является подмножеством множества транспортных средств с двумя колесами; Harley-Davidson входит в число двухколесных транспортных средств, но не обязательно в меньший круг.

    Попробуйте сейчас 11

    Определите правильность этого аргумента:

    \(\begin{array} {ll} \text{Предпосылка:} & \text{Нет фиолетовых коров. } \\ \text{Предпосылка: } & \text{Фидо не корова.} \\ \text{Вывод:} & \text{Фидо фиолетовый.} \end{массив}\)

    Ответ

    Недействительно. Поскольку фиолетовых коров нет, мы знаем, что набор коров и набор фиолетовых вещей не пересекаются. Мы знаем, что Фидо не находится в наборе коров, но этого недостаточно, чтобы сделать вывод, что Фидо находится в наборе фиолетовых вещей.

    Помимо этих категориальных стилевых посылок вида «все ___», «некоторые ____» и «нет ____», также часто встречаются условные посылки.

    Пример 33

    \(\begin{array} {ll} \text{Предпосылка:} & \text{Если вы живете в Сиэтле, вы живете в Вашингтоне.} \\ \text{Предпосылка:} & \text{Маркус не живет в Сиэтле.} \\ \text{Вывод:} & \text{Маркус не живет в Вашингтоне.} \end{array}\)

    Решение

    Из первой посылки мы знаем, что множество людей, живущих в Сиэтле, находится внутри множества тех, кто живет в Вашингтоне. Из второй посылки мы знаем, что Маркус не входит в набор Сиэтла, но у нас недостаточно информации, чтобы узнать, живет ли Маркус в Вашингтоне или нет. Это неверный аргумент.

    Попробуй сейчас 12

    Определите справедливость этого аргумента:

    \(\begin{array} {ll} \text{Предпосылка:} & \text{Если у вас есть помада на воротнике, то вы изменяешь мне.} \\ \text{Предпосылка:} & \text{Если ты мне изменяешь, я разведусь с тобой.} \\ \text{Предпосылка:} & \text{ У тебя нет помады на губах воротник.} \\ \text{Вывод:} & \text{Я с вами не разведусь.} \end{array}\)

    Ответить

    Недействительно. Помада на твоем воротнике — это подмножество сценариев, в которых ты изменяешь, а обман — это подмножество сценариев, в которых я разведусь с тобой. Хотя прекрасно, что у тебя на воротнике нет помады, ты все равно можешь мне изменять, и я разведусь с тобой. На самом деле, даже если ты мне не изменяешь, я могу развестись с тобой по другой причине. Тебе лучше привести себя в форму.


    Эта страница называется 17.9: Оценка дедуктивных аргументов с помощью диаграмм Эйлера распространяется под лицензией CC BY-SA 3. 0 и была создана, изменена и/или курирована Дэвидом Липпманом (OpenTextBookStore) через исходный контент, отредактированный в соответствии со стилем и стандартами платформы LibreTexts; подробная история редактирования доступна по запросу.

    1. Наверх
      • Была ли эта статья полезной?
      1. Тип изделия
        Раздел или страница
        Автор
        Дэвид Липпман
        Лицензия
        СС BY-SA
        Версия лицензии
        3,0
      2. Теги
        1. источник@http://www. opentextbookstore.com/mathinsociety

      Все о диаграмме Венна — Виззло

      Диаграмма Венна

      Узнайте все об определении и увлекательной истории диаграммы Венна!

      Чтение за 9 минут

      Диаграмма Венна Определение

      Диаграмма Венна (также известная как набор или логическая диаграмма) может показать различия, сходства и перекрывающиеся отношения между наборами, то есть группами данных. Чаще всего его используют для описания общих черт между противоборствующими фракциями — например, в приведенном ниже примере диаграммы Венна мы можем видеть общие характеристики клеток растений и животных.

      Исходные клетки растений и клетки животных — https://owlcation.com/stem/Plant-Cells-vs-Animal-Cells-With-Diagrams

      Набор данных визуализируется как совокупность фиксированных точек внутри замкнутой окружности (обычно это круги, но иногда эллипсы, сферы или даже треугольники): визуальное описание одной категории, общей для всех включенных данных. В приведенном выше примере окружность, содержащая характеристики клеток животных, имеет точки данных: неправильная/круглая, центросомы, центриоли, лизосомы, ядро, клеточная мембрана, цитоплазма, митохондрии, аппарат Гольджи и рибосомы. Только около из этих точек данных/характеристик являются общими для растительных клеток, визуализированных в синем пересечении.

      Нравится этот пример диаграммы Венна? Начните создавать свои собственные!

      Главной достопримечательностью диаграмм Венна является это пересечение — данные, которые попадают более чем в одну категорию (в приведенном выше примере это поясняется меткой «оба», хотя следует помнить, что пересечение — это ). не свой собственный, отдельный набор данных). Диаграммы Венна могут состоять из нескольких пересечений и наборов окружностей, но чаще всего используется 3-круговая или тройная диаграмма Венна.

      Особый фактор популярности тройки Венна заключается в том, что она создает в центре криволинейный треугольник идеальной и постоянной ширины, известный как «треугольник Рело». Эта форма очаровала инженеров, математиков, архитекторов и графических дизайнеров как своими математическими возможностями, так и визуальной красотой, и использовалась эрудитами от немецкого инженера Франца Рело (в честь которого они теперь названы) для создания кинетических машин до Леонардо Да. Винчи для картографической проекции.

      Начните создавать свои собственные диаграммы и бизнес-графику.

      Попробуйте Vizzlo бесплатно

      Развитие и история диаграммы Венна

      В той или иной форме диаграммы в стиле Венна существуют со времен Средневековья. Абстрактное визуальное представление соединений и пересечений различных наборов — невероятно старая концепция, поскольку существует глубокое человеческое стремление показать, что связано, а что нет. Как и в случае с большинством логических сред, с годами он органично рос и менялся, совершенствуясь, переориентируясь и перепрофилируясь в круговую диаграмму, с которой мы так знакомы сегодня.

      Первым математиком, непосредственно связанным с этим стилем диаграмм, был немецкий эрудит и философ Готфрид Вильгельм Лейбниц, живший примерно в конце 1600-х годов. По сути, Лейбниц был одним из первых, кто применил многие концепции визуализации данных: он верил в символическое представление сложных мыслей, чтобы не только лучше передавать эти идеи, но и оставлять меньше места для человеческих ошибок, неправильного толкования и вытекающих из этого споров. По мнению Лейбница, этого добились расчеты и визуализация различий во мнениях в формализованных логических диаграммах.

      Диаграмма Эйлера

      Примерно сто лет спустя известный швейцарский математик Леонард Эйлер стал широко известен тем, что использовал перекрывающиеся круги для представления логических выводов (или силлогистических рассуждений). Эти круги используются для изображения четырех категориальных утверждений силлогизма:

      1. Универсальное утверждение «Все графики Виззло выглядят красиво»
      2. Универсальное отрицательное «Ни один из графиков Виззло не сложен в производстве»
      3. Частное утвердительное «Некоторые из Виззардов Виззарда включают диаграммы Венна»
      4. Особый негатив «Некоторые из Виззардов Виззарда не являются диаграммами Венна»

      Примечание: Знаете ли вы, что в соответствии с логической семантикой в ​​английском языке до конца 16 века было четыре слова для «да» и «нет»?! Универсальное утвердительное было бы произнесено как «да», универсальное отрицательное — как «нет», частное утвердительное — как «да», а частное отрицательное — как «нет».

      Чтобы было ясно, стало обычным делом группировать диаграммы Эйлера и Венна под более известным названием «диаграмма Венна», чтобы избежать путаницы — только в очень технических математических ситуациях было бы необходимо проводить разграничение между два.

      Современная диаграмма Венна

      Известная нам сегодня диаграмма Венна была разработана английским математиком Джоном Венном. В 1880 г. он опубликовал статью под названием «О диаграммном и механическом изображении предложений и рассуждений». В нем Венн предложил изменить диаграмму Эйлера, чтобы она стала более полной визуализацией вероятности, требуя изображения всех возможностей. Однако название «диаграмма Венна» было придумано американским философом Кларенсом Ирвингом лишь несколько лет спустя, в 1918.

      С тех пор диаграмма Венна была центром обширных исследований многих других известных ученых, в том числе А.В.Ф. Эдвардс, Бранко Грюнбаум и Генри Джон Стивен Смит. Они стали важной визуализацией для математических исследований, будь то традиционная диаграмма с тремя кругами или более сложные версии, включающие сферы и эллипсы, для таких областей, как теория множеств, теория логики, вращательная симметрия и изучение сложных фигур и чисел, таких как тессеракт.

      Что НЕ является диаграммой Венна?

      Из-за такой простой диаграммы Venn неправильно маркировали на протяжении многих лет. На самом деле, чаще всего мы теперь ссылаемся почти исключительно на диаграммы Эйлера, когда обсуждаем «Венна».

      Диаграммы Венна, по сути, представляют собой более конкретный тип диаграмм Эйлера. Вот удобная диаграмма Эйлера, визуально описывающая, что все диаграммы Венна являются диаграммами Эйлера, но не все диаграммы Эйлера являются диаграммами Венна:

      Золотое правило Венна состоит в том, что он всегда визуализирует все возможности, даже если ни одна из данных не вписывается в эти пересечения, тогда как диаграмма Эйлера визуализирует только те категории, для которых есть данные. Если бы мы заставили приведенную выше диаграмму соответствовать параметрам диаграммы Венна, нам пришлось бы представить, что существуют некоторые диаграммы Венна, которые не являются диаграммами Эйлера. Если поместить это в контекст, это все равно, что заявить, что некоторые животные не являются пингвинами, и, следовательно, некоторые пингвины не являются животными.

      Чтобы показать эту разницу на практике, давайте представим, что трое друзей сравнивают свою домашнюю работу. Люси подготовила два домашних задания: одно по «Прокариотам и эукариотам» по естественным наукам и одно по «Вероятности и статистике» по математике. Анвар также написал домашнюю работу по математике «Вероятность и статистика», но выбрал для науки тему «Клетки животных и клетки растений». вообще

      Если оранжевый набор данных представляет Янса, розовый набор данных представляет Анвара, а синий набор данных представляет Люси, мы можем организовать эту информацию двумя различными способами.

      Диаграмма Эйлера показывает только возможности, для которых у нас уже есть фактические данные. Мы знаем, что Анвар и Люси сделали домашнюю работу по математике по теме «Вероятность и статистика», и пересечение показывает это. Мы знаем, что Янс не создавал домашних заданий по математике, как Анвар и Люси, и что они не выбирали ту же тему, что и он, для естественных наук, поэтому набор данных Янса не пересекается ни с каким другим.

      На диаграмме Венна показана одна и та же информация — темы, которые трое друзей изучили по отдельности, и тема по математике, которую выбрали Анвар и Люси. Однако у нас также есть пересечения, которые лишены данных, но представляют 90 200 всех 90 201 других возможностей.

      Оранжевый перекресток слева представляет собой случай, когда Янс вместе с Анваром помнит свою домашнюю работу по математике, а также по естественным наукам, но в этом гипотетическом результате Люси забывает выполнить работу по математике. Зеленый перекресток справа предполагает ту же судьбу для Анвара. Центральный перекресток показывает возможность того, что все трое друзей не забудут поработать над «Вероятностью и статистикой» по математике, а также над различными темами по естественным наукам, и никто не будет наказан…

      Готовы создать собственную диаграмму Венна? Вот небольшой ярлык на страницу Vizzard специально для вас.

      Для построения диаграммы Венна между двумя наборами данных должны быть как общие черты, так и различия. Если вся классифицированная информация идентична, ваша диаграмма будет выглядеть так:

      Даже с двумя наборами данных, если содержащаяся информация не имеет различий, две кривые будут полностью перекрываться, создавая только один круг.

      Противоположная проблема, два набора данных без общих черт, также не создадут диаграмму Венна (но вполне приемлемую диаграмму Эйлера, показывающую непересекающихся наборов ).

      Удивительно, но то, что выглядит столь же знакомым, как приведенный ниже пример, математически не считается диаграммой Венна.

      Как объяснялось ранее, одно из ключевых требований к диаграмме Венна состоит в том, что она должна отображать все возможные логические результаты. Поскольку в приведенном выше примере нет пересечения, в котором встречаются только желтый и красный или только синий и оранжевый, его нельзя считать полным изображением универсальной логики.

      Резюме

      Подводя итог, диаграмма Венна:

      • Тип диаграммы Эйлера
      • Диаграмма с двумя или более наборами данных, изображенная внутри окружностей
      • Используется для визуализации различий и общих черт между этими наборами данных
      • Используется для выработки всех логических выводов
      • Невероятно быстрое производство с использованием диаграммы Венна от Vizzlo Vizzard

      Vizzlo очень прост, и вы можете попробовать его бесплатно.

      Начните создавать собственные диаграммы и бизнес-графику.

      Попробуйте Vizzlo бесплатно

      Узнайте больше
      • Как создать диаграмму Венна в Google Slides
      • Создание диаграммы Венна в Excel

      Хотите создать собственную диаграмму Венна?

      Создать диаграмму Венна с помощью Vizzlo легко и быстро. Попробуйте бесплатно!

      Создать бесплатный аккаунт

      Продукт
      Дом
      Создать
      Цена
      Программы
      Список изменений
      Откройте для себя
      Галерея примеров
      Руководство по визуализации данных
      Блог
      Справочный центр
      Решение
      Консультанты
      Руководители проектов
      Предприятия
      Команды
      Разработчики
      Компания
      О нас
      Карьера
      Свяжитесь с нами
      Филиалы
      Безопасность
      Подписывайтесь на нас

      Знакомство с символами диаграммы Венна — с примерами

      Когда вы оглядываетесь на диаграммы Венна, созданные вами в начальной школе, вы, вероятно, вспоминаете о том, какие типы шоколадных батончиков понравились вам и вашим друзьям, или о сравнении ваших любимых героев фильмов. Хотя вы, возможно, думали, что ваши дни построения диаграмм Венна давно позади, эти инструменты на самом деле полезны во взрослой жизни. На самом деле, символы диаграммы Венна, которые вы так хорошо знаете, — это не просто творческая прихоть.

      Математики и смежные специалисты постоянно используют диаграммы Венна для представления сложных взаимосвязей и решения математических задач. Конечно, темы на профессиональных диаграммах обычно не шоколадные батончики или герои фильмов. И есть еще много чего, что вам нужно понять, чтобы использовать их эффективно.

      Чтобы полностью погрузиться в мир профессиональных диаграмм Венна, вы должны иметь базовое представление о разделе математической логики, называемом «теорией множеств», и связанных с ним символах и обозначениях. Используя теорию множеств, исследователи и математики установили основы многих математических понятий. Например, различные наборы структур, отношений и теорем могут применяться к различным областям изучения, включая топологию, абстрактную алгебру и дискретную математику.

      Используя основы, которые мы рассмотрим здесь, вы также можете начать использовать диаграммы Венна более сложными способами.

      Символы диаграммы Венна

      Хотя в теории множеств существует более 30 символов, вам не нужно запоминать их все, чтобы начать. На самом деле, следующие три являются идеальной основой.

      Символ объединения ∪

      Диаграммы Венна состоят из ряда перекрывающихся кругов, каждый из которых представляет категорию. Чтобы представить объединение двух множеств, мы используем символ ∪ — не путать с буквой «u».

      В приведенном ниже примере у нас есть кружок A зеленого цвета и кружок B фиолетового цвета. Эта диаграмма представляет объединение A и B, которое мы обозначаем как A ∪ B.

      Давайте на мгновение вернемся к тем школьным дням на примере шоколадных батончиков. Если бы в круг A вошли люди, которым нравятся батончики Snickers, а в круг B были бы включены люди, которым нравятся батончики 3 Musketeers, то A ∪ B представлял бы людей, которым нравятся Snickers, 3 Musketeers или и то, и другое.

      Символ пересечения ∩

      Область, в которой пересекаются два множества, — это место, где объекты относятся к обеим категориям. На нашей примерной диаграмме бирюзовая область (где зеленый и фиолетовый перекрываются) представляет собой пересечение A и B, которое мы обозначаем как A ∩ B.

      На этом перекрестке мы обычно встречаем людей, которые любят и «Сникерс», и «Три мушкетера».

      Символ дополнения A

      c

      Категории, не представленные в наборе, называются дополнением набора. Чтобы представить дополнение множества A, мы используем символ A c .

      Для представления абсолютного дополнения множества (т. е. всего, что не входит в множество) мы используем уравнение A c = U \ A, где буква «U» представляет данную вселенную. Это уравнение означает, что все во Вселенной, кроме А, является абсолютным дополнением А в U.

      Серая часть нашей примерной диаграммы представляет все, что находится за пределами A.

      Используя наш пример с шоколадным батончиком, это будет представлять всех, кто не любит Snickers.

      Другой пример

      Теперь попробуем новый пример. Допустим, мы планируем вечеринку на работе и пытаемся выяснить, какие напитки подать. Мы спрашиваем трех человек, какие напитки они любят. Вот ответы, которые мы получаем:

      Напиток А Б С
      Вино х х х
      Пиво х х
      Мартини х х
      Старомодный х х
      Ром и кола
      Джин-тоник

      Используя диаграмму Венна с тремя кругами, мы можем охватить все возможности. Каждый человек представлен кружком, обозначающим их буквами A, B и C. Используя символ ∩, мы можем показать, где должны располагаться пересечения между множествами.

      Когда мы заполняем диаграмму нашими данными, мы размещаем каждый объект в соответствии с формулами, которые мы указали выше. Например, мы помещаем мартини в область B ∩ C, потому что респонденты B и C указали, что они им нравятся. Поскольку ром с колой и джин с тоником никто не отбирал, они не входят ни в какой круг. Однако, поскольку они все еще существуют и доступны во вселенной, мы можем поместить их в пустое пространство.

      Вот наша окончательная диаграмма:

      Очевидно, вино — лучший выбор для нашей предстоящей вечеринки. Пиво, мартини и старомодные напитки могут быть хорошими вторичными напитками, но они, вероятно, не должны подавать ром с колой или джин с тоником.

      Примеры диаграмм Венна

      Глядя на эти примеры и все символы диаграмм Венна, которые вы выучили, вы сможете погрузиться в создание наглядных материалов, которые помогут вашей команде. Используйте серию шаблонов диаграмм Венна на Cacoo в качестве отправной точки.

      Вот еще несколько примеров для ознакомления по мере продолжения:

      Как читать диаграмму Венна

      Теперь, когда вы знаете, как построить диаграмму Венна и использовать официальную терминологию и символы, вы должны понимать как правильно читать один.

      С помощью обратного проектирования вы можете использовать информацию, уже представленную на диаграмме, чтобы увидеть, куда пойдут символы и уравнения, которые мы изложили. Независимо от того, сколько вариантов вы добавите, вы будете знать, как определить сходства или предпочтения, а также различия между элементами, которые находятся внутри или вне диаграммы.

      Теория множеств

      Хотя мы могли бы очень подробно изучить теорию множеств (всегда есть чему поучиться), подходящим способом завершить урок по диаграммам Венна будет изучение теории, стоящей за ними.

      Набор — это группа или набор вещей, также известных как элементы. Эти элементы действительно могут быть чем угодно. В приведенном выше примере набор состоит из вариантов, которые безымянная группа делает для своих предпочтений в напитках.

      В теории множеств мы бы вместо этого записали это как уравнение, перечислив все элементы в фигурных скобках:

      {человек 1, человек 2, человек 3, человек 4, …}

      Начиная с вопроса примера какой напиток они предпочитают, эти люди в конечном итоге делятся на группы по своему выбору:

      • Старомодный = {X человек}
      • Мартини = {X человек}
      • Пиво = {X человек}
      • Ром и кола = {X человек}
      • Джин с тоником = {X человек}

      Поскольку мы предлагаем пять различных вариантов напитков, мы получаем пять отдельных наборов, которые затем представлены на диаграмме Венна.

      В чем разница между диаграммами Венна и диаграммами Эйлера?

      Если вы когда-нибудь видели диаграмму Эйлера, вам может быть интересно, не являются ли диаграммы Венна просто тем же самым понятием под другим названием. Хотя они похожи и оба основаны на теории множеств, они представляют разные типы отношений.

      Диаграммы Венна отображают все возможные логические связи между набором элементов. И, как мы уже обсуждали, получившаяся диаграмма может содержать пустые места, в которых нет отношений. Диаграммы Эйлера, или эйлеровы круги, изображают отношения реального мира и подмножества категорий, которые существуют внутри каждого из них. На диаграммах Эйлера нет пустых пересечений, так как они показывают только определенные отношения.

      ResearchGate

      Простейшая форма диаграммы Эйлера обычно включает два круга рядом друг с другом, в каждом из которых указана категория. Меньшие круги подмножества входят в каждую категорию. Например, если вы сравнивали обувь, категории могли включать «ботинки» и «кроссовки». Возможные подмножества ботинок включают «высокие каблуки» и «снег», в то время как подмножества кроссовок могут включать «беговые» и «высокие».

      Диаграммы Эйлера могут иметь множество пересекающихся окружностей, но они не обязательны, если категории не имеют общих подмножеств. Возвращаясь к предыдущему примеру, «клин» будет подмножеством как «ботинок», так и «кроссовок», но «Челси» будет применяться только к «ботинкам».

      Одна из причин, по которой диаграммы Венна и Эйлера кажутся похожими, заключается в том, что они имеют родственное происхождение. Когда Джон Венн опубликовал свои теории диаграмм Венна в 1881 году, он назвал их эйлеровыми кругами, основанными на открытиях Леонарда Эйлера 18-го века.

      Заключительные мысли

      Здесь мы остановились на основных примерах для ясности, но существует гораздо больше информации, которую вы можете использовать для более глубокого понимания теории множеств. На самом деле статья Стэнфордской энциклопедии по теории множеств — отличное место для начала.

      По мере того, как вы изучаете все больше заданных взаимосвязей, визуализация вашей работы с помощью диаграмм Венна — это мощный и простой способ легко передать эти взаимосвязи.

      Когда вы будете готовы приступить к созданию собственных диаграмм Венна, воспользуйтесь нашим облачным инструментом для создания диаграмм Cacoo. Наша библиотека фигур может помочь вам легко создавать диаграммы с нуля, или вы можете начать с одного из наших сотен готовых шаблонов, просто вставьте свою информацию и приступайте к работе.

       

      Этот пост был первоначально опубликован 11 сентября 2018 г. и последний раз обновлен 26 июля 2020 г.

      VennDiagram: пакет для создания диаграмм Венна и Эйлера с широкими возможностями настройки в R | BMC Биоинформатика

      • Программное обеспечение
      • Открытый доступ
      • Опубликовано:
      • Ханбо Чен 1 и
      • Пол Бутрос 1  

      Биоинформатика BMC том 12 , номер статьи: 35 (2011) Процитировать эту статью

      • 106 тыс. Доступов

      • 1174 Цитаты

      • 16 Альтметрический

      • Сведения о показателях

      Abstract

      Background

      Визуализация ортогональных (непересекающихся) или перекрывающихся наборов данных является обычной задачей в биоинформатике. Существует несколько инструментов для автоматизации создания широко настраиваемых диаграмм Венна и Эйлера с высоким разрешением в статистической среде R. Чтобы восполнить этот пробел, введем VennDiagram , пакет R, который позволяет автоматически генерировать настраиваемые диаграммы Венна с высоким разрешением, содержащие до четырех наборов, и диаграммы Эйлера, содержащие до трех наборов.

      Результаты

      Пакет VennDiagram предлагает пользователю возможность настраивать практически все аспекты сгенерированных диаграмм, включая размеры шрифта, стили и расположение меток, а также общий поворот диаграммы. Мы внедрили масштабированные диаграммы Венна и Эйлера, которые повышают графическую точность и визуальную привлекательность. Диаграммы создаются в виде файлов TIFF высокого разрешения, что упрощает процесс создания показателей качества публикации и облегчает интеграцию с установленными конвейерами анализа.

      Выводы

      Пакет VennDiagram позволяет создавать высококачественные диаграммы Венна и Эйлера в статистической среде R.

      История вопроса

      Визуализация сложных наборов данных становится все более важной частью биологии. Многие эксперименты включают интеграцию нескольких наборов данных для понимания взаимодополняющих аспектов биологии. Эти перекрывающиеся результаты можно визуализировать несколькими способами, включая текстовые таблицы (например, двусторонние таблицы), сетевые диаграммы [1, 2] и, в некоторых случаях, тепловые карты [3, 4]. Диаграммы Венна находят все более широкое применение из-за их привычности, простоты интерпретации и графической простоты. Для целей данной публикации диаграммы Венна можно определить как диаграммы, использующие простые геометрические формы, такие как круги и эллипсы, для отображения всех 2 n -1 возможные области, созданные взаимодействием n множеств. Использование простых геометрических фигур снижает сложность и размер фигур по сравнению с занимающими много места таблицами или сетевыми схемами.

      Однако, несмотря на эту популярность, в настоящее время существует несколько пакетов для создания диаграмм Венна в широко используемой статистической среде R. Эти пакеты ограничены в своих возможностях генерировать диаграммы Венна с высоким разрешением и качеством публикации, поскольку они допускают небольшую настройку цветов, типов линий, размещения меток и шрифта меток. Многочисленные особые случаи обрабатываются неправильно, и выходные данные обычно не имеют формат файлов TIFF с высоким разрешением и качеством публикации. Существуют другие локальные или веб-программы, не основанные на R, способные генерировать диаграммы Венна, такие как Venny [5], BioVenn [6], ConSet [7] и VennMaster [8]. Все они страдают некоторыми из недостатков, перечисленных выше. Кроме того, интеграция в стандартные статистические/вычислительные конвейеры на основе R, такие как широко используемые библиотеки BioConductor статистической среды R [9].] является жизнеспособным, но технически не тривиальным.

      Кроме того, если некоторые пересекающиеся или непересекающиеся области на диаграмме Венна не существуют, может оказаться более желательным другой класс диаграмм, называемый диаграммами Эйлера. Диаграммы Эйлера эквивалентны диаграммам Венна, когда существуют все пересекающиеся и непересекающиеся области. Однако области, содержащие нулевые элементы, показаны на диаграммах Венна (по определению), тогда как диаграммы Эйлера показывают только ненулевые области. Во многих случаях диаграммы Эйлера дополнительно уменьшают сложность фигур, повышают графическую точность и улучшают общую читаемость по сравнению с диаграммами Венна. К сожалению, почти все существующие пакеты не могут генерировать диаграммы Эйлера с качеством публикации в R, хотя VennEuler создает диаграммы Эйлера.

      Для решения этих проблем мы представляем VennDiagram , пакет R для создания настраиваемых диаграмм Венна с высоким разрешением, содержащих до четырех наборов, и диаграмм Эйлера, состоящих из двух или трех наборов, в статистической среде R.

      Реализация

      Пакет VennDiagram был разработан и предназначен для статистической среды R. Среда R имеет открытый исходный код и доступна в Интернете по Стандартной общественной лицензии GNU (GPLv2). R был выбран из-за его природы с открытым исходным кодом, универсальных функций и общих предпочтений в сообществе биоинформатики. Использование R должно облегчить интеграцию с существующими конвейерами анализа данных. Весь код был разработан и протестирован с использованием версии 2.12.1 (32-разрядная и 64-разрядная версии) R. Пакет VennDiagram доступен в виде дополнительных файлов 1 (файл .tar.gz для Linux) и 2 (файл .zip для Windows).

      VennDiagram использует пакет сетки для графики. Пакет сетки является базовым (стандартным) пакетом, доступным во всех установках, и предлагает большую маневренность, чем графика R по умолчанию, с точки зрения графических параметров и наличия изменяемых объектов сетки. VennDiagram использует эти функции для динамического растяжения/сжатия диаграмм в соответствии с размерами выходного файла и предлагает большое количество графических опций.

      Результаты

      Почти все графические опции в 9Пакет 0034 VennDiagram был параметризован и стал настраиваемым. Значения по умолчанию были выбраны для создания разумных диаграмм, поэтому в простых случаях не требуется высокий уровень настройки. На рис. 1 показано разнообразие доступных параметризаций. Существуют четыре основные группы графических параметров: форма-заливка, форма-линия, метки и заголовки. Форма-заливка относится к цветам внутри каждого круга или эллипса. Можно использовать все цвета, доступные в среде R, а альфа-смешивание можно настроить для каждой фигуры. Линии фигуры — это линии, окружающие каждый круг или эллипс. Они могут полностью отсутствовать (рис. 1C), быть сплошными (рис. 1A и 1B) или иметь любой другой доступный тип R-линий (рис. 1D). Их цвет можно менять, и каждая фигура может иметь отдельный набор параметров. Метки относятся как к заголовкам, описывающим каждый круг или эллипс, так и к числам внутри них. Опять же, их можно настроить с точки зрения цвета, типа шрифта и размера шрифта с любым доступным параметром R. Положения меток заголовков также можно настроить. Заголовки, которые включают основной заголовок и подзаголовок, как показано на рисунке 1D, также можно настраивать так же, как и метки.

      Рисунок 1

      Четыре типа диаграмм Венна, нарисованные пакетом VennDiagram . A) Диаграмма Венна с одним набором, показывающая элементарные настраиваемые функции, такие как размер шрифта метки, шрифт метки и заливка формы. B) Диаграмма Венна с двумя наборами, показывающая более продвинутые функции, такие как масштабирование, индивидуальные характеристики заполнения формы и размещение отдельных подписей. C) Диаграмма Венна с тремя наборами, показывающая другой тип контурной линии («прозрачный») и «текстовый» вариант размещения метки заголовка, где метки заголовков прикреплены к меткам областей. D) Диаграмма Венна с четырьмя наборами, показывающая комбинацию всех предыдущих функций плюс возможность настройки заголовков. Код для создания всех показанных здесь диаграмм включен в дополнительный файл 3.

      Полноразмерное изображение

      Помимо этих конкретных графических элементов, VennDiagram также предлагает множество общих параметров, таких как масштабирование, вращение или инверсия диаграмм. Масштабирование диаграмм реализовано с целью отображения диаграмм Венна, где графические размеры частичных областей (областей, ограниченных со всех сторон кривыми и не поддающихся дальнейшему подразделению) фактически соответствуют числовым значениям количества элементов внутри каждой области. Масштабирование диаграмм Венна с двумя наборами и выбранного количества диаграмм Венна с тремя наборами возможно — ниже мы обсудим проблемы, связанные с тем, чтобы сделать это возможным для всех случаев с тремя наборами. Использование масштабирования иногда может привести к тому, что перекрывающиеся области будут слишком малы для числовых меток. На рис. 2, строка 1, столбец 1 показана программно сгенерированная соединительная линия, которая позволяет специально обрабатывать этот случай. Реализовано автоматическое распознавание большого количества диаграмм Эйлера, но этот режим можно отключить с помощью параметра вызова функции, чтобы вместо этого строить стандартные диаграммы Венна. На рис. 2, строка 1, столбцы 2 и 3 показаны два примера двухмножественных диаграмм Эйлера. В строках 2 и 3 показано подмножество реализованных диаграмм Эйлера с тремя наборами, а в строке 3 показаны диаграммы масштабируемого разнообразия. Отметим, что рисунок 2 представлен в черно-белом цвете, чтобы подчеркнуть композиционные различия между диаграммами, но графические параметры и настройки, использованные на рисунке 1, полностью доступны. В то время как VennDiagram по умолчанию записывает графику в файлы TIFF с высоким разрешением, если для параметра имени файла установлено значение NULL, необработанный объект сетки может быть возвращен и использован в любом графическом режиме, доступном в R. Код для создания всех рисунков приведен в дополнительном файле 3. а пример всех доступных параметризаций показан в дополнительном файле 4.

      Рисунок 2

      Избранные частные случаи диаграммы Венна и диаграммы Эйлера, нарисованные пакетом VennDiagram . Строка 1, столбец 1: автоматически рисуемые настраиваемые линии, которые оптимизируют отображение частичных областей, когда отдельные частичные области становятся слишком маленькими на диаграммах Венна, состоящих из двух наборов. Строка 1, столбец 2: диаграмма Эйлера с двумя наборами, показывающая полное включение одного из наборов. Строка 1, столбец 3: диаграмма Эйлера с двумя множествами, показывающая два различных множества. Строка 2, столбец 1: диаграмма Эйлера с тремя множествами, где одно множество не имеет дискретных элементов. Строка 2, столбец 2: диаграмма Эйлера с тремя множествами, в которой одно множество не имеет дискретных элементов, полностью включено в одно из двух других множеств. Строка 2, столбец 3: диаграмма Эйлера с тремя множествами, где два множества не имеют дискретных элементов и включены в большее третье множество. Строка 3, столбец 1: диаграмма Эйлера с тремя наборами, показывающая полное включение двух наборов, отличных от третьего набора. Строка 3, столбец 2: диаграмма Эйлера с тремя наборами, где один набор полностью включен в другой набор, который сам полностью включен в третий набор. Строка 3, столбец 2: диаграмма Эйлера с тремя множествами, показывающая три различных множества. Код для создания всех показанных здесь диаграмм включен в дополнительный файл 3.

      Изображение полного размера

      Обсуждение

      При разработке пакета VennDiagram было обнаружено, что невозможно нарисовать точные, масштабированные диаграммы Венна с тремя наборами, используя круги. Эта загадка иллюстрируется следующим сценарием. В системе двух окружностей A и B расстояния между центрами окружностей d AB можно определить, если площади (A A и A B соответственно) и площадь пересечения (A A ∩ A B ) известны. Это возможно, потому что в системе с двумя кругами единственное значение A A ∩ A B соответствует уникальному значению для d AB . Следовательно, система из трех окружностей A, B и C, d AB , d BC , d AC может быть вычислена до тех пор, пока A A , A B , A C , A A ∩ A B , A A ∩ A C , A B ∩ A C все известны. Однако д AB , d BC , d AC составляют уникальный треугольник, подразумевая, что диаграмму Венна можно нарисовать, даже не зная общего пересечения A A ∩ A B ∩ A C . Другими словами, размер перекрытия между всеми тремя кругами не меняет представления масштабированных диаграмм Венна — площадь не меняется, даже если одна система имеет нулевое общее пересечение (т.е. A A ∩ A B ∩ A С = 0)! Эта головоломка возникает из-за (произвольного) выбора кругов для представления размера набора, что уменьшает степень свободы на одну. Уникальные решения можно определить, используя эллипсы или многоугольники для рисования диаграмм Венна, но полученные диаграммы потеряют мгновенную узнаваемость и привычность, связанные с круговыми диаграммами Венна, что лишает возможности удобного отображения информации. Некруговые диаграммы также потребуют итерационных алгоритмов для вычисления положения и размеров фигур, что значительно увеличивает вычислительную нагрузку, как обсуждалось другими [10]. Следовательно, масштабирование трехмножественных диаграмм Венна отключено в Пакет VennDiagram . Точно так же диаграммы Венна, содержащие более четырех наборов [11, 12], не были реализованы в пакете VennDiagram , поскольку они стали слишком сложными для интуитивной визуализации.

      Общее предостережение при использовании диаграмм Эйлера состоит в том, что, хотя они уменьшают графическую сложность некоторых диаграмм Венна, их нетрадиционные формы в некоторых случаях также могут быть менее узнаваемыми. Когда присутствуют пустые области, пользователю необходимо выбирать между знакомыми диаграммами Венна и повышенной точностью диаграмм Эйлера. На рис. 3 показана ситуация, когда в зависимости от предпочтений пользователя может подойти либо диаграмма Венна, либо диаграмма Эйлера.

      Рисунок 3

      Наглядное сравнение диаграммы Эйлера и диаграммы Венна для одних и тех же гипотетических наборов . А) Диаграмма Эйлера показывает только ненулевые площади и поэтому может быть более точной графически. Б) На диаграмме Венна несуществующая область показана как область с нулевым содержимым. Хотя это не является графически точным, оно сохраняет узнаваемость диаграммы Венна.

      Изображение полного размера

      Пакет VennDiagram обрабатывает все двухмножественные диаграммы Эйлера и большинство всех мыслимых трехмножественных диаграмм Эйлера. Диаграммы Эйлера с тремя наборами, которые нельзя нарисовать с помощью кругов или эллипсов, не поддерживаются. Например, диаграмму Эйлера для случая, когда два непересекающихся набора составляют третий набор, нельзя нарисовать с помощью кругов и эллипсов, хотя ее можно нарисовать с помощью многоугольников. Этот тип рисунка не имеет готового аналитического макета и требует итеративной подгонки; диаграммы Эйлера, требующие многоугольников, недоступны, но для этих нескольких неподдерживаемых случаев доступны стандартные диаграммы Венна.

      По сравнению с другими программами, способными генерировать диаграммы Венна (таблица 1), преимущества пакета VennDiagram включают:

      • Рисование диаграмм Эйлера с использованием кругов и/или эллипсов с двумя или тремя наборами

      • Расширенные возможности настройки для создания более элегантных диаграмм

      • Доступность в широко используемой статистической среде R

      • Создание файлов TIFF с высоким разрешением, которые являются стандартными в публикациях

      Таблица 1 Сравнение функций различных программ, способных генерировать диаграммы Венна.

      Полноразмерная таблица

      Выводы

      Пакет VennDiagram повышает простоту использования и степень настраиваемости при создании диаграмм Венна в контексте биоинформатики. В то время как другие инструменты предлагают большую часть функций, представленных здесь, реализация всех функций вместе в широко используемой статистической среде R будет способствовать использованию автоматически сгенерированных диаграмм Венна в вычислительных конвейерах.

      Доступность и требования

      Сам пакет VennDiagram доступен как дополнительные файлы 1 и 2 и будет отправлен в CRAN — глобальное хранилище пакетов R. VennDiagram требует R (>2.12.1) и пакет grid для R. : Сетевой анализ, визуализация и построение графиков Торонто. Биоинформатика 2009, 25(24):3327–3329. 10.1093/биоинформатика/btp595

      PubMed Central КАС Статья пабмед Google ученый

    2. Мерико Д., Гфеллер Д. , Бадер Г.Д.: Как визуально интерпретировать биологические данные с помощью сетей. Nat Biotechnol 2009, 27(10):921–924. 10.1038/nbt.1567

      Центр PubMed КАС Статья пабмед Google ученый

    3. Boutros PC, Okey AB: Неконтролируемое распознавание образов: введение в причины и причины кластеризации данных микрочипов. Бриф Биоинформ 2005, 6(4):331–343. 10.1093/наг/6.4.331

      CAS Статья пабмед Google ученый

    4. Verhaak RG, Sanders MA, Bijl MA, Delwel R, Horsman S, Moorhouse MJ, van der Spek PJ, Lowenberg B, Valk PJ: HeatMapper: мощная комбинированная визуализация корреляций профиля экспрессии генов, генотипов, фенотипов и характеристик образцов . BMC Bioinfo 2006, 7: 337. 10.1186/1471-2105-7-337

      Статья Google ученый

    5. Оливерос Дж. : Венни. Интерактивный инструмент для сравнения списков с диаграммами Венна. для сравнения и визуализации биологических списков с использованием диаграмм Венна, пропорциональных площади. BMC Genomics 2008, 9: 488. 10.1186/1471-2164-9-488

      PubMed Central Статья пабмед Google ученый

    6. Ким Б., Ли Б., Сео Дж.: Визуализация соответствия множеств с помощью матриц перестановок и веерных диаграмм. Interact Comput 2007, 19(5):630–643. 10.1016/j.intcom.2007.05.004

      PubMed Central Статья пабмед Google ученый

    7. Кестлер Х.А., Мюллер А., Краус Дж.М., Бухгольц М., Гресс Т.М., Лю Х., Кейн Д.В., Зееберг Б.Р., Вайнштейн Дж.Н.: VennMaster: диаграммы Эйлера, пропорциональные площади, для функционального анализа GO микрочипов. БМС Биоинфо 2008, 9: 67. 10.1186/1471-2105-9-67

      Артикул Google ученый

    8. «>

      Джентльмен Р.С., Кэри В.Дж., Бейтс Д.М., Болстад Б., Деттлинг М., Дудойт С., Эллис Б., Готье Л., Ге Ю., Джентри Дж. и др.: Биокондуктор: открытая разработка программного обеспечения для вычислительной биологии и биоинформатики. Геном Биол 2004, 5(10):R80. 10.1186/gb-2004-5-10-r80

      Центр PubMed Статья пабмед Google ученый

    9. Чоу С., Роджерс П. Построение диаграмм Венна и Эйлера, пропорциональных площади, с тремя окружностями. В диаграммах Эйлера 2005: 2005. Париж, Франция; 2005.

      Google ученый

    10. Эдвардс А. Диаграммы Венна из семи наборов с вращательной и полярной симметрией. Комбинаторика, вероятность и вычисления 1998, 7 (2): 149–152. 10.1017/S0963548397003143

      Артикул Google ученый

    11. Schwenk A: Диаграмма Венна для пяти комплектов. Математический журнал 1984, 57 (5): 297–298. 10.2307/2689606

      Артикул Google ученый

    12. Ссылки на скачивание

      Благодарности

      Авторы благодарят всех сотрудников лаборатории Boutros за поддержку, и особенно доктора Кеннета Чу и Дэрила Ваггота за помощь в создании Windows-совместимой версии этого пакета. Это исследование было проведено при поддержке Института исследования рака Онтарио к ПХД за счет финансирования, предоставленного правительством Онтарио. Работа выполнена при финансовой поддержке гранта номер MOP579.03 от Канадских институтов исследований в области здравоохранения (ПХБ и доктору Аллану Б. Оки).

      Информация об авторе

      Авторы и организации

      1. Платформа информатики и биокомпьютеров, Институт исследований рака Онтарио, Центр MaRS, Южная башня, 101 College Street, Suite 800, Торонто, Онтарио, M5G 0A3, Канада

        Hanbo Chen Paul C Boutros

      Авторы

      1. Hanbo Chen

        Посмотреть публикации автора

        Вы также можете искать этого автора в PubMed Google Академия

      2. Paul C Boutros

        Посмотреть публикации автора

        Вы также можете искать этого автора в PubMed Google Scholar

      Автор, ответственный за корреспонденцию

      Пол Си Бутрос.

      Дополнительная информация

      Вклад авторов

      HC и PCB задумали проект. HC написал программное обеспечение, которое HC и PCB протестировали и отладили. ХК написал первый черновик рукописи, которую все авторы отредактировали и одобрили.

      Дополнительный электронный материал

      Дополнительный файл 1: Пакет

      VennDiagram R в виде linux-совместимого файла .tar.gz (GZ 20 КБ)

      Дополнительный файл 2: Пакет

      VennDiagram5 R в формате a 9003 -совместимый файл .zip (ZIP 64 КБ)

      12859_2010_5075_MOESM3_ESM.R

      Дополнительный файл 3: Код для создания всех диаграмм Венна на рисунках 1 и 2. (R 4 КБ)

      Дополнительный файл 4: Иллюстрация параметров доступно в

      Диаграмма Венна . (PPT 2 МБ)

      Оригинальные файлы изображений, представленные авторами

      Ниже приведены ссылки на оригинальные файлы изображений, представленные авторами.

      Оригинальный файл авторов для рисунка 1

      Оригинальный файл авторов для рисунка 2

      Оригинальный файл авторов для рисунка 3 , Это статья в открытом доступе, распространяемая в соответствии с лицензией Creative Commons Attribution License (http://creativecommons.

      Добавить комментарий

      Ваш адрес email не будет опубликован. Обязательные поля помечены *